You are on page 1of 170

I.

OLYMPIC HOÁ HỌC VIỆT NAM:


OLYMPIC HÓA HỌC SINH VIÊN TOÀN QUỐC 2003:
1) Nhà máy khí Dinh Cố xử lý khí thiên nhiên thành hai sản phẩm: khí hóa lỏng chứa trong các
bình thép và khí cung cấp cho khu điện - đạm Phú Mĩ.
a) Hãy cho biết thành phần hóa học chính của mỗi sản phẩm đó.
b) Vì sao không sản xuất chỉ khí hóa lỏng.
2) Dưới đây cho biết nhiệt độ và entanpi của các phản ứng thuận (từ trái sang phải)
Phản ứng ∆H(kJ)
1500oC
2CH4 C2H2 + 3H2O +380,4 (1)
500oC
CH4 + 2O2 CO2 + 2H2O -890,0 (2)
900oC
CH4 + 1/2O2 CO + 2H2 -33,4 (3)
900oC
CH4 + H2O CO + 3H2 +204,8 (4)
900oC
CO + H2O CO2 + H2 -46,0 (5)
o
400 C
CO + 2H2 CH3OH -110,8 (6)
a) Để sản xuất axetilen theo (1) vì sao người ta thêm một lượng nhỏ oxy vào hỗn hợp phản
ứng?
b) Để có chất đầu cho sản xuất metanol theo phản ứng (6) người ta nhiệt phân hỗn hợp metan và
hơi nước theo phản ứng (4). Vì sao trong sản xuất người ta lấy tỉ lệ mol CH4 : H2O = 1 : 2 mà
không phải là 1 : 1? Hỗn hợp sản phẩm thu được gồm những khí nào?
c) Hãy viết phương trình phản ứng của ankan ở nhiệt độ xấp xỉ 900oC với oxy và với nước.
d) Theo dự án nhà máy phân đạm Phú Mĩ, hydro được sản xuất từ khí thiên nhiên. Trong các
phản ứng tạo ra hydro đã cho ở trên, nên sử dụng những phản ứng nào?. Không sử dụng
những phản ứng nào?, vì sao?
e) Có hỗn hợp CO + H2 (sản phẩm phản ứng (3) và (4)). Hãy đề nghị phương pháp tinh chế H2
để có thể đưa vào phản ứng với N2 tạo khí NH3.
BÀI GIẢI:
1) a) Khí hóa lỏng: propan và butan.
Khí cho điện đạm: metan
b) Metan có nhiệt độ hóa lỏng rất thấp. Muốn hóa lỏng phải làm lạnh sâu, nén ở áp suất cao sẽ
không kinh tế bằng để ở dạng khí dẫn bằng đường ống cung cấp cho nhà máy điện đạm.
2) a) Phản ứng (1) thu nhiệt mạnh, ngoài việc cung nhiệt từ bên ngoài cần đốt cháy một ít CH4 theo
phản ứng (3) để luôn đạt được và duy trì nhịêt độ cao 1500oC.
b) Tăng nồng độ (áp suất riêng phần) của H2O để chuyển dịch cân bằng về phía tạo thành sản
phẩm. Nước rẻ tiền hơn metan. Nước còn có tác dụng giảm CO, tăng H2 (phản ứng 5). Hỗn hợp
sản phẩm thu được gồm CO, H2, H2O (dư) và CH4 (dư ít).
c) CnH2n + 2 + (n/2)O2 → nCO + (n + 1)H2
CnH2n + 2 + nH2O → nCO + 2(2n + 1)H2
d) Không nên dùng phản ứng (1), gía thành sẽ rất cao vì:
- Tốn năng lượng → thiết bị chịu nhiệt cao (đắt, mau hỏng)
- 2 mol CH4 mới thu được 3 mol H2 (so với phản ứng 3 và 4 đều kém). Axetilen tuy là nguyên
liệu qúy, đắt nhưng nếu không có nhà máy hóa chất sử dụng tại chỗ thì chỉ dùng làm nhiên
liệu mà thôi, sẽ lãng phí.
Nên sử dụng kết hợp đồng thời 3 phản ứng (3); (4) và (5), tức là CH4; H2O; O2 sẽ có lợi và
các phản ứng thu nhiệt (4) và toả nhiệt (3); (5) bổ sung cho nhau, đều thực hiện ở 900oC đỡ tốn
năng lượng và trang thiết bị. Từ 2 mol CH4, một cách gần đúng cho đến 6 mol H2.
e) Thêm một lượng H2O (có tính theo thành phần của hỗn hợp CO + H2) thực hiện phản ứng (5)
ở 900oC biến CO thành CO2, sau đó làm lạnh tách CO2.
OLYMPIC HÓA HỌC SINH VIÊN TOÀN QUỐC 2003:
1) Người ta tiến hành các phản ứng sau đây để xác định công thức cấu tạo của hợp chất thơm A
(C9H10O):
- Oxy hóa mạnh chất A với KMnO4 đậm đặc thu được hai axit C7H6O2 và C2H4O2.
- Cho A phản ứng với metyl magie bromua rồi thuỷ phân thu được ancol bậc ba (B) có một
nguyên tử cacbon bất đối.
a) Viết công thức cấu tạo và gọi tên A.
b) Hãy cho biết góc quay mặt phẳng ánh sáng phân cực của ancol B bằng 0 hay khác 0, vì sao?
2) Cho A tác dụng với metyl iodua dư trong môi trường bazơ mạnh người ta cô lập được C (C11H14O).
Hãy cho biết tên cơ chế phản ứng. Viết công thức cấu tạo và gọi tên C.
3) Cho ancol B phản ứng với H2SO4 đặc nóng thu được sản phẩm chính là E (C10H12). Dùng công thức
không gian thích hợp biểu diễn cơ chế phản ứng tạo thành E và gọi tên E.
BÀI GIẢI
1) a) Ta có:
[O ]
A ⎯⎯→ C7 H 6O2 + C2 H 4O2 A có nhân benzen, một mạch nhánh, có 1O và một liên kết đôi
axit benzoic axit axetic
CH3

C6H5 C CH2CH3 1) CH3MgBr C6H5 C CH2CH3 (B)


2) H3O+
O OH
A: etylphenylxeton
b) αB = 0 vì CH3MgBr tấn công như nhau vào hai phía nhóm C = O tạo ra hỗn hợp raxemic.
2) Ta có:
CH3

OH- CH3I
C6H5COCH2CH3 C6H5 C C CH3 (C)
SN2
O CH3
tert-butylphenylxeton
3) Cơ chế:
OH H 2O +
H CH3
H CH3 H + (H 2 S O 4 ); t o H CH3 -H 2 O
C 6H 5 CH3
C 6H 5 CH3 C 6H 5 CH3
H H H

-H +
C 6H 5 H
H
C C C 6H 5 CH3
CH3
H 3C CH3

E -2 -p h en y lb u t-2 -en
OLYMPIC HÓA HỌC SINH VIÊN TOÀN QUÓC 2004:
1) Thực hiện dãy biến hóa sau:
+ o
,t o
2-metylpropanal ⎯HCN
⎯⎯→ A ⎯NH ⎯3⎯ → B ⎯H⎯
3O ,t
⎯⎯→ C ⎯⎯⎯→ C7 H11O2 N
Ac 2 O

2) Hoàn chỉnh các phương trình phản ứng và gọi tên sản phẩm:
a) Xiclopentadien + HCl khí:
b) Pent-1-en + NBS, a,s
c) Hexa-1,3,5-trien + Br2 (1: 1)
d) 1-(Brommetyl)-2-metylxiclopenten, đun nóng trong CH3OH
BÀI GIẢI:
1) Công thức cấu tạo các chất là:
H H H H
A: H3C C C CN B: H3C C C CN

CH3 OH CH3 NH2


O
H H H H
C: H3C C C COOH D: H3C C C C

CH3 OH CH3 N O
C

CH3
2) Công thức cấu tạo các sản phẩm:
a)
Cl

3-cloxiclopenten
b) CH3CH2CHBrCH=CH2 (3-brompent-1-en) + CH3CH2CH=CHCH2Br (1-brompent-2-en)
c) CH2=CH-CH=CH-CHBr-CHBr: (5,6-dibromhexa-1,3-dien)
CH2=CH-CHBr-CH=CH-CH2Br: (3,6-dibromhexa-1,4-dien)
BrCH2-CH=CH-CH=CH-CH2Br: (1,6-dibromhexa-2,4-dien)
d)

CH2OCH3 CH2
+
OCH3

CH3 CH3
1-(metoximetyl)-2-metylxiclopenten 1-metoxi-1-metyl-2-metylenxiclopentan
OLYMPIC HÓA HỌC SINH VIÊN TOÀN QUÓC 2004:
Hợp chất A(C6H12N2O2) quang hoạt, không tan trong axit loãng và bazơ loãng, phản ứng với
HNO2 trong nước tạo thành B (C6H10O4). Khi đun nóng B dễ dàng mất nước chuyển thành C (C6H8O3).
Hợp chất A phản ứng với dung dịch brom và natri hydroxit trong nước tạo thành D (C4H12N2), hợp chất
này phản ứng với HNO2 khi có mặt axit clohydric cho metyletylxeton.
1) Hãy xác định công thức cấu tạo của A, B, C, D và gọi tên các hợp chất tạo thành.
2) Hợp chất A có thể có cấu trúc như thế nào? Dùng công thức Fisơ để mô tả.
BÀI GIẢI:
1) A phải là diamit nên có thể viết như sau:
NH2
CONH2 Br2; OH-
C6H12N2O2 C4H8 C4H8
H2O NH2
CONH2

D
D là diamin, deamin hóa khi phản ứng với HNO2 và chuyển vị giống như pinacolin. Như vậy có
thể viết như sau:
H H HNO2 H H H+
H3C C C CH3 H3C C C CH3 CH3COCH2CH3
-H2O
NH2 NH2 OH OH
2,3-diaminobutan
Như vậy hợp chất A là diamit của axit 2,3-dimetylsucxinic có thể tồn tại ở dạng quang hoạt,
phản ứng với axit nitrơ cho ra axit 2,3-dimetylsucxinic.
O
CH3 CH3 CH3

C
H CONH2 H COOH H3C C
o
HNO2 tC
-H2O O
H2NOC H HOOC H H3C C
C
CH3 CH3 H
O
anhydrit-2,3-dimetyl
sucxinic
2) Công thức Fisơ:

CH3 CH3

H CONH2 H2NOC H

H2NOC H H CONH2

CH3 CH3
OLYMPIC HÓA HỌC SINH VIÊN TOÀN QUÓC 2004:
1) Trong khu công nghiệp lọc hóa dầu tương lai, dự kiến có cả nhà máy sản xuất PVC.
a) Hãy đề nghị hai sơ đồ phản ứng làm cơ sở cho việc sản xuất vinylclorua từ sản phẩm
crackinh dầu mỏ và NaCl.
b) Hãy phân tích các ưu, nhược điểm của mỗi sơ đồ, nêu cách khắc phục và lựa chọn sơ đồ
có lợi hơn.
c) Trùng hợp vinylclorua nhờ xúc tác TiCl4-Al(C2H5)3 sẽ thu được PVC điều hoà lập thể có
độ bền cơ nhiệt cao. Hãy cho biết trong mạch polime ấy có trung tâm bất đối không?.
Viết công thức lập thể một đoạn mạch polime ấy.
2) Từ toluen hãy viết phương trình điều chế phlorogluxinol (1,3,5-trihidroxibenzen)
3) Hãy tổng hợp axit glutamic từ axit α-xetoglutaric.
4) Hãy tổng hợp prolin từ axit adipic.
BÀI GIẢI:
o
1) a) A: 2CH 4 ⎯1500
⎯⎯ C
→ C2 H 2 + 3H 2 ⎯HCl
⎯→
⎯ CH 2 = CH − Cl
NaCl + H2O ⎯dpmn
⎯⎯→ Cl2 + H 2 → 2 HCl
o o
B: CH 2 = CH 2 ⎯Cl⎯2 ;500 ⎯⎯ C
→ ClCH 2CH 2Cl ⎯500
⎯⎯ C
→ CH 2 = CH − Cl
b) Phương pháp B cần phải xử lý HCl để thu Cl2 và tránh ô nhiễm.
1 o
2 HCl + O2 ⎯t⎯ ⎯; xt
→ Cl2 + H 2O
2
Chọn phương pháp B vì phản ứng (1) của phản ứng A hiệu suất thấp, tiêu tốn nhiều năng lượng
và gía thành sản phẩm sẽ cao hơn.
c) Có nguyên tử C bất đối:
H Cl
Cl H
Cl H Cl
C

C CH2 CH2
H2 CH2 CH2
2) Qúa trình tổng hợp như sau:
CH3 CH3 COOH

O2N NO2 O2N NO2

HNO3 KMnO4 1)Sn + HCl


H2SO4 2) OH-

NO2 NO2
COOH COOH COOH

H2N NH2 HN NH O O
+H2O

NH2 NH O

HO OH

-CO2

OH

3) Qúa trình tổng hợp như sau:


CH2COOH CH2NH2 CH2NH3+
H2C
H2SO4 H2C H+ H2C

HN3 P, Br2
H2C H2C COOH H2C COOH
CH2COOH C CH
H2
(A) Br
(B)
OH-
NH

COOH
1) SOCl 2
) NH 3
hoặc: HOOC(CH 2 ) 4 COOH ⎯2⎯ ⎯
⎯→ H 2 NCO(CH 2 ) 4 COOH ⎯Br
⎯2 /⎯⎯→ A rồi tiếp tục như trên
KOH
4) Qúa trình tổng hợp như sau:
COOH COOH COOH COOH
CN COOH
CO HCN C
H3O+ C -CO2 CHNH2
NH2 NH2
NH3 to
(CH2)2 (CH2)2 (CH2)2 CH2

COOH COOH COOH CH2

COOH
OLYMPIC HÓA HỌC SINH VIÊN TOÀN QUÓC 2004:
1) Phản ứng của ancol anlylic với nước clo tạo ra C3H7ClO2 (A). Phản ứng của anlyl clorua với
nước clo thì tạo ra C3H6Cl2O (B). A và B khi chế hóa với dung dịch NaOH đều tạo thành
glixerol nhưng A tạo thêm sản phẩm C3H6O2(D). B tạo ra sản phẩm phụ C3H5ClO (E). Trên phổ
hồng ngoại của D và E đều không có vân hấp thụ ở vùng 1500 – 1800 cm-1, nhưng ở phổ của D
có vân hấp thụ mạnh và tù ở 3100 – 3400 cm-1 còn ở phổ của E thì không có:
a) Viết phương trình phản ứng tạo thành A, B và cho biết tính quang hoạt của chúng.
b) A có 2 nhóm OH, hydro ở nhóm OH nào linh động hơn.
c) Xác định công thức cấu tạo của D và E.
d) D, E được tạo thành từ A và B tương ứng như thế nào, có quang hoạt không?
2) Từ dầu mỏ, người ta tách được các hydrocacbon A(C10H16); B(C10H18) và C(C10H18). Cả ba đều
không làm mất màu dung dịch brom và chỉ chứa C bậc hai và ba. Tỉ lệ giữa số nguyên tử CIII : số
nguyên tử CII ở A là 2 : 3; còn ở B và C là 1 : 4. Cả ba đều chỉ chứa vòng 6 cạnh ở dạng ghế.
a) Hãy xác định công thức cấu tạo và viết công thức lập thể của A, B và C.
b) So sánh nhiệt độ nóng chảy của A, B, C và nêu nguyên nhân.
BÀI GIẢI:
1) a) CH2 = CHCH2OH + HOCl → CH2Cl – C*HOH – CH2OH (A): hỗn hợp raxemic.
CH2 = CHCH2Cl + HOCl → CH2Cl – CHOH – CH2Cl (B): không quang hoạt
b) H của nhóm OH ở giữa linh động hơn vì chịu hiệu ứng –I mạnh hơn.
c) Do E không hấp thụ ở vùng 1500 – 1800cm-1 chứng tỏ chúng không chứa liên kết C=C và
C=O suy ra chúng có cấu tạo vòng. D có vân mạng ở 3100 – 3400cm-1 → chứa nhóm –OH, E thì
không.
So sánh thành phần A với D, B với E thấy đều giảm + HCl và chế hóa với NaOH tạo ra
glixerol (qủa thế) nên đây là điều kiện cho phản ứng thế chứ không phảo là cho phản ứng tách
HCl (cần điều kiện phân cực và đun nóng). Như vậy chúng có công thức cấu tạo:
H2C *CHCH OH H C *CHCH Cl
2 2 2

O O
D E
d) Cơ chế của các phản ứng như sau:
H OH- Cl
H2
C
H
C CH2OH
-Cl- H2C
H
C CH2OH
H2C C CH2OH
-H2O
Cl OH O- O
raxemic
H OH- Cl
H2
C
H
C CH2Cl
-Cl- H2C
H
C CH2Cl
H2C C CH2Cl
-H2O
Cl OH O- O
raxemic

2) A, B, C không làm mất màu dung dịch brom → không chứa liên kết bội mà chứa vòng no.
A có 4 CIII, 6 CII; B và C có 2CIII và 8CII.
Công thức cấu tạo của chúng:

A B, C
Công thức lập thể:

A B C
tonc:
A > B > C vì tính gọn gàng giảm theo chiều đó
OLYMPIC HÓA HỌC SINH VIÊN TOÀN QUÓC 2005:
1. Viết công thức Newman và công thức phối cảnh (dùng nét đậm, nét chấm của các hợp chất sau):
H CH3 Br H H CH3
Br CH3 Cl CH3
H3C C C OH H C C Br H
Cl CH3
HO H Cl Cl H H OH
(I) (II) (III) (IV)
2. Viết các đồng phân lập thể của 1 – metyl – 2,3 – dicloxiclopropan.
3. Dùng công thức cấu tạo hoàn thành sơ đồ phản ứng sau:
1) EtMeCHCHO
Cl 2 , as Cl 2 , Fe KOH / EtOH Mg / ete 2) H 3O +
n − C3 H 7C6 H 5 ⎯⎯⎯→ B ⎯⎯⎯→ C ⎯⎯ ⎯ ⎯
(1) ( 2)
⎯→ D ⎯⎯ ⎯→ E ⎯⎯ ⎯ ⎯⎯→ F
( 3) ( 4) ( 5)

Các phản ứng đều xảy ra theo tỉ lệ 1 : 1, các chất từ B đến F đều là sản phẩm chính. Viết ký hịêu
hoặc gọi tên cơ chế các phản ứng (1); (2); (3); (5). Viết tên gọi hợp chất F và cho biết số
đồng phân lập thể của F.
BÀI GIẢI:
1. Công thức Newman và công thức phối cảnh của các hợp chất (I); (II); (III) và (IV):
H Br
CH3 H Br Cl H
H Br
Cl CH3 H3C CH3
C C C C
HO
H3C H Cl
HOH H3C H H OH
Cl
(I) (II) (III) (IV)
2. Các đồng phân lập thể của 1- metyl – 2,3 – dicloxiclopropan có 4 đồng phân.
Nếu hai nguyên tử clo ở vị trí trans thì có hai đối quang, còn nếu hai nguyên tử clo ở vị trí cis thì
tuỳ theo vị trí của nhóm CH3- mà chỉ có thêm hai đồng phân (các đồng phân này có ảnh qua
gương phẳng trùng với chúng, meso):

Cl H H Cl

CH3 H3C

H Cl Cl H

H H

Cl Cl Cl Cl

CH3 H

H H H H

H CH3
3. Các phương trình phản ứng:
H2 H2 hv H2 H
H3C C C + Cl2 H3C C C + HCl

Cl (B)
H2 H Fe H2 H
H3C C C + Cl2 H3C C C Cl + HCl

Cl (B) Cl (C)

H2 H C2H5OH
H3C C C Cl + KOH H3C C C Cl
H H

Cl (C) (D)
+ KCl + H2O
ete
H 3C C C Cl + Mg H3C C C MgCl
H H H H

(D) (E)

O
H
H 3C C C MgCl + C 2H 5 C C
H H

(E) CH3 H

OMgCl

H
H3C C C C C C2H5
H H H

(E) CH3

OMgCl

H
H3C C C C C C2H5 + H+
H H H

CH3

OH

H
H3C C C C C C2H5 + Mg2+ + Cl-
H H H

CH3
(F)
Các phản ứng xảy ra theo cơ chế:
1) SR hoặc thế gốc tự do
2) SE2 hoặc thế electrophin lưỡng phân tử.
3) E1 hoặc tách đơn phân tử.
4) AN hoặc cộng nucleophin.
Tên gọi của F: 2 – Metyl – 1 - [4 – (prop – 1 – enyl)phenyl]butan – 1 – ol.
Số đồng phân lập thể của chất F: 8
OLYMPIC HÓA HỌC SINH VIÊN TOÀN QUÓC 2005:
1) Hợp chất X (C10H16) có thể hấp thụ ba phân tử hydro. Ozon phân khử hóa X thu được axeton,
andehit fomic và 2 – oxopentadial.
a) Viết công thức cấu tạo của X thoã mãn tính chất trên.
b) Hydrat hóa hoàn toàn 2,72g chất X rồi lấy sản phẩm cho tác dụng với I2/NaOH thu được
15,76g kết tủa màu vàng. Dùng công thức cấu tạo của các chất X viết các phương trình
phản ứng (chỉ dùng các sản phẩm chính, hiệu suất coi như 100%, C = 12; H = 1; O = 16).
2) Từ benzen và các hoá chất vô cơ cần thiết khác viết sơ đồ phản ứng điều chế naphtalen bằng 5
giai đoạn.
3) Từ 1 – amino – 1,2 – diphenylpropan hãy điều chế 1,2 – diphenylpropen (được sử dụng các hóa
chất cần thiết).
BÀI GIẢI:
1. Chất X (C10H16) cộng 3H2; sản phẩm phải có công thức C10H22. Theo các sản phẩm ozon phân
suy ra X có mạch hở, có 3 liên kết đôi và tạo ra 2 mol CH2O nên có hai nhóm CH2 = C.
a) Các chất X thỏa mãn:
H3C C C (CH2)2 C CH2 H3C C C (CH2)2 C CH2
H H

CH3 HC CH2 CH3 HC CH2


(X1) (X2)
H3C C C C (CH2)2C CH2
H H

CH3 CH2
(X2)

b) Hydrat hóa X tạo ra ancol có phản ứng iodofom.


MX = 272 nên số mol X là 0,01 mol. Số mol CHI3 = 0,04mol. Vậy sản phẩm hydrat hóa
X phải có hai nhóm CH3-CHOH-. Suy ra chỉ có chất X2 ở trên thỏa mãn. Các phương trình phản
ứng:
OH

H H
H3C C C (CH2)2 C
H
CH2 H2SO4 H3C C C (CH2)2 C CH3
+ 3HOH
CH3 HC CH2 CH3 HC CH3 OH
(X2)
OH OH
OH
H H
H3C C C (CH2)2 C CH3
H
+12NaOH + 8I2 H3C C C (CH2)2 COONa
CH3 HC CH3 OH
CH3 COONa
OH +2CHI3 + 10NaI + 10H2O

2. Điều chế naphtalen:


O
O

O2; V2O5; 400 - 500oC C6H6; AlCl3


(1) O (2)

HOOC

O
(3) HF

Pd,to H2N - NH2/OH-


(5) (4)

3. Điều chế dẫn xuất propen:


H H H H
C6H5 C C C6H5 CH3I du C6H5 C C C6H5 Ag2O; to C6H5 C C C6H5
H

CH3 NH2 CH3 N(CH3)3I CH3

Có thể tiến hành theo sơ đồ sau:


H H H H
C6H5 C C C6H5 HNO ; H O
2 2
C6H5 C C C6H5 H2SO4; to C6H5 C C C6H5
H

CH3 NH2 CH3 OH CH3

OLYMPIC HÓA HỌC SINH VIÊN TOÀN QUÓC 2005:


1. Hợp chất Y (C6H9Obr) phản ứng với metanol trong môi trường axit sinh ra Z (C8H15O2Br). Cho
Z phản ứng với Mg trong ete khan sau đó cho phản ứng tiếp với andehit fomic thu được chất L.
Thủy phân L trong môi trường axit thu được M. Dehydrat hóa M thu được 2 –
vinylxiclopentanon.
a) Hãy xác định công thức cấu tạo của Y và viết các phương trình phản ứng.
b) Nếu muốn điều chế M đi từ Y, có nhất thiết phải qua các giai đoạn như trên không?. Vì sao?
2. Cho biết công thức cấu tạo và giải thích sự tạo thành sản phẩm trong các phản ứng sau:
a) N – axetylalanin + SOCl2 → C5H7NO2 + …
b) Tos – NH – CH2COOCOOC2H5 + NaOH → C18H18N2O6S2 + …
BÀI GIẢI:
1. Chất Z có chứa nhiều hơn chất Y hai nguyên tử C, 6 nguyên tử H và 1 nguyên tử O, suy ra chất
Y phản ứng với 2CH3OH và tách loại 1 phân tử H2O dẫn đến sự hình thành axetal metylic từ Y.
Chất M sau khi dehydrat hóa tạo ra 2 – vinylxiclopentanon. Vậy M phải có công thức cấu tạo:

O O
suy ra Y:

H2
H2C C OH H2C Br
a) Các phương trình phản ứng:
OMe
+
H
O + 2CH3OH + H2O
OMe

H2C Br H2C Br
(Y) (Z)
OMe OMe
ete
+ Mg
OMe OMe

H2C Br H2C MgBr

OMe OMe

+ CH2O
OMe OMe

H2
H2C MgBr H2C C OMgBr
(L)

OMe

+ 2H2O + H+ O +2CH3OH + Mg2+ + Br-


OMe

H2 H2
H2C C OMgBr H2C C OH
(M)

Al2O3; to
O O + H2O

H2
H2C C OH HC CH2

b) Khi điều chế M đi từ Y nhất thiết phải đi qua các giai đoạn trên. Vì:
- Nếu không đi qua giai đoạn tạo ra Z thì hợp chất cơ magie sẽ phản ứng với nhóm
C = O.
- Nếu đưa thêm 1C qua phản ứng với KCN thì nhóm C = O cũng có thể tham gia
phản ứng với KCN.
2. a) Hợp chất C5H7NO2 thuộc loại azalacton có công thức cấu tạo như sau:
O

O C

C CH CH3
H3C
N
Sự tạo thành:
O O O O
H -SOCl2 H
H3C C NH C C H3C C NH C C
-HCl; SO2
OH CH3 Cl
CH3

O
O
OH
O C Cl
-HCl
C CH CH3
C CH CH3 H3C
H3C N
N
b) Hợp chất C18H14N2O6S2 phải thuộc loại dẫn xuất dixetopiperazin. Công thức cấu tạo của chất đó là:
O
H2
C O
H3C S N C O

O C N S CH3
O C
H2
O
Sự tạo thành hợp chất trên như sau:
O O O O

H H2 - -H2O H2
Tos N C C O C O Et + OH Tos N C C O C O Et

O O
H2
H2 C O
Tos N C C O C O Et
Tos C
+2EtO- + 2CO2
H2 C N Tos
Et O C O C C N Tos
O C
H2
O O
KỲ THI CHỌN HỌC SINH GIỎI QUỐC GIA NĂM 2002:
1. Khi xiclotrime hoá 1,3-butatrien với sự có mặt của chất xúc tác cơ kim, người ta đã điều chế được
(Z, E, E)-1,5,9-xiclododecatrien. Đây là môt phương pháp đơn giản để điều chế hidrocacbon vòng
lớn. Khi dùng chất xúc tác thích hợp là các phức π-alyl của kim loại chuyển tiếp người ta điều chế
được (E, E, E)-1,5,9-xiclododecatrien và (Z, Z, E)-1,5,9-xiclododecatrien. Hãy viết công thức cấu
tạo của 3 hợp chất trên.
2. Sắp xếp sự tăng dần tính bazơ (có giải thích) của các chất trong từng dãy sau:
(a) CH3-CH(NH2)-COOH, CH2=CH-CH2-NH2, CH3-CH2-CH2-NH2, CH≡C-CH2-NH2 .
-NH-CH3 , -CH2-NH2 , C6H5-CH2-NH2, p-O2N-C6H4-NH2

BÀI GIẢI:
1. Công thức cấu tạo của ba chất :

Z, E, E E, E, E Z, Z, E
2. Trật tự tăng dần tính bazơ :
(a) CH3-CH-COOH < CH≡C-CH2-NH2 < CH2=CH-CH2-NH2 < CH3-CH2-CH2-NH2
NH2

Tồn tại ở dạng Độ âm điện CSP > CSP2 > CSP3


ion lưỡng cực

(b) O2N- -NH2 < -CH2-NH2 < -CH2-NH2 < -NH-CH3

(A) (B) (C) (D)

Nhóm p-O2N-C6H4- Nhóm -C6H4-CH2- Nhóm -CH2- - Nhóm C6H11-


hút electron mạnh do hút e yếu đẩy e, làm tăng và -CH3 đẩy e,
có nhóm -NO2 (-I -C) mật độ e trên - Amin bậc II
làm giảm nhiều mật nhóm NH2
độ e trên nhóm NH2
KỲ THI CHỌN HỌC SINH GIỎI QUỐC GIA NĂM 2002:
1. Viết các phương trình phản ứng theo sơ đồ chuyển hoá sau (các chất từ A, ... G2 là các hợp chất hữu
cơ, viết ở dạng công thức cấu tạo):
Fe
Cl2 (1 mol) Mg 1) Etilen oxit H 2SO4 Br2 (1 mol) E1 + E2
C6H5-CH3 A B C D
(1 mol) a.s. ete khan 2) H2O/H+ 15OC (1 mol) a.s. G1 + G2
2. (3,5 điểm). Viết sơ đồ phản ứng điều chế các hợp chất sau đây, ghi rõ các điều kiện phản ứng (nếu
có):
a) Từ etanol và các hoá chất vô cơ cần thiết, điều chế:
(A) Propin (không quá 8 giai đoạn). (B) 1,1-dicloetan (qua 4 giai đoạn).
b) Từ benzen và các chất vô cơ, hữu cơ (chứa không quá 3 nguyên tử cacbon), điều chế:
(C) (D)

BÀI GIẢI:
1. Các phương trình phản ứng: a.s

C6H5-CH3 + Cl2 (1 mol) C6H5-CH2Cl + HCl


ete khan
C6H5-CH2Cl + Mg C6H5-CH2MgCl
1) CH2_ CH2
C6H5-CH2MgCl C6H5-CH2-CH2-CH2-OH
2) H2O/H+
H2 H2 H2SO4; 15OC
C6H5 C C CH2OH + H2O

+ HBr
Fe
+ Br2 Br

+ HBr

Br Br

+ HBr
as
+ Br2
Br + HBr

2. a)
-H2O 1) O3
CH3CH2OH H2C CH2 HCHO
2) Zn
HX

Mg HCHO -H2O Br2


CH3CH2X CH3CH2MgX CH3CH2CH2OH CH3CH=CH2 CH3CHBrCH2Br

1) NaNH2 hay KOH/ancol


H3C C CH
2) H2O

-H2O Br2 1) NaNH2 hay KOH/ancol 2HCl


CH3CH2OH H2C CH2 CH2BrCH2Br HC CH CH3CHCl2
2) H2O
b)
H2/Ni [O] CH2OHCH2OH O
OH OH O
CuO H+
O
(C)

CH3Cl Cl2/as Mg/ete 1) etylenoxit


CH3 CH2Cl CH2MgCl C6H5CH2CH2CH2OH
AlCl3 2) H2O

1) HBr 1) CH3COCH3 H2 H2 H2 CH3 H2SO4


C6H5CH2CH2CH2MgBr C C C C CH3
2) Mg/ete 2) H3O+
OH
(D)

KỲ THI CHỌN HỌC SINH GIỎI QUỐC GIA NĂM 2002:


Hợp chất hữu cơ A có công thức phân tử C7H9N. Cho A phản ứng với
C2H5Br (dư), sau đó với NaOH thu được hợp chất B có công thức phân tử C11H17N. Nếu cũng
cho A phản ứng với C2H5Br nhưng có xúc tác AlCl3 (khan) thì tạo ra hợp chất C có cùng công thức phân
tử với B (C11H17N). Cho A phản ứng với H2SO4 (đặc) ở 180oC tạo hợp chất D có công thức phân tử
C7H9O6S2N, sau khi chế hoá D với NaOH ở 300oC rồi với HCl sẽ cho sản phẩm E (E có phản ứng màu
với FeCl3). Mặt khác, nếu cho A phản ứng với NaNO2 trong HCl ở 5oC, rồi cho phản ứng với β-naphtol
trong dung dịch NaOH thì thu được sản phẩm có màu G.
Xác định công thức cấu tạo của A, B, C, D, E, G và viết các phương trình phản ứng (nếu có) để minh
hoạ.
BÀI GIẢI:
Hợp chất hữu cơ A có công thức phân tử C7H9N, số nguyên tử C lớn hơn 6 và gần bằng số
nguyên tử H. Vậy A có vòng benzen.
A phản ứng với NaNO2 trong HCl ở 5oC, rồi cho phản ứng với β-naphtol trong dung dịch NaOH
thì thu được sản phẩm có màu G, chứng tỏ A có nhóm chức amin bậc I và A còn có nhóm metyl.
A phản ứng với H2SO4 (đặc) ở 180oC tạo hợp chất D có công thức phân tử C7H9O6S2N, đây là
phản ứng sunfo hoá nhân thơm, có 2 nhóm -SO3H nên nhóm metyl sẽ ở vị trí para và ortho so với nhóm
amin.
Sau khi chế hoá D với NaOH ở 300oC rồi trung hoà bằng HCl sẽ cho sản phẩm có nhóm chức
phenol E (E có phản ứng màu với FeCl3).
A phản ứng với C2H5Br nhưng có xúc tác AlCl3 (khan) tạo ra hợp chất C có cùng công thức phân
tử với B (C11H17N), là sản phẩm thế vào nhân benzen, và ở vị trí para so với nhóm -NH2 đã có nhóm -
CH3 nên nhóm -C2H5 sẽ thế vào vị trí ortho.
Các amin bậc I rất dễ tham gia phản ứng thế ở nguyên tử nitơ bằng các dẫn xuất halogen để tạo
ra các amin bậc II hoặc bậc III (sau khi đã xử lý bằng kiềm). A phản ứng với C2H5Br (dư) nên sản phẩm
B có công thức phân tử C11H17N sẽ là N,N-dietylanilin.
Công thức cấu tạo của A, B, C, D, E, G và các phương trình phản ứng:
1) C2H5Br dư H3C N(C2H5)2 (B)
2) NaOH
NH2
H5C2 C2H5
NH2 C2H5Br/ AlCl3 khan (C)

CH3
NH2 NH2
o o
CH3 H2SO4 đặc/180 C HO3S SO3H1) NaOH, 300 C HO OH
(A) 2) H+

CH3 (D) ONa CH3 (E)

NaNO2 + HCl, 5OC H3C N=N


β-naphtol/NaOH (G)
KỲ THI CHỌN HỌC SINH GIỎI QUỐC GIA NĂM 2002:
Thuỷ phân một protein (protit) thu được một số aminoaxit có công thức và
pKa như sau:
Ala CH3CH(NH2)COOH (2,34; 9,69); Pro COOH (1,99; 10,60);
Ser HOCH2CH(NH2)COOH (2,21; 9,15); N
Asp HOOCCH2CH(NH2)COOH (1,88; 3,65;9,60); H
Orn H2N[CH2]3CH(NH2)COOH (2,10; 8,90; 10,50);
Arg H2NC(=NH)NH[CH2]3CH(NH2)COOH (2,17; 9,04; 12,48);
1. Viết tên IUPAC và công thức Fisơ ở pHI của Arg, Asp, Orn. Trên mỗi công thức đó hãy ghi (trong
ngoặc) giá trị pKa bên cạnh nhóm chức thích hợp. Biết nhóm -NHC(=NH)NH2 có tên là guanidino.
2. Ala và Asp có trong thành phần cấu tạo của aspactam (một chất có độ ngọt cao hơn saccarozơ tới 160
lần). Thuỷ phân hoàn toàn aspactam thu được Ala, Asp và CH3OH. Cho aspactam tác dụng với 2,4-
dinitroflobenzen rồi thuỷ phân thì được dẫn xuất 2,4-dinitrophenyl của Asp và một sản phẩm có công
thức C4H9NO2. Viết công thức Fisơ và tên đầy đủ của aspactam, biết rằng nhóm α-COOH của Asp
không còn tự do.
3. Arg, Pro và Ser có trong thành phần cấu tạo của nonapeptit bradikinin. Thuỷ phân bradikinin sinh ra
Pro-Pro-Gly ; Ser-Pro-Phe ; Gly-Phe-Ser ; Pro-Phe-Arg ; Arg-Pro-Pro ; Pro-Gly-Phe ; Phe-Ser-Pro.
a) Dừng kí hiệu 3 chữ cái (Arg, Pro, Gly,...), cho biết trình tự các aminoaxit trong phân tử bradikinin.
b) Viết công thức Fisơ và cho biết nonapeptit này có giá trị pHI trong khoảng nào? (≈ 6; <6; << 6; > 6;
>> 6).
BÀI GIẢI:
1. Aminoaxit sinh ra từ protein đều có cấu hình L
COO− (2,17) COO− (1,88) COO− (2,10)
H2N H H3N H H2N H
(9,04) (9,60) (8,90)
[CH2]3-NH"C"NH2 CH2COOH [CH2]3-NH3
NH2 (3,65) (10,50)
(12,48)
Axit (S)-2-amino-5- Axit (S)-2-amino- Axit (S)-2,5-diamino-
guanidinopentanoic butandioic pentanoic
COOCH3
2. Aspactam H2N-CH− C− NH − CH-COOCH3 O=C NH H
CH2COOH CH3 H2N H CH3
Metyl N-(L-α-aspactyl) L-alaninat CH2COOH
3. Bradikinin
Arg-Pro-Pro-Gly-Phe-Ser-Pro-Phe-Arg
pHI >> 6 và phân tử chứa 2 nhóm guanidino, ngoài ra còn có 3 vòng piroliddin

COOH
CO-NH−H
CO-NH−H [CH2]3NHC(=NH)NH2
CO-N −H CH2C6H5
CO-NH−H
CO-NH-CH2-CO-NH−H CH2OH
CO−N −H CH2C6H5
CO-N −H
NH2−H
[CH2]3NHC(=NH)NH2
KỲ THI CHỌN HỌC SINH GIỎI QUỐC GIA NĂM 2002:
1. Oxi hoá 150 mg amilozơ bởi NaIO4 thu được 0,0045 mmol axit fomic.
(a) Tính số lượng trung bình các gốc glucozơ trong phân tử amilozơ; biết rằng khi CHO
oxi hoá 1 mol amilozơ bằng NaIO4, số gốc glucozơ đầu mạch tạo ra 1 mol axit H OH
fomic, số gốc glucozơ cuối mạch tạo ra 2 mol axit fomic. HO H
(b) Viết sơ đồ các phương trình phản ứng xảy ra. H O H
HO H
2. Viết sơ đồ các phương trình phản ứng chuyển D-glucozơ thành L-gulozơ có công C H 2O H
thức bên. L -g u lo z a
BÀI GIẢI:
1. (a) Số lượng trung bình các gốc glucozơ trong phân tử amilozơ :
(C6H10O5)n ⎯HIO⎯⎯4
→ 3HCOOH
1 0,0045
⇒ n amiloz¬ = n HCOOH = = 0,0015( mmol)
3 3
150
⇒ M amiloz¬ = = 100000
0,0015
100000
⇒ n= ≈ 617
162
(b) Phương trình phản ứng:

CH2OH CH2OH CH2OH CH2OH CH2OH


O O O + (n+4) HIO4 O O CHO
OH OH OH OHC
O O OH - 3 HCOOH OHC O CH HC O CHO
OH
OH OH n-2 OH HCHO O O n-2
(n+4) NaIO3
2. Sơ đồ chuyển hóa :
CHO COOH CO CH2OH
H OH H OH H OH H OH
O
HO H + HNO3 HO H HO H + Na(Hg) HO H
H OH H OH - H2O H H OH
H OH H OH H OH H OH
CH2OH COOH COOH COOH

CH2OH CH2OH
H OH H OH
H + Na(Hg) HO H
- H2O H OH pH = 7 H OH
O H OH
H OH
CO CHO
KỲ THI CHỌN HỌC SINH GIỎI QUỐC GIA NĂM 2003:
1. Khi có mặt enzim aconitaza, axit aconitic bị hidrat hóa tạo thành axit A không quang hoạt và axit B
quang hoạt theo một cân bằng:
H2O HOOC COOH H2O
B C C A
(C6H8O7) H CH2COOH (C6H8O7)
6% Axit aconitic 4% 90%
a) Viết công thức cấu tạo của A và B, ghi tên đầy đủ của chúng và của axit aconitic theo danh pháp
IUPAC. Axit A có pKa: 3,1 ; 4,8 ; 6,4. Ghi các giá trị pKa bên cạnh nhóm chức thích hợp.
b) Viết sơ đồ điều chế A từ axeton và các chất vô cơ cần thiết.
2. Ozon phân một tecpen A (C10H16) thu được B có cấu tạo như sau:
. Hidro hóa A với xúc tác kim loại tạo ra hỗn hợp sản
CH3 C CH2 CH CH CH2 CH O phẩm X gồm các đồng phân có công thức phân tử
C C10H20.
O
H3C CH3
a) Xác định công thức cấu tạo của A.
b) Viết công thức các đồng phân cấu tạo trong hỗn hợp X.
c) Viết công thức lập thể dạng bền của các đồng phân trong hỗn hợp X.

BÀI GIẢI:
1. a)
HOOC CH CH CH2COOH HOOC COOH H2O (3,1) COOH
C C
OH COOH HOOC-CH2 C CH2COOH
H2O H CH2COOH
4,8(6,4)
(B) OH 6,4(4,8)
Axit (Z)-3-cacboxipentendioc
(A)
Axit-3-cacboxi-2-hidroxipentadioic Axit-3-cacboxi-3-hidroxipentadioic
b)
CH3 CH2Cl CH2Cl CH2CN CH2COOH
Cl2 HCN KCN H3O+
C O C O HO C CN HO C CN HO C COOH
CH3COOH
CH3 CH2Cl CH2Cl CH2CN CH2COOH
2.a) b)
+2H2
; ;

(vòng 7 cạnh kém bền hơn vòng 6 cạnh)


c) Đồng phân lập thể dạng bền:
4 3 3
;
1 1 1
(e e > a a) (e e > a a) (e e > a a)
4 3 3
;
1
1 1
(a e > e a) (a e > e a) (a e > e a)
KỲ THI CHỌN HỌC SINH GIỎI QUỐC GIA NĂM 2003:
1. Hợp chất A (C5H11O2N) là một chất lỏng quang hoạt. Khử A bằng H2 có xúc tác Ni sẽ được B
(C5H13N) quang hoạt. Cho B tác dụng với axit HNO2 thu được hỗn hợp gồm ancol C quang hoạt và
ancol tert-amylic (2-metyl-2-butanol).
Xác định công thức cấu tạo của A. Dùng công thức cấu tạo, viết phương trình các phản ứng tạo
thành B, C và ancol tert-amylic từ A.
2. Hợp chất A (C5H9OBr) khi tác dụng với dung dịch iot trong kiềm tạo kết tủa màu vàng.
A tác dụng với dung dịch NaOH tạo ra 2 xeton B và C cùng có công thức phân tử C5H8O. B, C đều
không làm mất màu dung dịch kalipemanganat ở lạnh, chỉ có B tạo kết tủa màu vàng với dung dịch iot
trong kiềm. Cho B tác dụng với CH3MgBr rồi với H2O thì được D (C6H12O). D tác dụng với HBr tạo ra
hai đồng phân cấu tạo E và F có công thức phân tử C6H11Br trong đó chỉ có E làm mất màu dung dịch
kali pemanganat ở lạnh.
Dùng công thức cấu tạo, viết sơ đồ phản ứng từ A tạo thành B, C, D, E, F. Viết tên A và D theo
danh pháp IUPAC.
BÀI GIẢI:
1. * * * *
*

CH3 CH CH2 CH3 CH3 CH CH2 CH3 CH3 CH CH2 CH3 CH3 CH CH2 CH3
CH2NO2 +
H2/ Ni CH2NH2 CH2 H2O CH2OH
HNO2
(A)
* + OH
chuyển vị H2O
CH3 CH CH2 CH3 CH3 C CH2 CH3
+ CH3 C CH2 CH3
CH2 CH3
CH3
2.
H2O Br
CH3 C CH2 CH2 CH2 CH3 C CH CH2 CH2 CH3 C (B )
OH
O Br O Br O

CH2 C CH2 CH2 CH2 Br (C)


O Br
O
CH3 CH3
1) CH3MgBr H2O
CH3 C CH3 C CH3 C (E )
O 2) H2O HBr
OH Br
(D ) CH3
+
CH3 C CH CH2 CH2 (F )

Br
OH
H2O + Br
CH3 C CH3 C CH3 C CH CH2 CH2
+
+H
CH3 CH3 chuyển vị CH3 Br
Tên gọi : A : 5-brom – 2 – pentanol ; D : 2 – xiclopropyl – 2 – propanol

KỲ THI CHỌN HỌC SINH GIỎI QUỐC GIA NĂM 2003:


1. Hãy đề nghị sơ đồ phản ứng với đầy đủ điều kiện để:
a) từ etylen và các chất vô cơ tổng hợp các hợp chất sau và sắp xếp chúng theo thứ tự tăng dần nhiệt độ
sôi:
C2H5OCH2CH2OCH2CH2OH (Etylcacbitol) ;
O O O NH
b) Từ benzen hoặc toluen và các chất vô cơ tổng Dioxan hợp
được các dược chất sau: Axit 4-amino-2- Mopholin
hidroxibenzoic; axit 5-amino – 2,4 – dihidroxibenzoic.
2. a) Dùng công thức cấu tạo, hãy hoàn thành sơ đồ tổng hợp sau đây:
→ C2H3O2Cl ⎯⎯⎯⎯
COCl2 + CH3OH ⎯⎯ 6 5 2
→ B ⎯⎯⎯⎯
2
→ C8H8O4NSCl
C H NH HOSO Cl
⎯⎯⎯
NH 3
→D
+
⎯⎯⎯
H 3O
→ C6H8O2N2S.
b) Giải thích hướng của phản ứng tạo thành C8H8O4NSCl và C6H8O2N2S.
BÀI GIẢI:
+

1. a) CH2=CH2 ⎯⎯⎯⎯ → C2H5OH


H 2 O, H ,
t 0 ,p
CH2 CH2
O2/Ag C2H5OH
CH2 CH2 O
CH2 CH2 CH2CH2OH CH2CH2OCH2CH2OH (A)
t0 H3O+ H3O+
O
C2H5O C2H5O

H3O+ H2SO4
CH2 CH2 HOCH2CH2OCH2CH2OH O O (B)
2
t0
O
NH3 1) H2SO4
CH2 CH2 HOCH2CH2NH-CH2CH2OH O NH ( C)
2 H2O 2) Na2CO3
O
Nhiệt độ sôi: A > C > B.
COOH
NO2 NH2 OH - OH
HNO3 Sn/ HCl HNO2 CO2,OH
0
H2SO4, t0 t ,p

b) NO2 NH2 NH2 NH2


COOH COOH COOH
NH2 OH OH OH Sn /HCl OH
HNO2 CO2,HO HNO3
0
t,p H2SO4, t0
O2N H2N
NH2 OH OH OH OH

hoặc
CH3 CH3 COOH COOH COOH
NO2 NO2 NH2 OH
HNO3 K 2Cr2O7 Sn / HCl HNO2
H2SO4, t0 H+

NO2 NO2 NH2 OH

COOH COOH
HNO3 OH OH
Sn / HCl
H2SO4
O2N H2N
OH OH
2. a) COCl2 + CH3OH ⎯⎯
→ CH3O-COCl

NHCOOCH3 NHCOOCH3 NHCOOCH3 NH2

C6H5NH2 HOSO2Cl +
NH3 H3O
CH3OCOCl
d-

SO2Cl SO2NH2 SO2NH2


(B) (C) (D) (E)
b) –NHCOOCH3 định hướng o, p; do kích thước lớn, tác nhân lớn nên vào vị trí p.
–NHCOOCH3 este-amit thủy phân thành CO2↑ , còn – SO2NH2 bền hơn.
KỲ THI CHỌN HỌC SINH GIỎI QUỐC GIA NĂM 2003:
TRF là tên viết tắt một homon điều khiển hoạt động của tuyến giáp. Thủy phân hoàn toàn 1 mol
TRF thu được 1 mol mỗi chất sau:
N CH2-CH-COOH
NH3 ; ; HOOC-CH2-CH2-CH-COOH ; NH2
COOH
N NH2 N
(Pro) (Glu) (His)
H H
Trong hỗn hợp sản phẩm thủy phân không hoàn toàn TRF có dipeptit His-Pro. Phổ khối lượng cho biết
phân tử khối của TRF là 362 đvC. Phân tử TRF không chứa vòng lớn hơn 5 cạnh.
1. Hãy xác định công thức cấu tạo và viết công thức Fisơ của TRF.
2. Đối với His người ta cho pKa1 = 1,8 ; pKa2 = 6,0 ; pKa3 = 9,2. Hãy viết các cân bằng điện ly và ghi
cho mỗi cân bằng đó một giá trị pKa thích hợp. Cho 3 biểu thức:
pHI = (pKa1+pKa2+pKa3) : 3 ; pHI = (pKa1+pKa2) : 2 ; pHI = (pKa2+pKa3) : 2 ;
biểu thức nào đúng với His, vì sao?
3. Hãy đề nghị sơ đồ phản ứng với đầy đủ điều kiện để tổng hợp axit (D, L) – glutamic từ hidrocacbon
chứa không quá 2 nguyên tử cacbon trong phân tử.

BÀI GIẢI:
1. *Từ dữ kiện thủy phân suy ra 2 công thức Glu-His-Pro và His-Pro-Glu (đều có 1 nhóm
–CO – NH2)
* Từ M = 362 đvC suy ra có tạo ra amit vòng (loại H2O)
* Từ dữ kiện vòng ≤ 5 cạnh suy ra Glu là aminoaxit đầu N và tạo lactam 5 cạnh, còn Pro là
aminoaxit đầu C và tạo nhóm – CO – NH2.
Vậy cấu tạo của TRF:
HN CH CO-NH CH CO N CH CO-NH2
CH2
O
N

NH
Công thức Fisơ:
CO NH2
CO N H
CO NH H
NH H CH2
O N
NH
2. Cân bằng điện ly của His:
COOH COO COO COO
+ + +
H3 N H H3 N H H3 N H H2N H
+ + +
-H -H -H
CH2 CH2 CH2 CH2
(1) (2) (3)
+ +
HN HN N N
NH NH NH NH
(+2) 1,8 (+1) 6,0 (0) (-1)
(hoặc viết 3 cân bằng riêng rẽ; không cần công thức Fisơ)
* pHI = (pKa2 + pKa3) : 2 là đúng,
vì phân tử His trung hòa điện (điện tích = 0) nằm giữa 2 cân bằng (2) và (3)
3. Tổng hợp axit (D,L)-glutamic
HC ≡ CH ⎯⎯⎯
HCN
→ NC – CH = CH2 ⎯⎯⎯
xt, t 0
2
→ NC – CH2– CH2– CH=O ⎯⎯⎯⎯
CO, H HCN, NH3

1) H2O, OH
N C CH2 CH2 CH C N + HOOC CH2 CH2 CH COOH
2) H3O
NH2 NH2
KỲ THI CHỌN HỌC SINH GIỎI QUỐC GIA NĂM 2003:
Disaccarit X (C12H22O11) không tham gia phản ứng tráng bạc, không bị thủy phân bởi enzim
mantaza nhưng bị thủy phân bởi enzim emulsin. Cho X phản ứng với CH3I rồi thủy phân thì chỉ được
2,3,4,6-tetra-O-metyl-D-gulozơ. Biết rằng: D-gulozơ là đồng phân cấu hình ở C3 và C4 của D-glucozơ;
mantaza xúc tác cho sự thủy phân chỉ liên kết α -glicozit, còn emulsin xúc tác cho sự thủy phân chỉ liên
kết β -glicozit.
1. Viết công thức lập thể cña X.
2. Deoxi- D-gulozơ A (C6H12O5) được chuyển hóa theo 2 hướng sau:
HIO4 1) LiAlH4 H3O+
C D Glixerin, 3-hidroxipropanal
+ 2) H2O
CH3OH, H
A B
KOH
C6H11BrO4 (E) C6H10O4 (F)H2O/ DCl hh G
HBr
a) Xác định công thức cấu tạo của A.
b) Viết công thức cấu tạo của B, C, D, E, F.
c) Xác định công thức cấu tạo các chất có trong hỗn hợp G, biết phân tử khối của chúng đều lớn hơn 160
và nhỏ hơn 170 đvC.
BÀI GIẢI:
1. *Từ D-glucozơ suy ra cấu hình của D-gulozơ. X không khử nên có liên kết 1,1-glicozit.
* Sù thủy phân chỉ bởi emulsin chứng tỏ tồn tại liên kết 1 β -1 β ’-glicozit.
CH2OH

HO O OH
O
HO HO
HOCH2
O

OH OH
CH2OH
HO OH
O
HO
O O OH CH2OH
OH
hoặc OH
2. *a) b) Từ hướng chuyển hóa thứ nhất xác định được công thức cấu tạo của A.
CH2OH
O
CH2OH CH2OH HIO4
HOC OH
O O
HO HO
CH3OH HOC
OH + OCH3 (C)
H CH2OH
O
OH OH HBr HO
Br
(A) (B)
(E)
OH
CH2OH CH2OH
O LiAlH 4 O CH2OH CHO
H3O+
OCH3 HOCH2 OCH3 CHOH + CH2
HOC
HOC HOCH2 CH2OH CH2OH
(C) (D)
CH2OH CH2OH
O O
HO HO
KOH H2O/ DCl
Br (G)
(E)
(F)
OH OH

c) H2O + DCl → HOD + HCl


Vì H+ hoặc D+ đều có thể tấn công electrophin, sau đó H2O hoặc HOD tấn công nucleophin nên
thu được cả 4 chất:
CH2OH CH2OH CH2OH CH2OH
O O O O
HO HO HO HO
OH OH OD OD

CH2 CHD CH2 CHD


OH OH OH OH

(G1) 164 (G2) 165 (G3) 165 (G4) 166


KỲ THI CHỌN HỌC SINH GIỎI QUỐC GIA NĂM 2004 (Bảng A):
1. 3-metylbuten-1 tác dụng với axit clohidric tạo ra các sản phẩm, trong đó có A là 2-clo-3-metylbutan
và B là 2-clo-2-metylbutan. Bằng cơ chế phản ứng, hãy giải thích sự tạo thành hai sản phẩm A và B.
2. 2-metylbuten-2 phản ứng với axit clohidric. Trình bày cơ chế của phản ứng, cho biết sarn phẩm chính
và giải thích?
3. Trong phản ứng clo hoá nhờ chất xúc tác FeCl3 , khả năng phản ứng tương đối ở các vị trí khác nhau
trong các phân tử biphenyl vad benzen như sau:
0 250 250 0 1 1
790 790 1 1
0 250 250 0
1 1
a) Trình bày cơ chế phản ứng clo hoá biphenyl theo hướng ưu tiên nhất.
b) Tốc độ monoclo hoá biphenyl và benzen hơn kém nhau bao nhiêu lần?
c) Trong một phản ứng clo hoá biphenyl thu được 10 gam 2-clobiphenyl, sẽ thu được bao nhiêu gam 4-
clobiphenyl?
BÀI GIẢI:
1. CH3
+
CH3 CH3-CH-CH2-CH2 (I)
+
H
CH3-CH-CH=CH2 CH3 CH3
chuyÓn vÞ
CH3-CH-CH-CH3 (II) CH3-C-CH2-CH3 (III)
+ +
- Cl -
Cl CH3 CH3
CH3-CH-CH-CH3 CH3-C-CH2-CH3
Cl Cl
2-Clo-3-metylbutan 2-Clo-2-metylbutan
Do cacbocation bậc hai (II) có khả năng chuyển vị hidrua tạo thành cacbocation bậc ba (III) nên tạo
thành hai sản phẩm A, B.
2.
CH3 CH3
Cl -
CH3-C-CH2-CH3 (I) CH3-C-CH2-CH3
CH3 +
H+ Cl 2-Clo-2-metylbutan
CH3-C=CH-CH3
CH3 CH3
Cl -
CH3-C-CH-CH3 (II) CH3-CH-CH-CH3
+
Cl
2-Clo-3-metylbutan
2-Clo-2-metylbutan là sản phẩm chính.
Do cacbocation bậc ba (I) bền hơn cacbocation bậc hai (II), mặt khác do cacbocation bậc hai (II) có khả
năng chuyển vị hidrua tạo thành cacbocation bậc ba (I) nên sản phẩm 2-clo-2-metylbutan là sản phẩm
chính.
3.
a) Cơ chế SE2 , ưu tiên vào vị trí cacbon số 4.
Cl2 + FeCl3 Cl+FeCl4-
H
b) + Cl +
chËm
+
nhanh
Cl
Cl - H+
Tốc độ monoclo hoá của biphenyl hơn
kbiphenyl (250 × 4) + (790 × 2) 430 benzen 430 lần.
kbenzen 1×6 1
c) Đặt x là số gam 4-clobiphenyl, ta có:

x 790 × 2 790 × 2 × 10 15,8


x
10 250 × 4 1000 (g)

KỲ THI CHỌN HỌC SINH GIỎI QUỐC GIA NĂM 2004 (Bảng A):
1. Từ etilen và propilen có xúc tác axit, platin và điều kiện cần thiết, hãy viết sơ đồ tổng hợp isopren.
2. Cho sơ đồ sau:
O
CH2 C
NBr
CH2 C
1. Li
Xiclohexanol HBr 2. CuI N H /O
O
A B (NBS) C 2 4 2 D KOH
3. Br C2H5OH

Viết công thức các sản phẩm hữu cơ A, B, C và D.


3. Từ axetilen và các hoá chất vô cơ cần thiết, hãy viết các phưong trình phản ứng tajo ra p-
(dimetylamino)azobenzen:
CH3
N N N CH3

BÀI GIẢI:
1.
H+ +
CH2=CH2 CH3-CH2 cacbocation này alkyl hóa propen
+ CH2=CH-CH3 + - H+
CH3-CH2 CH3-CH2-CH2-CH-CH3 CH3-CH2-CH=CH-CH3
+ - H+
CH3-CH2-CH-CH2 CH3-CH2-C=CH2 (II)
CH3 CH3

+ CH2=CH2 +
CH3-CH2-CH2 CH3-CH2-CH2-CH-CH2
H+
CH3-CH=CH2
+ CH2=CH2 +
CH3-CH-CH3 CH3-CH-CH2-CH2
CH3
+ - H+
CH3-CH2-CH2-CH-CH2 CH3-CH2-CH2-CH=CH2
+ -H +
CH3-CH-CH2-CH2 CH3-CH-CH=CH2 (IV)
CH3 CH3
Tách (II) và (IV) ra khỏi hỗn hợp:

CH3-CH2-C=CH2 (II)
CH3 Pt , to
CH2=CH-C=CH2
CH3-CH-CH=CH2 (IV) - H2 CH3
CH3
(Nếu học sinh làm theo cách khác cũng ra sản phẩm thì cho nửa số điểm theo biểu điểm)
2.
Br
Br Br
A B C D
KỲ THI CHỌN HỌC SINH GIỎI QUỐC GIA NĂM 2004 (Bảng A):
Monosaccarit A (đặt là glicozơ A) có tên là (2S,3R , 4S , 5R)–2,3,4,5,6–
–pentahidroxihexanal. Khi đun nóng tới 1000C, A bị tách nước sinh ra sản phẩm B có tên là 1,6–
anhidroglicopiranozơ. D–glucozơ không tham gia phản ứng này. Từ A có thể nhận được các sản phẩm E
(C5H10O5) và G (C5H8O7) theo sơ đồ phản ứng:
Br2 CaCO3 H2O2 HNO3
A C D E G
H2O
1. Viết công thức Fisơ của A và B.
2. A tồn tại ở 4 dạng ghế (D-glicopiranozơ). Viết công thức của các dạng đó và cho biết dạng nào bền
hơn cả?
3. Dùng công thức cấu dạng biểu diễn phản ứng chuyển hoá A thành B. Vì sao
D–glucozơ không tham gia phản ứng tách nước như A?
4. Viết công thức cấu trúc của E và G. Hãy cho biết chúng có tính quang hoạt hay không?
BÀI GIẢI:
3.
NO2 NH2
6000C, XT HNO3 + H2SO4 ® Fe/HCl
3 CH CH ;

(I)
(II) (III)
H2O Ag2O/NH3 NaOH
CH CH
2+ CH3CHO CH3COOH CH3COONaCaO.NaOH CH4
Hg t0
Cl2, a.s
CH4 CH3Cl + HCl
(IV)
Cho (III) ph¶n øng ví i (IV)
CH3
NH2 N CH3

+ 2CH3Cl 2NaOH +
+ 2NaCl 2 H2O

NH2 NaNO2 HCl N 2+

0-50C
N 2+ CH3
N CH3 C6H5 N=N-C6H4 N(CH3)2

Câu III (3,5 điểm): 1. 0,75 điểm ; 2. 1,25 điểm ; 3. 0,75 điểm ; 4. 0,75 điểm
1.
CHO CH
HO HO
OH 0 OH
HO 100 C
HO
OH O + H2O
CH2OH O-CH2
2
HO OH OH OH
OH
O O
OH
HO OH 1 C - β
C1 - β
HO HO
HO OH OH
OH
O O OH
HO OH 1C-α
C1 - α
HO HO HO

1 C - α Bền nhất vì số liên kết e – OH nhiều nhất


3. OH O
OH
O 1000C HO
O
HO OH + H2O
OH
HO HO
D- Glucozơ không phản ứng tách nước vì các nhóm – OH ở C1 và C6 luôn ở xa nhau.
CHO COOH
4.
OH OH
HO HO
OH OH
CH2OH COOH
Quang hoạt Không quang hoạt

KỲ THI CHỌN HỌC SINH GIỎI QUỐC GIA NĂM 2004 (Bảng A):
Câu IV (4 điểm):
1. Từ nhựa thông người ta tách được xabinen và chuyển hoá theo sơ đồ sau:
1) O3 ; 2) Zn/HCl KMnO4 ,H+ H2/ Ni , t0
A B C1 , C2 , C3
(1) (2) (3)
H2N OH (4) P2O5 (5)
D E
A có công thức C9H14O.
a) Viết công thức cấu tạo của các sản phẩm hữu cơ: A , B , C1 , C2 , C3 , D , E .
b) Sản phẩm nào có tạo thành đồng phân và chỉ rõ số lượng đồng phân của mỗi sản phẩm.
2. Pirol là một hợp chất dị vòng với cấu trúc nêu trong hình vẽ. Pirol phản ứng với axit
nitric khi có mặt anhidrit axetic tạo thành sản phẩm X với hiệu H H
suất cao .
a) Viết phương trình phản ứng tạo thành X .
H N H
b) Phản ứng này thuộc loại phản ứng gì ? Giải thích dựa trên cấu
tạo của pirol.
H
c) Giải thích vị trí của pirol bị tấn công khi tiến hành phản ứng này bằng các chất trung gian và độ bền
của chúng.
d) So sánh phản ứng nêu trên với phản ứng nitro hóa của benzen và toluen bằng hỗn hợp HNO3 /H+.
BÀI GIẢI:
1.
O
COOH COOH COOH
COOH COOH
COOH COOH COOH

B C2
A C1 C3

C O
O
HON

O
C
D E
C3 có 2 đồng phân quang học.
C2 có 4 đồng phân quang học.
D có đồng phân E, Z.
2. a)
Anhidrit axetic
HNO3
NO2 + CH3COOH
N H H N

b) Đây là phản ứng thế electrophin vì pirol là một hợp chất dị vòng có tính thơm do có 2 cặp electron
π và cặp electron chưa tham gia liên kết của nitơ.

N H N H N H H N H N

c) Phản ứng thế electrophin của pirol về vị trí ortho đối với nguyên tử nitơ, tức là ở cacbon cạnh
nguyên tử nitơ do cacbocatron trung gian bền hơn nhờ 3 cấu trúc liên hợp

NO2 NO2 NO2


N H H N N
H H
H H
+
Nếu nhóm NO2 tấn công ở cacbon số 3 so với nitơ chỉ có 2 cấu trúc liên hợp kém bền và không ưu
tiên.
H NO2 H NO2

N N
H H
d)
NO2

+ HNO3 + H+

CH3 CH3 CH3

NO2
+
+ HNO3 + H HoÆ
c

NO2

Do hiệu ứng liên hợp của đôi electron không liên kết trên nitơ của pirol nên vòng pirol có mật độ
electron cao hơn so với vòng benzen vì vậy phản ứng thế electrophin của pirol dễ hơn của benzen.
Vòng benzen của toluen có thêm nhóm –CH3 đẩy electron định hướng nhóm NO2 vào vị trí ortho như
pirol hoặc có thể định hướng vào para.
KỲ THI CHỌN HỌC SINH GIỎI QUỐC GIA NĂM 2004 (Bảng A):
1. Các aminoaxit phản ứng với nhau tạo thành NH2
polipeptit. Hãy cho biết cấu trúc của các dipeptit tạo N
thành từ leuxin (CH3)2CHCH2CH(NH2)COOH và CH2 CH COOH
histidin (hình bên). N
H Histidin
2. Gäi A, B là các α-aminoaxit ở môi trường axit, bazơ tương ứng với X là ion lưỡng cực.
a) Xác định tỉ số nồng độ của A và B ở điểm đẳng điện.
b) Vết alanin chuyển về cực nào khi pH < 5 và pH > 8?
c) Xác định hàm lượng tương đối của ion lưỡng cực X của alanin ở điểm đẳng điện, biết rằng hằng số
axit của alanin: pK1 = 2,35 đối với cân bằng A X + H+
pK2 = 9,69 đối với cân bằng X B + H+ .
BÀI GIẢI:
1. Cấu trúc của các dipeptit :
( CH3)2CH - CH2 - CH(NH2) - CO - NH - CH - CH2 N
H - Leu - His - OH COOH HN
N CH2 - CH(NH2) - CO - NH - CH - CH2 - CH(CH3)2 H
NH
H - His - Leu - OH
COOH O N CH2 CH CH3
CH3

N CH2 N O
NH H His-Leu

Leu - Leu ( CH3)2CH - CH2 - CH - CO - NH - CH - CH2 - CH(CH3)2


NH2 COOH
N
His - His CH2 - CH - CO - NH - CH - H2C N
N NH2 COOH HN
H
2.
a) Vết của aminoaxit ở điểm đẳng điện không dịch chuyển về phía catot cũng như anot nên nồng độ
các ion trái dấu phải bằng nhau :
[A] = [B] nên tỉ số bằng đơn vị; [A] (1)
=1
[B]
b) Lập biểu thức tính các hằng số axit

K1 =
[X ][H + ] ; [H+] =
K1[A]
(2) K2 =
[B][H + ] ; [H+] =
K 2 [X ]
(3)
[A] [X ] [X ] [B]
K1K 2 [A][ X ]
[H+]2 = từ (1) , (2) , (3) có [H+] = (K1K2)1/2
[X ][B]
pK1 + pK 2 2,35 + 9,69
pHI = ; Đối với alanin: pHI = = 6,02
2 2

Vì điểm đẳng điện của alanin là 6,02 nên vết di chuyển về phía cực âm khi pH < 5, và
theo hướng cực dương khi pH > 8

c) Từ (2):
[X ] = K1
=
10 −2,35
= 4680
[A ] K2 10 −9, 69

Như vậy nồng độ tương đối của [X] là:


[X ] =
1
= 0,9996 ~1
[A] + [B] + [X ] 2 [A] + 1
[X ]

KỲ THI CHỌN HỌC SINH GIỎI QUỐC GIA NĂM 2004 (Bảng B):
Phân tích nguyên tố một hợp chất hữu cơ A chỉ chứa C, H và O có 70,97% C và 10,12% H.
a) Xác định công thức phân tử của A, biết khối lượng mol của A là 340 gam.
b) Thực nghiệm cho biết: A tác dụng với axit tạo este; hidro hoá A có xúc tác Pd tạo hợp chất X; X cũng
tác dụng với axit tạo este; oxi hoá X bằng KMnO4 tạo thành CO2 và một axit dicacboxylic mạch dài.
Dựa vào các tính chất hoá học trên, hãy đưa ra một cấu tạo phù hợp có thể có của X, A với cấu trúc đối
xứng và mạch cacbon không phân nhánh. Viết các phương trình phản ứng (dùng công thức thu gọn). để
giải thích.
BÀI GIẢI:
a) Lượng C chiếm 70,97%, trong 340 g A có ∼ 241,3 g → Số nguyên tử C ∼ 20
Lượng H chiếm 10,12%, trong 340 g A cã ∼ 34,4 g → Số nguyên tö H ∼ 34
Lượng O trong 340 g A có ∼ 64,3 g → Số nguyên tử O ∼ 4.
Vậy công thức phân tử gần đúng của A là: C20H34O4 (M = 338)
b) A và X tác dụng với axit tạo este. Suy ra A và X trong phân tử có nhóm OH.
Hidro hoá A có xúc tác Pd tạo hợp chÊt X. Suy ra A trong phân tử có liên kết bội.
Oxi hoá X bằng KMnO4 tạo thành CO2 và một axit dicacboxylic mạch dài. Suy ra A và X trong phân tử
có 4 nhóm –OH, hoặc 2 nhóm OH và 2 nhóm CHO, hoặc 2 nhóm OH và 2 nhóm C=O.
Cấu tạo phù hợp có thể có của A, X là:
A: HOCH2-CH(OH)-C16H24-CH(OH)-CH2OH
X: HOCH2-CH(OH)-C16H32-CH(OH)-CH2OH
(hoặc A: HOCH2-CO-C16H24-CO-CH2OH ; OHC-CHOH -C16H24-CHOH-CHO. Có thể viết công thức
cấu tạo của mạch cacbon - C16H24- trong đó có các liên kết bội phân bố đối xứng).
Các phương trình phản ứng:
Thí dụ: A là HOCH2-CH(OH)-C16H24-CH(OH)-CH2OH
H+
A: HOCH2-CH(OH)-C16H24-CH(OH)-CH2OH + 4 CH3COOH
(CH3COO)CH2-CH(OCOCH3)-C16H24-CH(OCOCH3)-CH2(OOCCH3)
X: HOCH2-CH(OH)-C16H32-CH(OH)-CH2OH + 4 CH3COOH
H+

(CH3COO)CH2-CH(OCOCH3)-C16H32-CH(OCOCH3)-CH2(OOCCH3)
A: HOCH2-CH(OH)-C16H24-CH(OH)-CH2OH + 3 H2 Pt

HOCH2-CH(OH)-C16H32-CH(OH)-CH2OH
X: HOCH2-CH(OH)-C16H32-CH(OH)-CH2OH KMnO4

HOOC-C16H32- COOH + 2 CO2


KỲ THI CHỌN HỌC SINH GIỎI QUỐC GIA NĂM 2004 (Bảng B):
1. Hãy gọi tên các sản phẩm tạo thành khi ankyl hoá benzen bằng 1-dodexen với xúc tác axit.
2. Atropin có trong cây Atropa belladonna. Nó kết tinh dưới dạng hình lục lăng, nóng chảy ở 1150C.
Nhờ có tác dụng làm giãn đồng tử nên nó được dùng trong khoa mắt. Atropin là một este có cấu trúc
như hình vẽ.
a) Hãy đánh dấu sao (*) vào nguyên tử cacbon bất đối và cho CH3
N
biết số đồng phân lập thể.
H
b) Khi thủy phân atropin trong môi trường axit nhận được
tropin và axit (R, S)–tropic. Viết phương trình phản ứng. OCOCH C6H5

Atropin CH2OH
BÀI GIẢI:
1. C6H5-CH2-(CH2)10-CH3 (I)
C6H6 + CH2=CH-(CH2)9-CH3
C6H5-∗CH(CH3)-(CH2)9-CH3 (II)

(I): 1-Phenyldodecan (hoặc dodexylbenzen) ; (II): (R,S)-2-phenyldodecan


2.
CH3
N

H Atropi n cã 3 cacbon bÊt ®èi,


cã 2 ®ång ph©n quang häc.
OCOCH C6H5

Atropin CH2OH
Thuû ph©n atropi n trong m«i tr- êng axit
CH3
N
+ HOOC-CH-CH2OH
H
C6H5
OH
Tropin Axit (R,S)-tropic

KỲ THI CHỌN HỌC SINH GIỎI QUỐC GIA NĂM 2004 (Bảng B):
Một disaccarit A không có tính khử. Khi thuỷ phân trong môi trường axit, A cho sản phẩm duy nhất là
pentozơ B. Cũng có thể thủy phân A nhờ enzim α-glicozidaza song không dùng được β-glicozidaza.
Từ B có thể tạo ra D-glucozơ bằng cách cho tác dụng với HCN rồi thuỷ phân (xúc tác axit) và khử.
1) Viết công thức Fisơ và gọi tên B theo danh pháp hệ thống.
2) Viết sơ đồ các phản ứng chuyển hoá B thành D-glucozơ.
3) Viết công thức cấu trúc của A ở dạng vòng 6 cạnh phẳng.
4) Để khẳng định cấu trúc vòng 6 cạnh của A, người ta cho A tác dụng với CH3Br trong môi
trường bazơ rồi thủy phân (xúc tác H+). Dùng công thức cấu trúc, viết sơ đồ các phản ứng.
BÀI GIẢI:
1) B có 3C bất đối giống như 3C bất đối cuối cùng ở D – glucozơ
CHO
CHO
H OH
HO H
Từ HO HSuy ra
H OH
H OH
H OH
H OH
CH2OH B
CH2OH
D-Glucozơ (2S, 3R, 4R)-tetrahidroxipentanal
2)
CN
HO H
HO H
H OH
CHO H OH
HO H CH2OH
HCN
H OH
H OH CN COOH
CH2OH H OH H OH
HO H H2O/H+ HO H
H OH H OH
H OH H OH
CH2OH CH2OH
COOH O C CHO
H OH H OH H OH
-H2O khử
HO H HO H HO H
H OH H OH H OH
H OH H O H OH
CH2OH CH2OH CH2OH
3) Từ các dữ kiện của đề bài, suy ra A do hai phân tử B liên kết α-1,1 glycozit nên:
OH
O O O OH
O OH hay O
OH OH OH
OH O OH OH
OH OH OH 4)
OH OCH3
O O
CH3Br OCH3
O OH OCH3 O
OH baz
OH OH OCH3 O OCH3
O
OH OCH3

CHO
O H3CO H
H3O+
2 OCH3 H OCH3
OCH3 OH H OCH3
OCH3 CH2OH
KỲ THI CHỌN HỌC SINH GIỎI QUỐC GIA NĂM 2005 (Bảng A):
1. Viết sơ đồ điều chế các axit sau đây:
a) Axit: benzoic, phenyletanoic, 3-phenylpropanoic từ benzen và các hóa chất cần thiết khác.
b) Axit:xiclohexyletanoic, 1-metylxiclohexan-cacboxylic từ metylenxiclohexan và các hóa chất cần thiết
khác.
2. Sắp xếp (có giải thích) theo trình tự tăng dần tính axit của các chất trong trong dãy sau:
a) Axit: benzoic, phenyletanoic, 3-phenylpropanoic, xiclohexyletanoic,
1-metylxiclohexan-cacboxylic.
b) COOH COOH CH2COOH

; ; ;
N COOH N
(A) (B) (C) (D)
3. Sắp xếp (có giải thích) theo trình tự tăng dần nhiệt độ nóng chảy của các chất sau:
COOH COOH COOH

; ;
S
N
(A) (B) (C)

BÀI GIẢI:
1. a)
Br2/Fe CH3Br Br2/h KCN
C6H6 C6H5Br C6H5CH3 C6H5CH2Br C6H5CH2CN
Zn
Mg [O] CH2(COOC2H5)2/NaOEt
H3O+
1. CO2
C6H5MgBr C6H5COOH C6H5CH2CH(COOC2H5)2
2. H3O+
1. OH-
2. H3O+ C6H5CH2COOH
3. to
C6H5CH2CH2COOH
b)
CH2 CH3 Br CH3 MgBr CH3 COOH
HBr Mg/ ete 1. CO2
H3O+
CH2Br CH2COOH
HBr/peoxit 1. KCN
c 1. Mg/ ete )
(hoÆ
2. H3O+ 2. CO2
3. H3O+
2. a)
+I 2
H3C COOH CH2COOH CH2COOH COOH
+I 1 -I1CH2CH2COOH -I 2 -I 3
< < < <
+I 1
<
+I 2 -I 1 < -I 2 < -I 3
Các gốc hidrocacbon có hiệu ứng +I lớn thì Ka giảm và -I lớn thì Ka tăng
b)
CH2COOH COOH COOH
-I 1 -I 2 -I 4
< C O <
< -C3 N -I 3
H O N -C4
(D) (C) (A) (B)
Vì: - I1 < - I2 nên (C) có tính axit lớn hơn (D).
(A) và (B) có N nên tính axit lín hơn (D) và (C)
(A) có liên kết hidro nội phân tử làm giảm tính axit so với (B).
3. Tăng dần nhiệt độ nóng chảy của các chất:
COOH COOH COOH

< < MC < MA.
S N (B) có thêm liên kết hidro liên
(C) (A) (B) phân tử với N của phân tử khác.

KỲ THI CHỌN HỌC SINH GIỎI QUỐC GIA NĂM 2005 (Bảng A):
L-Prolin hay axit (S)-pirolidin-2-cacboxylic có pK1 = 1,99 và pK2 = 10,60. Pirolidin (C4H9N) là
amin vòng no năm cạnh.
1. Viết công thức Fisơ và công thức phối cảnh của L-prolin. Tính pHI cña hợp chất này.
2. Tính gần đúng tỉ lệ dạng proton hoá H2A+ và dạng trung hoà HA của prolin ở
pH = 2,50.
3. Tính gần đúng tỉ lệ dạng deproton hoá A− và dạng trung hoà HA của prolin ở
pH = 9,70.
4. Từ metylamin và các hoá chất cần thiết khác (benzen, etyl acrilat, natri etylat và các chất vô cơ), hãy
viết sơ đồ điều chế N-metyl-4-phenylpiperidin.
BÀI GIẢI:
1.
COOH
NH H COOH pHI = 1,99 + 10,60 = 6,30
H 2
N
H
2. Áp dụng phương trình Henderson - Hasselbalch
K1 [HA][H+]
H2A+ HA + H+ K1 =
[ H2A+]
lg
[HA] ; Suy ra:
= pH - pK1 = 2,50 - 1,99 = 0,51
[HA]
= 3,24
[H A ]
2
+ [H2A+]
Vậy ở pH = 2,50 dạng trung hoà chiếm nhiều hơn dạng proton hóa 3,24 lần.
Hay tỉ lệ giữa dạng proton hóa và dạng trung hoà là 1/3,24 = 0,309
3. K2 [ A−]
[HA]
HA H+ + A− lg = pH − pK2 = 9,70 − 10,60 = − 0,90
[ A−] 1
= 0,126 ≅
[HA] 8
Vậy ở pH = 9,7 tỉ lệ giữa dạng deproton hoá và dạng trung hoà là 1/8
4. 2 CH2=CH-COOC2H5 CH2-CH2-COOC2H5
CH3NH2 CH3-N
CH2-CH2-COOC2H5
C2H5ONa
O O
-
1. OH
COOC2H5 2. H3O+, to

N N

CH3 CH3

+Br2/Fe, to Br Mg MgBr
ete

O OH C6H5 C6H5 C6H5


1. C6H5MgBr H2SO4, 170oC H2, Ni, to
N 2. H3O+
N N N
CH3 CH3 CH3 CH3
KỲ THI CHỌN HỌC SINH GIỎI QUỐC GIA NĂM 2005 (Bảng A):
Hợp chất hữu cơ A chứa 79,59 % C; 12,25 % H; còn lại là O chỉ chiếm một nguyên tử trong phân tử.
Ozon phân A thu được HOCH2CH=O ; CH3[CH2]2COCH3 và CH3CH2CO[CH2]2CH=0. Nếu cho A tác
dụng với brom theo tỉ lệ mol 1:1 rồi mới ozon phân sản phẩm chính sinh ra thì chỉ thu được hai sản
phẩm hữu cơ, trong số đó có một xeton. Đun nóng A với dung dịch axit dễ dàng thu được sản phẩm B
có cùng công thức phân tử như A, song khi ozon phân B chỉ cho một sản phẩm hữu cơ duy nhất.
1. Xác định công thức cấu tạo và gọi tên A.
2. Tìm công thức cấu tạo của B và viết cơ chế phản ứng chuyển hóa A thành B.
BÀI GIẢI:
1.
79,59 : 12,25: 8,16= 13 : 24 : 1
12 1 16
A có công thức phân tử C13H24O.
Từ sản phẩm ozon phân tìm ra 2 công thức cấu tạo có thể phù hợp:
CH3CH2CH2C=CH CH2CH2C=CHCH2OH CH3CH2CH2C = CCH2CH2CH=CHCH2OH
CH3 CH2CH3 H3C CH2 CH3
(A1) (A2)
Từ phản ứng brom hóa rồi ozon phân suy ra (A1) phù hợp, vì:
Br2
1:1
CH3CH2CH2C=CH CH2CH2C=CHCH2OH
(A1) CH3 CH2CH3
CH3CH2CH2CBrCHBrCH2CH2C=CHCH2OH ozon phân xeton + O=CHCH2OH
CH3 CH2CH3
Br2
CH3CH2CH2C = CCH2CH2CH=CHCH2OH
(A2) H3C CH2CH3 1:1
ozon phân
CH3CH2CH2CBrCBrCH2CH2CH=CHCH2OH andehit + O=CHCH2OH
H3C CH2CH3
Tên của A: 3-Etyl-7-metyldeca-2,6-dien-1-ol
2. B phải là hợp chất mạch vòng có chứa 1 nối đôi trong vòng. B sinh ra từ A do phản ứng đóng vòng:

H+, to HOH
CH2OH + CH2
- H2O - H+
OH
+
(A) (B)
KỲ THI CHỌN HỌC SINH GIỎI QUỐC GIA NĂM 2005 (Bảng A):
1. D-Galactozơ là đồng phân cấu hình ở vị trí số 4 của D-glucozơ. Trong dung dịch nước D-galactozơ
tồn tại ở 5 dạng cấu trúc khác nhau trong một hệ cân bằng. Hãy dùng công thức cấu hình biểu diễn hệ
cân bằng đó và cho biết dạng nào chiếm tỉ lệ cao nhất.
2. D-Galactozơ là sản phẩm duy nhất sinh ra khi thủy phân hợp chất A (C12H22O11). Để thực hiện phản
ứng này chỉ có thể dùng chất xúc tác là axit hoặc enzim β-galactozidaza.
A không khử được dung dịch Fehling, song tác dụng được với CH3I trong môi trường bazơ cho sản
phẩm rồi đem thủy phân thì chỉ thu được 2,3,4,6-tetra-O-metyl-D-galactozơ.
Hãy tìm cấu trúc của A, viết công thức vòng phẳng và công thức cấu dạng của nó.
3. Đun nóng D-galactozơ tới 165oC sinh ra một hỗp hợp sản O OH
phẩm, trong đó có một lượng nhỏ hợp chất B. Cho B tác O
dụng với CH3I (có bazơ xúc tác) rồi thủy phân sản phẩm
sinh ra thì thu được hợp chất C là một dẫn xuất tri-O-metyl HO
cña D-galactozơ. Hãy giải thích qúa trình hình thành B và OH B
viết công thức Fisơ của C.

BÀI GIẢI:
1. 5 dạng cấu trúc của D-galactozơ:
CH 2OH O OH
HO O OH OH
OH
OH
OH
OH CH=O CH 2OH
β-Galactopiranozơ OH
β-Galactofuranozơ
HO
HO
CH 2OH OH O
HO O CH2OH OH
OH
OH
OH OH
OH
OH
CH 2OH
∝- ∝-
Galactopiranozơ Galactofuranozơ

Chiếm tỉ lệ cao nhất là β-Galactopiranozơ.


2. Các dữ kiện lần lượt cho biết A là disaccarit do 2 đơn vị D-galactozơ liên kết β-1,1 với nhau, cả hai
đều ở dạng vòng piranozơ. Từ đó viết công thức vòng phẳng:
CH 2OH
CH 2OH OH
HO O O HO O OH
OH
O HO HOCH 2 OH
O
hoÆc HO HOCH 2
OH O
OH OH
OH
Công thức cấu dạng:

HO OH OH
OH HO HO OH
O HO O O
hoÆ
c O HO
HO O O
OH HO
OH OH HO
OH
3.
O 5
O O O 4 OH
4 1 4 1 4 1
OH OH OH O
OH 2 2
3 3
OH - H2O
2
1 6
OH 3
5 OH 5 HO 5 HO O
OH
HO HO HO
3
CH2OH 6 2
6 6 OH
Từ công thức cấu trúc trên suy ra rằng 3 nhóm -OH bị metyl hoá là ở các vị trí 2, 3, 5. Do
đó công thức Fisơ của C:
CH=O CH(OH)
OCH3 OCH3
O
CH3O CH3O
HO
OCH3 OCH3
CH2OH CH2OH
2, 3, 5-Tri-O-metyl-D- galactozơ.
KỲ THI CHỌN HỌC SINH GIỎI QUỐC GIA NĂM 2005 (Bảng A):
2-(1-Hidroxipentyl)xiclopentanon (A) là chất trung gian trong quá trình tổng hợp một chất dùng
làm hương liệu là metyl (3-oxo-2-pentylxiclopentyl)axetat (B).
1. Viết công thức cấu tạo của A và sơ đồ các phản ứng tổng hợp A từ axit adipic (hay axit hexandioic)
với các chất không vòng và các chất vô cơ khác.
2. Viết công thức cấu tạo của B và sơ đồ các phản ứng tổng hợp B từ A và các hóa chất cần thiết khác.
3. B có tất cả bao nhiêu đồng phân cấu hình? Hãy viết công thức lập thể của đồng phân có cấu hình toàn
là R.
BÀI GIẢI:
1.
1. Ca(OH)2 CH2[CH2] 3CH=O
o
2. t OH- CH[CH2] 3CH3
HOOC COOH
OH
Axit a®ipic O O (A)
2. B: COOCH 3

H+ H+ CH2(COOC2H5)2
- H2O ChuyÓn vÞ NaOEt
O OH O O
(A)
CH(COOC2H5)2
CH2COOH COOCH3
H3O+/ to CH3OH
- CO2 H+
O O (B)
O
3. B chứa 2 nguyên tố CËnên có 4 đồng phân quang học.
Đồng phân có cấu hình toàn R là:
COOCH3
H
CH3
H
O
KỲ THI CHỌN HỌC SINH GIỎI QUỐC GIA NĂM 2005 (Bảng B):
1. Dùng cơ chế phản ứng để giải thích các kết quả thực nghiệm sau:
a) Hằng số tốc độ dung môi phân 3-metylbut-2-enyl clorua trong etanol lớn hơn dung môi phân anlyl
clorua 6000 lần.
b) Sau khi hoà tan but-3-en-2-ol trong dung dịch axit sunfuric rồi để yên một tuần thì thu được cả but-3-
en-2-ol và but-2-en-1-ol.
c) Xử lý but-2-en-1-ol với hidro bromua thì thu được hỗn hợp 1-brombut-2-en và
3-brombut-1-en.
d) Xử lý but-3-en-2-ol với hidro bromua cũng thu được hỗn hợp 1-brombut-2-en và
3-brombut-1-en.
2. Cho biết sản phẩm nào là sản phẩm chính trong mỗi hỗn hợp sau khi xử lý but-2-en-
-1-ol, but-3-en-2-ol với hidro bromua ở trên? Vì sao?
BÀI GIẢI:
1. a) Dung môi phân (CH3)C=CHCH2Cl trong etanol xảy ra theo cơ chế SN1 và tạo cacbocation trung
gian:
(CH3)C=CHCH2Cl (CH3)2C=CH-CH2+ (CH3)2C+-CH=CH2
- Cl - +
C+ bậc ba bền hơn C+ bậc một CH2-CH=CH2 nên hằng số tốc độ lớn hơn.
b) Có sự chuyển vị anlylic nên tồn tại 2 sản phẩm:
H2SO4 +
CH2=CH-CH-CH3 - H2O CH2=CH-CH-CH3 CH2=CH - CH-CH3
OH + - H2O
OH 2

H2O +
CH2-CH=CH-CH3 CH2-CH=CH-CH3
- H+
OH
c) Có sự chuyển vị anlylic nên tồn tại 2 sản phẩm:
H + +
CH3- CH=CH-CH2OH CH3- CH=CH-CH2 +
- H2O CH3- CH-CH=CH2
Br- Br-

CH3- CH=CH-CH2Br CH3- CH-CH=CH2


d) Tương tự c): Br
CH3- CHOH-CH=CH2 H+ + +
- H2O CH3- CH-CH=CH2 CH3- CH=CH-CH2
Br- Br-

CH3-CH-CH=CH2 CH3-CH=CH-CH2Br
Br
3. Sản phẩm chính là CH3-CH=CH-CH2Br vì: dẫn xuất bậc mét cân đối hơn dẫn xuất bậc hai và dẫn
xuất bậc một là anken có nhiều nhóm thế hơn nên bền hơn anken đầu mạch.
KỲ THI CHỌN HỌC SINH GIỎI QUỐC GIA NĂM 2005 (Bảng B):
1. Viết công thức chiếu Fisơ của dạng mạch hở các chất sau:
OH CH2OH OH OH
HOCH2 O H2C O OH O O
OH HO CH2OH
OH H3C
OH OH OH HO OH OH HO OH
OH
(A) (B) (C) (D)
2. Trong các chất (A), (B), (C), (D) trên, chất nào:
a) thuộc dãy L? b) là đường deoxi? c) là đường có mạch nhánh?
d) thuộc loại xetozơ? e) có dạng furanozơ? g) có cấu hình α ở nhóm anomeric?
BÀI GIẢI:
Công thức chiếu Fisơ của (A), (B), (C), (D):
CH2OH
CHO CHO CHO C=O
HO OH OH OH
HO OH OH
OH H3C OH
OH HO OH
HO
CH2OH OH
CH2OH CH2OH CH2OH
(A) (B) (C) (D)

2. Trong các chất (A), (B), (C), (D) trên:


a) (A), (C) là đường thuộc dãy L
b) (B) là đường deoxi.
c) (C) là đường có mạch nhánh.
d) (D) thuộc loại xetozơ.
e) (B) có dạng furanozơ.
f) (D) có cấu hình α ở nhóm anomeric.
II. OLYMPIC HÓA HỌC QUỐC TẾ:
OLYMPIC HÓA HỌC QUỐC TẾ 1996:
Hai hydrocacbon đồng phân A và B chứa 85,7% cacbon theo khối lượng.
1) Viết công thức tổng quát (không phải công thức thực nghiệm thoả điều kiện này).
Hydrocacbon A và B có những tính chất sau: Phản ứng của mỗi chất với ozon và xử lý tiếp theo
với bột kẽm trong axit tạo sản phẩm hữu cơ duy nhất C. Sự oxy hóa hợp chất C cho ra sản phẩm duy
nhất là axit cacboxylic D. Số liệu phổ cho thấy tất cả các nguyên tử hydro trong hợp chất D, trừ hydro
của nhóm cacboxyl đều thuộc nhóm metyl. Khối lượng riêng của hơi D quy về điều kiện tiêu chuẩn (0oC
và 1atm) là 9,1g/L (thể tích mol bằng 22,4L tại 0oC và 1atm).
Khi phản ứng với kali pemanganat trung tính trong nước, hợp chất A phản ứng dễ hơn hợp chất
B. Trong phản ứng này A tạo thành một hợp chất F duy nhất còn B tạo thành một hỗn hợp đồng phân
G1 và G2 theo tỉ lệ 1:1.
Phải viết và cân bằng mọi phương trình phản ứng. Không đòi hỏi viết cơ chế phản ứng.
2) Viết công thức cấu tạo của hợp chất D khi ở trong dung dịch nước và khi ở pha hơi.
3) Viết công thức cấu tạo của hợp chất C.
4) Viết công thức cấu tạo các đồng phân A và B.
5) a) Viết các phương trình phản ứng của qúa trình chuyển A hoặc B thành C và D.
b) Viết các phương trình phản ứng của qúa trình chuyển hóa A thành F, B thành G1 và G2.
6) Các hợp chất G1 và G2 phản ứng dễ dàng với axeton có mặt axit để tạo các hợp chất H1 và H2.
Viết công thức cấu tạo của H1 và H2.
7) Các hợp chất A và B phản ứng với brom. Một trong các sản phẩm của phản ứng này là không
cực (momen lưỡng cực của phân tử này coi như bằng 0) và không có tính quang hoạt. Viết công
thức hóa học lập thể của sản phẩm này và phương trình tạo thành chất ấy. Hãy xác định cấu hình
tuyệt đối của các nguyên tử có tính đối xứng gương trong phân tử này (nếu có) và đánh dấu
chúng theo quy tắc đọc tên R và S bằng cách chỉ định đúng R và S tại mỗi tâm lập thể.
Anken phản ứng với peaxit (peoxiaxit) dẫn đến sự cộng hợp một nguyên tử oxy vào liên kết đôi
để tạo một vào ba cạnh có chứa oxy. Phản ứng “epoxy hóa” này có tính lập thể đặc thù rõ rệt dẫn đến sự
lưư giữ các vị trí tương đối của các nhóm thế trên liên kết đôi mà nguyên tử oxy gắn vào.
Sự epoxy hóa hợp chất A bằng axit peaxetic tạo thành một hợp chất duy nhất K. Trong cùng điều
kiện B tạo thành một hỗn hợp hai đống phân L1 và L2 (theo tỉ lệ 1:1).
8) Hợp chất K có tính quang hoạt không? Viết công thức hóa học lập thể của K và chỉ rõ hóa học
lập thể của nó. Mỗi hợp chất L1 và L2 có tính quang hoạt không? Viết công thức hóa học lập thể
của L1 và L2 và chỉ rõ hóa học lập thể của nó.
BÀI GIẢI:
1) Công thức tổng quát: CnH2n.
2) Trong dung dịch nước: (CH3)3CCOOH
Ở trạng thái hơi:
O O

O OH
Cấu tạo nhị hợp ở pha hơi được dự đoán nhờ trị số khối lượng riêng.
Khối lượng mol phân tử ở pha hơi bằng 9,1.22,4 = 204 gần gấp đôi khối lượng mol phân tử của
(CH3)3CCOOH. Như các axit béo đơn giản khác, axit này cũng có thể dime hóa ở dạng hơi.
3) Hợp chất C: (CH3)3CCHO
4) Công thức cấu tạo của A và B:
Me3C CMe3 CMe3

Me3C
A B
5) Các phản ứng chuyển hóa A → C → D
Me3C O CMe3
Me3C CMe3
O3 Zn/H+/H2O [O]
2Me3CCHO Me3CCOOH

O O
Các phản ứng chuyển hóa A → F:
Me3C CMe3
Me3C CMe3
KMnO4

HO OH
H H
Các phản ứng chuyển hóa B → G1, G2.
CMe3 Me3C CMe3 Me3C CMe3
KMnO4
+
Me3C HO OH HO OH
hh triet quang
6) Công thức cấu tạo của H1 và H2
CMe3 CMe3

Me3C O Me3C O
O O

H1 H2
7) Sản phẩm brom hóa không phân cực:
CM e3
Br S
Br C M e3
C M e3 Br
hay H hay
M e3C Br Br
M e 3C H
Br R
CM e3

M e3C M e3C C M e3
B r2

C M e3 Br Br
8) Ta có:
Me3C CMe3

O : Phân tử không có tính quang hoạt


Me3C CMe3

O : Phân tử có tính quang hoạt.


Me3C CMe3

O : Phân tử có tính quang hoạt


OLYMPIC HÓA HỌC QUỐC TẾ 1996:
Đôi khi hoá học lập thể của các hợp chất hữu cơ có thể được xác định bằng cách khảo sát tính
chất hóa học đặc trưng của chúng. Cấu tạo của một trong các đồng phân của axit 5-norbonen-2,3-
dicacboxylic, hợp chất X (kí hiệu ~ để chỉ không nêu rõ hóa học lập thể) được hình thành nhờ các thí
nghiệm sau:

COOH

COOH
X
Khi đun nóng X nóng chảy và tách nước để tạo ra hợp chất mới Y. Hợp chất Y tan chậm trong
lượng dư dung dịch nước của NaOH để tạo X1, cùng một sản phẩm như X tạo thành trong lượng dư
dung dịch nước của NaOH. Dung dịch thu được của X1 trong NaOH được tác dụng với iot, tạo thành các
hợp chất có chứa iot. Axit hóa dung dịch tạo một hỗn hợp hai chất đồng phân triệt quang (tiêu triền hay
raxemic) A và B theo tỉ lệ 3:1. Chuẩn độ 0,3913g hợp chất A với NaOH 0,1000M có mặt phenolphtalein
cần dùng 12,70mL bazơ. Cần cùng một thể tích NaOH 0,1000M NaOH để chuẩn độ 0,3913g hợp chất
B. Khi đun nóng, hợp chất A chuyển chậm thành hợp chất mới C không chứa iot và có thể phản ứng với
nước. Trong cùng điều kiện, hợp chất B không xảy ra chuyển hóa này nhưng khi đun nóng với axit
clohydric thì chuyển chậm thành A.
Phải viết và cân bằng mọi phương trình phản ứng. Không cần viết cơ chế phản ứng.
1) Đánh dấu (*) các nguyên tử cacbon phi đối xứng (asymmetric) trong axit 5-norbonen-2,3-
dicacboxylic.
2) Viết công thức hóa học lập thể của từng đồng phân lập thể có thể có của hợp chất X và cấu tạo
của sản phẩm tách nước nếu có xảy ra.
3) Viết các phương trình phản ứng của một đồng phân lập thể bất kỳ của X và hợp chất tương ứng
Y với lượng dư dung dịch NaOH trong nước.
4) Tính khối lượng mol phân tử của chất A. Viết các phương trình phản ứng từ X1 đến A.
5) Viết phương trình phản ứng tạo thành C từ A và phản ứng của C với nước.
6) Viết công thức hóa học lập thể của hợp chất X thoả mãn tất cả các dữ kiện đã cho.
7) Viết các phương trình phản ứng dẫn từ B đến A.
8) A và B có phải là các đồng phân không đối quang (diastereoisomers)?

BÀI GIẢI:
1) Các trung tâm bất đối của X:

* * COOH
*
COOH
*
2) Công thức cấu tạo của các đồng phân lập thể của X được ghi ở bên trái, cột bên phải là cấu tạo
của sản phẩm tách nước tương ứng (nếu sản phẩm không tồn tại có dấu gạch ngang).
COOH
OC
COOH O
OC
O
COOH C
O
COOH C
O

COOH

COOH

COOH
COOH
3) Phản ứng của một đồng phân lập thể của X với NaOH:

COOH COONa
+ 2NaOH + 2H2O
COOH COONa

Phản ứng của một đồng phân lập thể của Y với NaOH:

O
C COONa
O + 2NaOH + H2O
C COONa
O
4) MA = 0,3913/(12,7.0,100/1000) = 308 (g/mol-1)
Các phản ứng từ X đến A:
I I

I2 HCl
-NaI
COONa COONa -NaCl COOH

COONa O CO O CO
5) Chuyển hóa A thành C:
I

COOH
+ HI
O CO

O CO O CO

Phản ứng của C với nước:

O CO + 2H2O OH COOH

O CO OH COOH
6) Cấu tạo của X:

COOH

COOH
7) Chuyển hóa B thành A:
I I I

+H2O -H2O
CO COOH COOH

O COOH OH COOH O CO

8) Không, A và B không phải là các đồng phân không đối quang.

OLYMPIC HÓA HỌC QUỐC TẾ 1997:


Hợp chất X là một đường ba (tri – saccarit) có chủ yếu trong các thức ăn làm từ hạt bông. Hợp
chất X không phản ứng với dung dịch Benedict cũng như không đối quang. Sự thuỷ phân xúc tác axit
tạo ra ba đường D – hexozơ khác nhau A, B và C. Tất cả các hợp chất A và B cũng như hợp chất I (xem
dưới đây) đều cho cùng một osazon khi phản ứng với lượng dư phenylhydrazin trong môi trường axit.
Hợp chất C phản ứng với axit nitric tạo thành một hợp chất D không có tính quang hoạt (không triệt
quang). Để thiết lập quan hệ giữa cấu hình giữa D – glyxerandehit và C, chất đường andehit 4 cacbon
(andotetrozơ) trung gian khi bị oxy hóa bởi axit nitric không tạo thành được một hợp chất meso. Khi A
được xử lý bởi axit nitric tạo thành axit aldaric có tính quang hoạt. Cả A và B đều phản ứng với 5 mol
HIO4; A tạo thành 5 mol axit metanoic (axit fomic) và 1 mol metanal (fomandehit), trong khi đó B tạo
thành 4 mol axit metanoic, 1 mol metanal và 1 mol CO2. Cả A và B có liên quan với 1 andotetrozơ,
andotetrozơ này là một đồng phân không đối quang (diastereoisomer) của chất mà C có tương quan. Sự
metyl hóa của X rồi thủy phân kế tiếp tạo thành 2,3,4-tri-O-metyl-D-hexozơ (E) (chuyển hóa từ A);
1,3,4,6-tetra-O-metyl-D-hexozơ (F) (chuyển hóa từ B) và 2,3,4,6-tetra-O-metyl-D-hexozơ (G) (chuyển
hóa từ C).
1) Xác định công thức chiếu Fischer của A, B, C và D.
2) Vẽ đầy đủ công thức chiếu Haworth tương ứng để chỉ rõ kích thước vòng và hóa học lập thể
tuyệt đối của E, F và G.
3) Viết công thức chiếu Haworth của X.
O

C H
O
HO H
C H
HO H
H OH
H OH
CH2OH
D-glyxerandehit H OH

CH2OH
I
BÀI GIẢI:
Hợp chất X là một tri-saccarit, không phản ứng với dung dịch thuốc thử Benedict, không quang
hoạt. Điều này cho thấy X là một đường không khử và vì vậy chỉ có các liên kết axetal và xetal tồn tại ở
tất cả các cacbon anome. Trong ba monosaccarit thì A và B cho cùng một osazon như vậy có hóa học
lập thể như nhau tại C-3; C-4 và C-5 (và C-6). A và B cũng khác với hợp chất I (là D-mannozơ) tuy cho
cùng một osazon và như vậy một trong số đó phải là C-2 epime của D-mannozơ (là D-glucozơ) và chất
kia phải là đường xeton tương ứng ở C-2 (như D-fructozơ) (Suy luận này được kiểm nhận sau này bằng
các phản ứng cắt mạch oxy hóa). Hợp chất C, sau phản ứng với axit nitric tạo một axit dicacboxylic
không quang hoạt là axit andaric D. Axit andaric như vậy có thể có hai dạng: là AA1 (D) và AA2
Andotetrozơ tạo thành trước C (cũng như trước D) không cho một hợp chất meso sau phản ứng
với axit nitric và như vậy buộc phải là D-threozơ:
CHO COOH

H OH H OH
meso
H OH H OH
CHO

CH2OH COOH
H OH

CHO COOH
CH2OH
D-glyxerandehit HO H HO H
doi xung
guong
H OH H OH

CH2OH COOH
D-threoza

Như vậy axit andaric D tạo thành từ C nêu trên là AA1 và như vậy C phải là D-galactozơ. Hợp
chất A phản ứng với 5 mol axit HIO4 để tạo ra 5 mol axit metanoic (axit fomic) và một mol metanal
(fomandehit) cho phép đề nghị A là một andohexozơ trong khi đó B phản ứng với 5 mol HIO4 tạo được
3 mol axit metanoic, 1 mol metanal và 1 mol CO2 giúp dự đoán nó là một xetohexozơ.
Các hợp chất A và B có liên hệ với một tetrozơ không giống như C (liên quan với D –
erithreozơ). Tetrozơ liên quan đến A và B vì thế phải có cấu tạo sau đây và A là D – glucozơ còn B là D
– fructozơ.
CHO CH2OH CHO

H OH O H OH
CHO

HO H HO H HO H
H OH

H OH H OH HO H
H OH

H OH H OH H OH
CH2OH

CH2OH CH2OH CH2OH


A B C
(D-glucoza) (D-fructoza) (D-galactoza)
Metyl hóa X rồi thuỷ phân kế tiếp tạo thành E, F và G dưới đây:
CHO
CH2OH
H OCH3
O OH
H
H3CO H H
OCH3 H
H OCH3
OCH3 H
H OH
H OCH3
CH2OH
E chuyển hóa từ A
CH2OH

O
CH2OCH3 OH
HO H O
H OCH3
H OH
CH2OCH3
H
H OH
OCH3 H

CH2OH
F chuyển hóa từ B
CHO
CH2OCH3
H OCH3
O OH
H
H3CO H H
OCH3 H
H3CO H
OCH3 H
H OH
H OCH3
CH2OCH3
G chuyển hóa từ C
Trong sự metyl hóa, chỉ các nhóm hydroxyl không tham gia vào sự hình thành axetal/xetal (hoặc
nội phân tử hoặc liên phân tử) mới bị ete hóa. Từ dữ kiện metyl hóa, chỉ E có hai nhóm hydroxyli tự do
có thể liên kết với các cacbohydrat khác. Như vậy A phải là cacbohydrat trung tâm.
Các kết qủa này chỉ ra rằng trật tự của các monosaccarit trong X là C-A-B (hay B-A-C)
Nếu: A5 biểu thị dạng furanozơ (vòng 5 cạnh) của cacohydrat A.
A6 biểu thị dạng pyranozơ (vòng 6 cạnh) của cacbohydrat A.
B5 biểu thị dạng furanozơ (vòng 5 cạnh) của cacbohydrat B.v.v…
thì saccarit X có thể được biểu thị là C6 – A6 – B5
Một trong 4 cấu tạo khác nhau có thể có của X được cho dưới đây:
CH2OH

O H
OH
H
OH H

H O CH2
C
H OH O H
H
H
OH H

O CH2OH
OH
O
Tri-saccarit X A OH H
H OH

CH2OH H

H B OH
Ghi chú: Bản chất của các liên kết anome là không thiết yếu trong đề bài. Sự sắp xếp các liên kết
của A với B và C cũng có thể được đảo lại (liên kết 1,1’ giữa C và A và liên kết 1,6 giữa A và B.
OLYMPIC HÓA HỌC QUỐC TẾ 1997:
Các nhà hóa học của công ty Merck Frosst Cânda ở Montréal đã phát triển một dược phẩm rất có
triển vọng và hữu hiệu để trị bệnh suyễn. Cấu tạo của MK-0476 như sau.

COOH
CH3
H3C OH
S

Cl N

MK-0476
Trong qúa trình kiểm tra, họ phát minh một qúa trình tổng hợp đơn giản và hiệu qủa, mô tả dưới
đây cho sự thiol hóa một phần của MK – 0476 bắt đầu từ este dietyl A
1) Hãy cho biết cấu tạo của các sản phẩm trung gian B – F trong qúa trình tổng hợp này.
COOC2H5
1) LiAlH4 C6H5COCl 1) CH3-SO2Cl/(C2H5)3N
B C (C12H14O3) D(C12H13O2N)
2) H3O+ Pyridin 2) NaCN
A COOC2H5
1) KOH(aq)
2) CH2N2
COOH 1) NaOH(aq)
2) H3O+ 1) CH3-SO2Cl/(C2H5)3N
F(C9H14O3S) E
2) CH3COS-Cs+
SH
G

Một trong những giai đoạn cuối của qúa trình tổng hợp MK – 0476, muối diliti của thiol axit (G)
ở trên được ghép với mạch của phân tử nêu dưới đây:
H3C O
CH3
S H3C OH
O
O

Cl N
H

1)

- +
S Li CO2-Li+

2) H+

COOH
CH3
H3C OH
S

Cl N

MK-0476
2) Căn cứ trên hóa học lập thể quan sát được của phản ứng trên, gọi tên cơ chế của qúa trình ghép
này?
3) Nếu qúa trình phản ứng xảy ra theo cơ chế được đề nghị như trên thì tốc độ toàn phần (chung) sẽ
thay đổi như thế nào nếu nồng độ của cả muối thiolat và chất nền đều cùng lúc tăng gấp ba?
4) Các nghiên cứu mẫu được tiến hành với việc sử dụng brom etan như là một chất nền để tối ưu
hóa qúa trình ghép nói trên. Vẽ cấu tạo sản phẩm chính của phản ứng giữa một mol đương lượng
brom etan với:
a) G thêm hai mol đương lượng bazơ.
b) G thêm một mol đương lượng bazơ.
5) Qúa trình ghép có thể trở nên phức tạp do sự nhị hợp oxi hóa (dime hóa – oxy hóa của G)
Viết công thức cấu tạo Lewis, chỉ rõ tất cả các electron không liên kết của sản phẩm nhị hợp -
oxy hóa trên.
BÀI GIẢI:
1) Công thức cấu tạo của các chất:
O O
CH2OH CH2O CH2O

CH2OH CH2OH H2C C N

B C
D

CH2OH CH2S C CH3

CH2CO2CH3 CH2CO2CH3

E F

2) SN2: thế nucleophin lưỡng phân tử.


3) v = k[tác nhân][tác chất nucleophin]
Tốc độ chung tùy thuộc vào nồng độ của cả tác nhân lẫn tác chất nucleophin. Như vậy, gấp ba
nồng độ của các chất tham gia phản ứng sẽ dẫn đến tốc độ chung của phản ứng tăng gấp 9 lần.
4) Với hai đương lượng bazơ:
CH2SCH2CH3

CH2CO2H
Với một đương lượng bazơ:
CH2SH

CH2CO2CH2CH3
5) Công thức Lewis của sản phẩm dime:
CH2S SCH2

CH2 H2C

HO O O OH

OLYMPIC HÓA HỌC QUỐC TẾ 1998:


Nấm Aspergillus nidulans tạo ra hai lacton (este vòng) thơm A và B (C10H10O4) mỗi đồng phân
tan trong dung dịch NaOH lạnh trong nước nhưng không tan trong dung dịch NaHCO3 trong nước. Cả A
và B đều cho màu tím với dung dịch FeCl3 trong nước. Phản ứng của A với CH3I có mặt K2CO3 tạo
thành C (C11H12O4) mà phổ 1H NMR của nó thấy có chứa ba nhóm metyl không giống nhau, một nhóm
liên kết trực tiếp với vòng thơm. Sự tách loại nhóm metyl có chọn lọc của C với BCl3 rồi xử lý kết\ tiếp
trong nước tạo ra D là một đồng phân mới của A. Phổ 1H NMR của hợp chất D cho thấy rõ sự hiện diện
của một nhóm hydroxyl có tạo liên kết hydro nội phân tử tại δ = 11,8pm
OH

H3C
H3C CO2CH3

O
HO
E I
Hợp chất D được tổng hợp như sau: Phenol E được metyl hóa (MeI/K2CO3) để tạo F(C9H12O2)
sau đó F được khử bằng liti kim loại trong amoniac lỏng và 2 – metylpropan – 2 –ol để cho một dien đối
xứng và không liên hợp G. Có thể chuyển dien này thành liên hợp bằng phản ứng với KNH2 trong
amoniac lỏng rồi xử lý kế tiếp trong nước, qúa trình này chỉ tạo một sản phẩm H. Sự ozon phân H rồi xử
lý không khử lế tiếp tạo ra nhiều sản phẩm, trong đó có xetoeste I. Thực hiện phản ứng Diels – Alder
hợp chất H với dimetyl but – 2 –indioat J tạo thành K (C15H20O6) mà khi đun nóng sẽ loại eten để tạo ra
một este thơm L. Thủy phân L trong môi trường bazơ rồi axit hóa dung dịch tạo thành M(C11H12O6) mà
khi đun nóng trong chân không tạo ra N (C11H10O5). Khử N bằng NaBH4 trong dimetylfomamit tạo
thành C và một lacton O đồng phân, O cũng có thể thu được nhờ metyl hóa B.
1) Viết công thức cấu trúc của tất cả các hợp chất từ A đến O.
2) Hãy trình bày một cấu trúc khác của B.
BÀI GIẢI:
1) Công thức cấu tạo của các chất:
OCH3 OCH3 OCH3
O O

H3C H3C H3C

O O O

HO HO H3CO
A B C
O
OH OCH3 OCH3
OH
O
H3C H3C H3C
H3C

HO H3CO H3CO
H3CO
D E F G

OCH3 OCH3
CO2CH3
H3C H3C CO2CH3 H3C CO2CH3

O
H3CO H3CO CO2CH3
H I CO2CH3
J K
OCH3 OCH3 OCH3
O

H3C CO2CH3 H3C CO2H H3C

H3CO CO2CH3 H3CO CO2H H3CO


N
L M
O

OCH3

H3C

H3CO
O
O
2) Công thức khác của B:
OH

H3C

H3CO

O
OLYMPIC HÓA HỌC QUỐC TẾ 1999:
Glycozit A (C20H27NO11) có trong hạt Rosaceae không phản ứng với dung dịch Benedict cũng
như nước Fehling. Sự thuỷ phân bằng enzym của A cho (-)B (C8H7NO) và C ( C12H22O11) nhưng sự
thuỷ phân hoàn toàn bằng axit tạo thành các sản phẩm hữu cơ (+) D (C6H12O6) và (-) E (C8H8O3).
C có một liên kết β - glycozit và cho phản ứng với dung dịch Benedict cũng như nước Fehling.
Metyl hóa C với MeI/Ag2O tạo thành C20H38O11 mà khi thuỷ phân trong môi trường axit cho 2,3,4-tri-O-
metyl-D-glucopyranozơ.
(±) B có thể được điều chế từ benzandehit và NaHSO3, tiếp theo với NaCN. Thuỷ phân (±) B
trong môi trường axit cho (±) E (C8H8O3).
1) Viết các cấu tạo của A – D với hóa học lập thể phù hợp theo công thức chiếu Haworth trừ chất B.
Glycozit A có độc tính và được cho là do hợp chất F rất độc, được giải phóng trong điều kiện
thủy phân. Sự khử độc của hợp chất F trong cây cối có thể kèm theo các phản ứng (không trình bày hóa
học lập thể).
O O

F + H2N enzym enzym


CH C OH G + H (C4H6N2O2) H2N CH C OH

CH2 CH2

SH C O
L - cystein
NH2
L - asparagin

Một lượng nhỏ hợp chất F trong cơ thể người được giải độc bằng một phản ứng trực tiếp với
cystin cho L – cystein và hợp chất I (C4H6N2O2S) được bài tiết theo nước tiểu (không trình bày hóa học
lập thể).
NH2
H2 NH2
S C C COOH
H enzym H2
F + H2 HS C C COOH + I (C 4 H 6 N 2 O 2 S )
H H
S C C COOH
L - c y s te in
NH2
c y stin
Hợp chất I cho thấy không bị hấp thụ tại 2150 – 2250cm-1 trong phổ IR của nó nhưng quan sát
thấy một dải tại 1640cm-1 và các dải ứng với nhóm cacboxyl.
2) Hãy viết công thức phân tử của các hợp chất F và G và các công thức cấu tạo của các hợp chất H
và I. Chỉ rõ hóa học lập thể của H.
(-) – 1 – Phenyletan – 1 – d C6H5CHDCH3 có thể được điều chế ở dạng quang hoạt và khả năng
triền quang khá mạnh [α]D = -0,6.
CH3

C6H5SO2Cl 1) LiAlD4/ete
C8H10O O D H
(-)N Pyridin 2) H3O+
C6H5
(-) - 1 - phenyletan -1 -d
Cấu hình tuyệt đối của (-) – 1 – phenyletan – 1 – d liên hệ với (-) E theo các phản ứng sau:
Ag2O 1) LiAlH4/ete
C8H8O3 C12H16O3 C10H14O2
(-) E C H
2 5 I (-) J 2) H3O+ (-) K
C6H5SO2Cl 1) LiAlH4/ete C H CHCH (OC H )
L 6 5 3 2 5
Pyridin 2) H3O+ (-) M
Hợp chất (-) M cũng có thể thu được từ hợp chất M như sau:
1)K
C8H10O C6H5CHCH3(OC2H5)
(-) N 2) C2H5I (-) M
3) Suy ra cấu hình tuyệt đối của (-) E và cấu tạo với cấu hình của mỗi chất trung gian ( J – O) trong
qúa trình.
4) Cho biết cơ chế có liên quan trong sự chuyển hợp chất O thành 1 – phenyletan – 1 – d.
BÀI GIẢI:
1) Công thức cấu tạo của các chất:
CH2OH

O O CH2
H
H
OH H
H
H H O O C CN
OH
H
H OH OH H

OH H

H OH
A
H
HO C CN CH2OH

O O CH2
H
H
OH H

H H O OH
OH
B
H
H OH OH H

OH H

H OH
CH2OH
C
O OH
H
H
OH H

OH H

H OH
D
2) Công thức phân tử F: HCN
Công thức phân tử G: H2S
COOH HOOC H HOOC
N N

H2N H NH hay NH2

S S
CH2CN
H I
3) Công thức cấu tạo của các chất từ E đến O:
COOH COOC2H5 CH2OH CH2OSO2C6H5
R
H OH H OC2H5 H OC2H5 H OC2H5

C6H5 C6H5 C6H5 C6H5


(-) E (-) J (-) K L
CH3 CH3 CH3 CH3
R
H OC2H5 H OH H OSO2C6H5 D H
S
C6H5 C6H5 C6H5 C6H5
(-) M (-) N O (-)1 - phenyletan - 1 -d
4) Cơ chế phù hợp: SN2.
OLYMPIC HÓA HỌC QUỐC TẾ 1999:
Peptit A có khối lượng phân tử 1007. Thuỷ phân hoàn toàn bằng axit cho các aminoaxit sau với
số mol bằng nhau: Asp, Cystin, Glu, Gly, Ile, Leu, Pro và Tyr. Oxy hóa A với HCO2OH chỉ cho B chứa
hai gốc axit cysteic (ký hiệu là Cya), là một dẫn xuất của cystein với nhóm thiol bị oxy hóa thành axit
sunfonic.
1) Có bao nhiêu nhóm chứa axit sunfonic được tạo thành từ sự oxy hóa một liên kết disunfua?
Thuỷ phân không hoàn toàn B cho một số di và tri-peptit (B1 – B6). Trật tự của mỗi sản phẩm
thuỷ phân được xác định trong những cách sau.
Aminoaxit có N cuối được xác định bằng cách xử lý peptit với 2,4 – dinitroflobenzen (DNFB) để
cho DNP – peptit. Sau khi thuỷ phân hoàn toàn DNP – peptit bằng axit, thu được một DNP – aminoaxit,
chất này có thể được xác định dễ dàng bằng cách so sánh với các DNP – aminoaxit chuẩn.
2) Khi xử lý B1 với DNFB rồi thuỷ phân kế tiểp bằng axit tạo thành một sản phẩm là DNP – Asp.
Điều này cho thấy B1 có axit aspartic tại N cuối. Hãy viết cấu tạo đầy đủ của DNP – Asp tại
điểm đẳng điện của nó (không cần hóa học lập thể).
Kế đó, aminoaxit có C cuối được xác định bằng cách đun nóng peptit tại 100oC với hydrazin,
chất này bẻ gãy tất cả các liên kết peptit và chuyển tất cả trừ aminoaxit C cuối thành hydrazit của
aminoaxit, còn nhóm cacboxyl ở C cuối còn nguyên vẹn.
Theo cách này, các aminoaxit N- và C- cuối được xác định thứ tự toàn bộ của B1 – B6 như sau:
B1: Asp – Cya B4: Ile – Glu
B2: Cya – Tyr B5: Cya – Pro – Leu
B3: Leu – Gly B6: Tyr – Ile - Glu.
Thuỷ phân B với một enzym từ Bacillus subtilis cho B7 - B9 với thành phần như sau:
B7: Gly – NH2 (glyxinnamit)
B8: Cya, Glu, Ile, Tyr
B9: Asp, Cya, Leu, Pro
3) Viết trình tự của B8 nếu thu được DNP – Cya khi xử lý B8 với DNFB rồi thuỷ phân hoàn toàn
sau đó bằng axit.
4) Nếu các aminoaxit N- và C- cuối của B9 được xác định theo thứ tự là Asp và Leu, viết trình tự
của B9.
5) Viết cấu tạo đầy đủ của A và chỉ rõ vị trí của liên kết disunfua.
Tuy nhiên khối lượng phân tử của A tính được thì lớn hơn gía trị thực nghiệm hai đơn vị. Quan
sát kỹ lưỡng hỗn hợp thu được từ sự thủy phân hoàn toàn bằng axit của A ngoài các aminoaxit tìm được
lúc đầu còn có 3 đương lượng mol amoniac cũng được tạo thành.
6) Đề nghị cấu tạo điều chỉnh của A và khoanh tròn (một hay nhiều vị trí) trên cấu tạo này để cho
thấy tất cả các nguồn tạo amoniac có thể có.
BÀI GIẢI:
1) 2
2) Công thức cấu tạo:
O

O2N
OH

OH
N
H

NO2 O
3) Trình tự của B8 là: Cya – Tyr – Ile – Glu.
4) Trình tự của B9 là: Asp – Cya – Pro – Leu.
5) Cấu tạo đầy đủ của A là: Cys – Tyr – Ile – Glu – Asp – Cys – Pro – Leu – Gly – NH2

6) Cấu tạo điều chỉnh của A:


Cys – Tyr – Ile – Gln – Asn – Cys – Pro – Leu – Gly – NH2

Các vị trí gạch chân là các vị trí tạo amoniac.


OLYMPIC HÓA HỌC QUỐC TẾ 2000:
Shikonin là hợp chất có màu đỏ được tìm thấy ở rễ của cây Lithospermum erythorizon
mọc ở Châu Á. Một đoạn rễ đã được sử dụng trong nhiều thế kỷ để làm bài thuốc dân gian và
ngày nay được sử dụng làm thuốc mỡ để chữa các vết bỏng. Shikonin có công thức cấu tạo như
sau:
OH O

OH O OH
1. Shikonin có bao nhiêu đồng phân lập thể?
2. Có phải các đồng phân lập thể của Shikonin có cùng nhiệt độ nóng chảy không?
Đây là một phần trong qúa trình tổng hợp Shikonin:
O OCH3 OH O

+A
B (xt)

O OCH3 OH O OH

+C
OH O

OH O OH
3. Viết công thức cấu tạo A.
4. Gọi tên A.
5. Viết công thức cấu tạo C
Lượng lớn những hợp chất tương tự Shikonin được tổng hợp để nhận được các chất có
hiệu lực hơn. Một chuỗi phản ứng được dẫn ra dưới đây
SOCl2 KOH/EtOH
Shikonin C6H15ClO4 C16H14O4
70oC
6. Viết công thức cấu tạo của E.
7. E có thể có bao nhiêu đồng phân lập thể.
Một cách khác để điều chế những dẫn xuất có ích của Shikonin là:
OCH3 OCH3

HBr
F
C21H29BrO5

OCH3 OCH3 OCH3 1) Mg/(C2H5O)2


2) CO2
3) HCl(dd)

G
C22H30O7
8. Viết công thức cấu tạo F.
9. Viết công thức cấu tạo G.
BÀI GIẢI:
1. Có 2 đồng phân.
2. Tất cả các đồng phân lập thể của Shikonin đều có cùng nhiệt độ nóng chảy.
3. A có công thức cấu tạo như sau:
O

Cl

4. Tên IUPAC của A: 4-Metyl-3-pentanoylclorua.


5. Công thức của C: NaBH4 (LiAlH4 cũng có thể được chấp nhận).
6. Công thức cấu tạo của E:
OH O

OH O
7. E có thể có 2 đồng phân lập thể .
8. Công thức cấu tạo của F :
OCH3 OCH3

OCH3

Br

OCH3 OCH3
9. Công thức cấu tạo của G:
OCH3 OCH3

OCH3

COOH

OCH3 OCH3
OLYMPIC HÓA HỌC QUỐC TẾ 2000:
Hợp chất thiên nhiên A chứa C, H, O có % mỗi nguyên tố theo khối lượng như sau: C:
63,2%; H: 5,3%; O: 31,5%.
Phổ khối của A:
1. Xác định công thức nguyên của A.

2. Viết công thức phân tử chất A.


Dung dịch chất A trong ete được lắc với dung dịch NaOH trong ete. Sau phản ứng thấy A
không còn trong ete. Lại lắc kỹ dung dịch A trong ete với dung dịch NaHCO3 rồi để yên thấy A
vẫn còn trong ete.
3. A có thể có những nhóm chức nào?
A phản ứng được với thuốc thử Tollens.

4.4. A có nhóm chức nào?


Phổ HNMR của A ghi ở tần số 300MHz cho ở hình sau (dung môi CDCl3). Cho vạch ở
7,27ppm và 3 vạch đơn ở 3,9; 6,3 và 9,8ppm.

Hình sau là sự cụ thể hóa hình phổ trong khu vực từ 6,9 đến 7,6ppm:

Tín hiệu ở 6,3ppm mất đi khi nhỏ D2O vào:


5. Thông tin nào sau đây đúng:
+ Trao đổi liên kết giữa C và H.
+ Trao đổi liên kết giữa O và H.
+ Hiệu ứng pha loãng.
+ Sự thủy phân.
Khi them CDCl3 thì vạch này xuất hiện ở tín hiệu thấp hơn.
6. Thông tin nào sau đây đúng:
+ Tăng mức độ liên kết hydro.
+ Giảm mức độ liên kết hydro.
+ Có liên kết hydro liên phân tử.
+ Có liên kết hydro nội phân tử.
+ Không có liên kết hydro.
7. Viết 4 công thức cấu tạo có thể có của A.
8. Mảnh nào biến mất tại pic có khối lượng 137 và 123.
9. Hai trong số 4 đồng phân đã viết có pKa thấp hơn hai đồng phân còn lại. Xác định chúng.
BÀI GIẢI:
1. C8H8O3.
2. C8H8O3.
3. Nhóm phenol.
4. Nhóm andehit.
5. Trao đổi liên kết O-H.
6. Giảm mức độ liên kết hydro và có sự tạo thành liên kết hydro liên phân tử
7.
CHO
CHO
OCH3

HO OCH3

CHO
CHO
OH

OH

OH OCH3 OCH3
8. CH3 và HC=O
9
CHO

OCH3

CHO
OH

OH OCH3
OLYMPIC HÓA HỌC QUỐC TẾ 2001:
Rễ gừng (Zingiber officinale) được biết nhiều về mặt dược tính và những tính chất của hương
liệu. Tại Ayurveda (Nơi có truyền thống về thuốc ở Ấn Độ), rất nhiều đơng thuốc có sử dụng gừng như
là một vị thuốc trị các bệnh liên quan đến ruột, cảm lạnh thông thường và một số bệnh khác. Một vài
hợp chất được coi như là nguyên nhân gây ra vị cay của gừng. Rất nhiều trong số đó là những dẫn xuất
thế đơn giản của vòng thơm với độ dài mạch có khác nhau. Ba trong số đó là Zingeron; (+)[6] Gingerol
và Shogaol là quan trọng nhất.
Công thức phân tử của các chất này là:
+ Zingeron: C11H14O3.
+ Gingerol: C17H26O4.
+ Shogaol: C17H24O3.
1) Zingeron cho phản ứng dương tính với FeCl3 và 2,4 –DNP (2,4 – dinitrophenylhydrazin). Nó
không cho phản ứng với thuốc tử Tollens. Vậy có thể kết luận nó có những nhóm chức nào?
Brom hóa Zingeron bằng dung dịch nước brom chỉ thu được duy nhất một sản phẩm monobrom.
Phổ IR của zingeron cho biết có sự xuất hiện của liên kết hydro liên phân tử (yếu). Điều tương tự cũng
xuất hiện khi kết thúc phản ứng khử hóa Zingeron theo Clemensen.
2) Từ các thông tin trên cho biết:
(i) Mạch chính của Zingeron
(ii) Các nhóm thế vào vòng thơm.
(iii) Vị trí của tất cả các nhóm thế.
3) Viết công thức cấu tạo của Zingeron.
4) Hoàn thành quy trình tổng hợp sau:
dd NaOH + H2/to Zingerol
A + B C
(C3H6O) xt (C 11H14O3)
Biết A có CTCT:
CHO

HO

OCH3
5) Zingeron có thể dễ dàng chuyển hóa thành Gingerol bằng các phản ứng sau:
Zingeron 1) Me3SiCl/(Me3Si)2NH D 1) hexanal Gingerol
(C11H14O3) 2) LDA; -78oC 2) H3O+ (C17H26O4)
Chú ý:
(1) Me3SiCl/(Me3Si)2NH được sử dụng để chuyển nhóm –OH thành nhóm –SiMe3; nhóm
-SiMe3 dễ bị thuỷ phân trong môi trường axit để cho trở lại nhóm –OH ban đầu.
(2) LDA: liti diisopropylamit là một bazơ mạnh nhưng rất cồng kềnh (về mặt cấu tạo).
(ii) Viết CTCT D.
(iii) Viết CTCT của Gingerol.
(iv) Viết công thức chiếu Fischer đồng phân (R) của Gingerol.
(v) Ngoài Gingerol ta còn có thể nhận được sản phẩm E ( 2 – 3%) là đồng phân của
Gingerol. Viết CTCT E
(vi) Thông tin nào sau đây đúng về sự hình thành E?
a) Sản phẩm của phản ứng điều chế Gingerol là một cặp đồng phân đối quang.
b) Thu được hỗn hợp các đồng phân quang học không đối quang.
c) Thu được hỗn hợp của cặp đồng phân đối quang và đồng phân meso.
(vii) Loại nước của Gingerol bằng cách đun nó với KHSO4 thu được Shogaol. Viết CTCT
của Shogaol.
7) Củ nghệ (Curcuma Longa) là một loại gia vị trong thực phẩm của người Ấn. Nó cũng được sử
dụng nhiều trong các đơn thuốc ở vùng Ayurvedic. Curcumin (C21H20O6) là thành phần chính
của củ nghệ có cấu trúc tương tự như Gingerol. Nó có cân bằng xeto – enol. Curcumin liên quan
đến màu vàng của củ nghệ và cũng góp phần tạo ra vị cay.
Phổ 1H NMR của hình dạng xeto trong Curcumin cho tín hiệu vòng thơm tương tự Gingerol. Nó
cũng cho một vạch đơn ở độ chuyển dịch δ = 3,5 (2H) và hai vạch đôi (mỗi 2H là một vạch ở vùng δ=6-
7 với J = 16Hz. Nó có thể được tổng hợp bằng cách ngưng tụ hai mol A (câu 4) với 1 mol pentan–2,4-
dion.
(i) Viết CTCT của Curcumin.
(ii) Viết CTCT dạng enol của Curcumin.
(iii) Curcumin có màu do:
a) Có vòng thơm.
b) Có nhóm cacbonyl.
c) Có hệ liên hợp.
d) Có nhóm hydroxyl.
BÀI GIẢI:
1) Các nhóm chức có thể có: xeton, hydroxiphenolic
2) Ta có:
(i) Cấu tạo mạch chính của Zingeron: CH2CH2COCH3.
(ii) Nhóm thế vào vòng thơm: OH, CH3.
(iii) Các vị trí có nhóm thế: 1, 2, 4.
3) Cấu tạo của Zingeron:
CH2CH2COCH3

HO

OCH3
4) Các phản ứng:
H
CHO C CHC CH3

dd NaOH O
+ H3C C CH3
HO HO
O
OCH3 OCH3
B C
A

H2; to/xt

CH2CH2COCH3

HO

OCH3
zingeron

5) Ta có:
(i) Công thức cấu tạo D:
O

H3CO CH2CH2C CH2-Li+

(H3C)3SiO
(ii) Công thức cấu tạo của Gingerol:
O
H2 H
H3CO CH2CH2C C C (CH2)4CH3

OH

(H3C)3SiO
(iii) Công thức chiếu Fischer đồng phân R của Gingerol:
CH2COR

HO (CH2)4CH3

H
(iv) Công thức cấu tạo của E:
O
H2 H
H3CO C C CH3

HC (CH2)4CH3

(H3C)3SiO OH
(v) Câu b đúng.
6) (i) Công thức cấu tạo của Curcumin:
O O
H2
H C H
H3CO OCH3
H H

HO OH
(ii) Công thức cấu tạo dạng enol của curcumin:
OH O

H H
H3CO C C C C C C C OCH3
H H H

HO OH
(iii) Câu c đúng
OLYMPIC HÓA HỌC QUỐC TẾ 2001:
1. Dạng gấp nếp của protein:
Protein (polipeptit) có nhiều loại khung có cấu dạng khác nhau. Ở một cấu dạng khung ở trạng
thái bị kéo căng hầu như hoàn toàn (ở dạng song song hay dạng đối song trong dạng gấp nếp β) và một
cấu dạng khác xảy ra sự xoắn hoàn toàn (ở cấu trúc xoắn α).
1) Khoảng cách giữa hai đuôi mạch trong một hexapeptit trong cấu dạng xoắn hoàn toàn là:
a) 10 Å
b) 15 Å
c) 20 Å
d) 25 Å
2) Giả sử rằng hexapeptit đã cho có cấu trúc xoắn α, vẽ một mũi liên kết nối giữa nguyên tử oxy và
hydro của nhóm –NH nếu có liên kết hydro tồn tại giữa hai nguyên tử đó.
O R2 H O R4 H O R6

+
H3N N N O-
N N N

R1 H O R3 H O R5 H O

3) Hai hexapeptit (A và B) biểu diễn hai cấu dạng tương phản trong nước ở tại pH = 7,0, đặc biệt là
nhóm hydro ở serin đã được photphoryl hóa. A có sự xoắn vừa phải và sự xoắn trở nên mạnh
hơn khi serin đã được photphoryl hóa. B có sự xoắn rất yếu và hoàn toàn trở nên rối loạn khi
serin được photphoryl hóa. Vẽ các mũi tên để chỉ sự ảnh hưởng giữa các phần còn lại phù hợp
với sự khác nhau đã nêu.
CH3CONH Lys Ser Leu Phe Glu Arg COOCH3
CH3CONH Lys Ser Leu Phe Arg Glu COOCH3
2. β - lactamaza và tính kháng thuốc.
Penixilin là một dược phẩm có hịêu lực để chống lại sự nhiểm khuẩn. Tuy nhiên thình trjang vi
khuẩn kháng thuốc đã trở nên đáng báo động. Xuất hiện sự kháng penixilin trong vi khuẩn được quy kết
cho một enzym chưa biết gọi là β - lactamaza (hay còn gọi là penixilinaza), nó làm giảm hoạt tính của
vòng penixilin bằng cách mở vòng β - lactam. Cơ chế của phản ứng mở vòng β - lactam liên quan đến
sự tấn công nucleophin bằng nhóm –OH của serin ở hướng tấn công của enzym.
Cơ chế qúa trình này được trình bày như sau:
R R
S S

N N
- -
COO - COO
O O
E nz - Ser - O H E n z -S e r-O H

R
S

N
+ -
HO O COO
H
OH
E nz - Ser

R
R S
S
+ H 2O
+ H 2O
HN
HN -
-
O O COO
HO O COO

+ E nz - Ser - O H E nz - S er
Các nhà khoa học đã cố gắng để xác định được enzym β - lactamaza từ Staphylococcus aureus.
Khi enzym tinh khiết được đánh dấu bằng đồng vị 32P thì ta chỉ đánh dấu một mình Serin. Trong phân
tích thì serin chỉ chiếm 0,35% về khối lượng của enzym β - lactamaza. Biết MSerin = 105 Da (đơn vị khối
lượng nguyên tử).
4) Ước lượng Menzym.
5) Số lượng aminoaxit còn lại hiện diện trong protein của mạch này sẽ là:
a) 100
b) 150
c) 275
d) 375
6) Để đánh dấu đầu hoạt động, β - lactamaza được phân cắt bởi tripsin. Hexapeptit ban đầu (P1)
chứa các aminoaxit sau: Glu, Lys, Met, Phe và Ser
• Sử dụng phương pháp Edman ta biết được aminoaxit “đầu N” là Phe và dãy peptit P2.
• Sử dụng BrCN để cắt mạch P1 thu được dipeptit P4 và tetrapepetit P3
• Sử dụng 1 – flo – 2,4 – dinitrobenzen sau đó thuỷ phân hoàn toàn ta thu được N–2,4–
dinitrophenyl–Glu .
Biết P1; P2; P3 đều chứa Ser. Hãy xác định vị trí các aminoaxit trong P1; P2; P3; P4.
7) Tính M(P3) biết MPhe = 165Da; MGlu = 147Da; MHms = 119Da (Hms: Homoserin); Mnước = 18Da.
8) Chỉ với một lượng nhỏ enzym β - lactamaza thì phản ứng sẽ xảy ra nhanh hơn so với serin tự do.
Khi sử dụng enzym β - lactamaza làm xúc tác thì hằng số tốc độ phản ứng k1 = 350s-1 còn nếu
không sử dụng enzym thì k1 = 0,5s-1.
Từ những thông tin đã cho ở trên, tính nồng độ hiệu lực của tác nhân nucleophin liên kết với
trung tâm hoạt động của enzym. Cho biết trong trường hợp không sử dụng enzym thì nồng độ Ser – OH
tự do trong dung dịch là 1M.
9) Khi cho chất ức chế liên kết với trung tâm hoạt động của β - lactamaza thì có thể ức chế được
enzym. Hằng số phân ly của phức chất ức chế - lactamaza đối với ba chất ức chế khác nhau được
cho như sau:
Chất ức chế Hằng số phân ly (KD)
A 2,0.10-3
B 1,0.10-6
C 5,0.10-9
Trong ba chất A, B, C chất nào có hiệu qủa bảo vệ penixilin khỏi enzym β - lactamaza
Công thức cấu tạo của chất ức chế:
Br
H2C
N

COO-
Để có thể liên kết với trung tâm hoạt động của enzym thì nhóm OH của Serinsex tấn công
nucleophin và kết qủa dẫn đến sự mở vòng và tách Br- ra khỏi chất ức chế. Tác nhân electrophin mạnh
sẽ được sinh ra và sẽ tấn công vào trung tâm hoạt hóa của enzym làm mất hoạt tính của enzym.
10) Viết sơ đồ các phản ứng xảy ra.
BÀI GIẢI:
1) Câu d đúng.
2) Ta có:

O R2 H O R4 H O R6

+
H3N N N O-
N N N

R1 H O R3 H O R5 H O

3) Ta có:

CH3CONH Lys Ser Leu Phe Glu Arg COOCH3

CH3CONH Lys Ser Leu Phe Arg Glu COOCH3

4) 30000Da.
5) Câu c đúng
6) P1: Phe – Glu – Ser – Met – Leu – Lys.
P2: Glu – Ser – Met – Leu – Lys.
P3: Phe – Glu – Ser – Hms/Met.
P4: Leu – Lys.
7) 482Da
8) 700M
9) Câu c đúng
10) Sơ đồ phản ứng:
Br
H2C
N
Enz - Ser - OH +
O

COO-

CH2

N N
O hay O
O O

COO-
Enz - Ser Enz - Ser COO-

X
H2C
HN
O
O

Enz - Ser
COO-
B
OLYMPIC HÓA HỌC QUỐC TẾ 2004:
Phản ứng Diels – Alder là phản ứng đóng vòng [4 + 2] giữa một olefin và một dien để tạo thành
xiclohexen đã được khám phá bởi giáo sư Otto Diels và đồng sự Kurt Alder khi họ trộn benzoquinon với
lượng dư xiclopentadien và nhận được kết qủa sau:
O O

+ A
(C11H10O2)

O O
B

1) Viết CTCT A (không cần chú ý đến mặt lập thể)


Phản ứng Diels – Alder là đồng bộ, một giai đoạn và có tính đặc thù lập thể cao. Ví dụ: chỉ một
đồng phân C được tạo thành từ phản ứng sau:
H
CN
CN

+
CN
CN
CN
C CN H
mà không tạo thành:
CN
H

H
CN
Nếu sử dụng (E) – 1,2 – dixianmetylen thì thu được hai đồng phân lập thể D1 và D2
2) Viết CTCT D1; D2.
Ở phản ứng ban đầu (tạo ra B từ xiclopentadien và benzoquinon). Diels và Alder đã thu được
một trong 6 đồng phân lập thể của B.
O O O
H H H H H H

H H H H H H
O O O
1 2 3
O O O
H H H H H H

H H H H H H
O O O
4 5 6
3) Hãy cho biết đó là đồng phân nào?
Sau khi đun nóng lâu (15h, 120oC) thì từ B (tonc = 157oC), Diels và Alder thu được hai đồng phân
lập thể mới là E(tonc = 153oC) và F (tonc = 163oC). Dưới tác dụng của xúc tác ở 25oC ta thu được đồng
phân lập thể G (tonc = 184oC).
B ⇌ E + F
10% 20% 70%
B ⇌ G
60% 40%
Không cần thiết phải biết E, F, G là chất nào trong số 6 đồng phân lập thể trên
4) Các câu khẳng định sau là đúng hay sai hoặc không đủ dữ kiện để chứng minh:
• Phản ứng Diels – Alder là thuận nghịch.
• B bền nhiệt động hơn E.
• E kém bền nhiệt động hơn F.
• G là đồng phân quang học không đối quang của B.
• G bền nhiệt động hơn F.
5) Cho sơ đồ phản ứng:
OMe

O CO2Me
bazo manh bazo manh OMe
+ I K L
(C12H16O5) (C11H12O4) ∆

MeO CO2Me
−CO2
-MeOH
OMe

CO2Me
Xác định I, K, L. Biết K chỉ có một nhóm metyl và L là sản phẩm cộng Diels – Alder của K và
anken cho trước.
BÀI GIẢI:
1) Công thức cấu tạo của A:
O

O
2) Công thức cấu tạo của D1 và D2:
H H
CN CN

D1: D2:
CN CN
H H
D1 và D2 là hai đồng phân đối quang.
3) Cấu trúc 2 (chính) và cấu trúc 1 (phụ). Theo đề bài thì cấu trúc thoả mãn sẽ là 2
Phản ứng Diels – Alder cho sản phẩm có hóa học lập thể endo. Vấn đề này đã được đề cập đến ở
câu 2, cấu trúc C. Như đã chỉ ra ở cấu trác C thì cấu dạng endo được đặc trưng bởi 2 nguyên tử H và
nhóm CH2 của hệ hai vòng ở cùng phía so với mặt phẳng vòng. Chỉ có cấu trúc 1 và 2 trong số 6 cấu
trúc đã cho là có hóa lập thể endo, endo. Tất cả các cấu trúc khác đều có ít nhất một nhóm thế theo
hướng exo. Trong cấu trúc 1, 3 vòng tạo ra phân tử có hình dạng chữ U bị ảnh hưởng không gian nhiều
hơn so với cấu trúc 2 có dạng phân tử hình zig – zag.
4) Câu đúng: 1, 4.
Câu sai: 2, 3.
Chưa đủ dữ kiện: 5
5) Cấu trúc của I, K, L:

OMe
O O O
O O
CO2Me OMe

CO2Me CO2Me CO2Me

I K L

OLYMPIC HÓA HỌC QUỐC TẾ 2004:


Quy tắc Cahn – Ingold – Prelog (CIP) được sử dụng để xác định hóa lập thể của phân tử.
1) Dựa vào quy tắc CIP hãy xác định độ hơn cấp của các nhóm thế sau:
a) -SCH3 và –P(CH3)2
b) –CCl3 và –CH2Br.
c) Giữa

O ; O

CH3
Pseudoephedrin (1) là chất hay gặp trong các loại thuốc thông thường chống cảm lạnh. Nó có
công thức cấu tạo:
OH

CH3

NHCH3

1
2) Đánh dấu * vào các trung tâm lập thể và cho biết nó là R hay S.
3) Vẽ công thức Newman hay công thức phối cảnh của (1) và vẽ công thức chiếu Fischer của 1.
Đun nóng 1 với KMnO4 trong điều kiện nhẹ nhàng thu được Methcanthinon 2
OH

CH3
MnO4-/H+
2

NHCH3

4) Viết công thức cấu tạo 2 (có xét lập thể) và cân bằng phương trình phản ứng xảy ra. Chỉ rõ sự
thay đổi số oxy hóa của tất cả các nguyên tử có sự thay đổi số oxy hóa.
Đun nóng 2 với LiAlH4 thu được 3. Chất này chỉ khác 1 ở tonc.
5) a) Viết công thức lập thể của 3
b) Phát biểu nào sau đây đúng?
• 1 và 3 là các đồng phân lập thể.
• 1 và 3 là cặp đối quang.
• 1 và 3 là hai đồng phân lập thể không đối quang.
• 1 và 3 là hai đồng phân cấu dạng.
c) Viết công thức của trạng thái chuyển tiếp trong phản ứng 2 → 3
BÀI GIẢI:
1) Nhóm có độ hơn cấp cao hơn sẽ là: -SCH3; -CCl3 và

CH3
2) Ta có:
OH

* CH3
S S
*

NHCH3

3) Công thức chiếu Newman hay công thức phối cảnh của 1:
CH3

CH3
H3CHN H
HO H
hay
H3CHN H
HO H
Ph

Ph
Công thức chiếu Fischer của 1:
Ph CH3

H OH H NHCH3
hay
H3CHN H HO H

CH3 Ph
4) Phương trình phản ứng:
OH O

+0 +2
CH3 CH3 +2
+ 2 MnO4 6H + 2 Mn2+ + 8 H2 O
+7
5 Ph
-
+ +
5 Ph
NHMe NHMe

5) a) Công thức cấu tạo của 3 (đã xác định lập thể):
OH

CH3

NHCH3

b) Câu đúng: 1, 3
Câu sai: 2, 4
c) Cấu trúc của các sản phẩm trung gian:
CH3
CH3
H3C

H N H
H3CHN H
Li + 1) H+
Li
2 2) H2O
HO Ph
O
Ph

H
-
H
OLYMPIC HÓA HỌC QUỐC TẾ 2005:
Phản ứng ngưng tụ giữa axit cacboxylic và amin sinh ra amit. Ví dụ: ngưng tụ axit fomic với
dimetylamin sinh ra N,N-dimetylfomamit, nó có các cấu trúc cộng hưởng sau:
O O

C CH3 C CH3

H N H N

CH3 CH3
1. Xếp các chất N,N-dimetylfomamit (A). N-metylaxetamit (B) và propanamit (C) theo thứ tự tăng
dần nhiệt độ sôi.
2. Nhóm cacbonyl thường được nhận diện bằng dải hấp thụ mạnh trong phổ hồng ngoại (IR). Vị trí
vân hấp thụ phụ thuộc vào độ bền liên kết C = O. Đối với amit thì độ bền của liên kết C = O có thể
được xác định dựa vào hình vẽ trên. Ví dụ: nhóm C = O của xiclohexanon cho vân hấp thụ ở
1715cm-1. Để so sánh với xiclohexanon thì các giá trị nào sau đây là phù hợp với nhóm C = O của
propanamit?
a) 1660cm-1 do độ dài liên kết ngắn của nhóm cacbonyl.
b) 1660cm-1 do độ dài liên kết dài của nhóm cacbonyl.
c) 1740cm-1 do độ dài liên kết ngắn của nhóm cacbonyl.
d) 1740cm-1 do độ dài liên kết dài của nhóm cacbonyl.
3. Glyxin (H2N – CH2 – COOH) là α - aminoaxit. Ba phân tử glyxin có thể tạo ra tripeptit Gly – Gly
– Gly thông qua phản ứng ngưng tụ tạo thành amit và kèm theo sự tách loại hai phân tử nước. Hãy
viết công thức cấu tạo của tripeptit.
4. Khi α - aminoaxit chứa nhoam thế thìu lúc này sẽ xuất hiện hiện tượng đồng phân quang học. Ví
dụ: L – alanin và D – alanin là hai enantiome. Như vậy đối với 3 peptit glyxin, L – alanin và D –
alanin ta có thể thu được bao nhiêu tripeptit?
O O O

H2N H2N H2N


OH OH OH

H H H CH3 H3C H
Glyxin (Gly) L - Alanin (L - Ala) D - Alanin (D - Ala)
5. Tổng cộng có bao nhiêu đồng phân quang học từ các tripeptit trên?
Hiện nay, “polyacrylamide gel with electrophoresis” (PAGE) được sử dụng rộng rãi trong việc
phân tích protein và axit nucleic. Tuy nhiên một trong số những ứng dụng của keo polyamit là phân lập
các hợp chất phenol bằng sắc ký bản mỏng. Các phenol có chứa các nhóm thế khác nhau thì có tính axit
khác nhau. Tính axit khác nhau thì liên kết với keo PAGE càng mạnh.
6. Sắp xếp các chất sau: phenol (D), 4 – metylphenol (E) và 4–nitrophenol (F) theo thứ tự giảm dần
khả năng liên kết với PAGE.
Khả năng hấp thụ một chất trong phổ tử ngoại - khả kiến (UV – Vis) phụ thuộc vào số liên kết
đôi liên hợp trong phân tử đó. Một hợp chất có từ 5 nối đôi liên hợp trở lên thì có xu hướng hấp thụ ánh
sáng trong vùng khả kiến nên kết qủa là chúng có màu. Ví dụ phenolphtalein là một chất chỉ thị axit –
bazơ thông dụng. Trong dung dịch có tính axit và trung tính nó không có màu còn trong dung dịch bazơ
nó có màu đỏ tím (pH: 8,3 – 10,0)
HO

OH

H2SO4 dac; 180oC, 5h -


OH
G +2 H
H+

O
OH

O
phenolphtalein
7. Viết công thức cấu tạo của H.
8 Phenolphtalein có thể được điều chế bằng cách cho chất G phản ứng với hai mol phenol. G phải là
chất nào trong số các chất dưới đây để phản ứng đạt hiệu suất cao nhất.
O O O
(a) (b) (c)
H H H

H OH H

O O
O (d) (e) O

O O

O
BÀI GIẢI:
1. Thứ tự sắp xếp các chất theo nhiệt độ sôi: C > B > A.
Giải thích: Từ cấu trúc cộng hưởng của amit cho thấy nó có một phần điện tích âm trên nguyên
tử oxy và một phần điện tích dương trên nguyên tử nitơ. Amin bậc 1 và bậc 2 có liên kết hydro
mạnh hơn amin bậc 3 (Propanamit: 79oC; N-metylaxetamit: 28oC và N,N-dimetylfomamit: -
61oC).
2. Câu b
3.
H O H O

N N hay
H N OH

H O
Gly - Gly - Gly
O H O

H3N N
N O

4. 27 H O
5 Trong số đó thì 26 tripeptit có đồng phân quang học
Aminoaxit không có tính quang hoạt: H2N – GGG – OH
Aminoaxit có tính quang hoạt: H2N–GGLA–OH; H2N–GGDA–OH …
6 Khả năng liên kết với PAGE giảm dần theo thứ tự F > D > E.
7.
HO O

O O

O O

O O
H H
8 Chất e
OLYMPIC HÓA HỌC QUỐC TẾ 2005:
Các cacbohydrat thiên nhiên đều được tạo thành từ phản ứng quang hợp trong thực vật. Tuy
nhiên một số cacbohydrat không gặp trong thiên nhiên có thể được tổng hợp nhân tạo trong phòng thí
nghiệm. Sau đây sẽ trình bày sơ đồ điều chế L – ribozơ (hợp chất I):
B
CO2Me O
HO
CO2Me
100oC OsO4
O + A HO
ong han kin

CO2Me

CO2Me
Me2C(OMe)2
H+, CH3COCH3

O
O
O
CO2Me O
CO2Me
O enzym pig liver
O

D (spc) CO2H
C CO2Me

+ O3; MeOH O
O CO2Me

O MeO2C
CO2H

O
O
O
E (spp) CO2Me

MCPBA
CO2Me

O O CO2Me
CH2OH
H3O+ 1) MeOH/H+ O
HO H
2) LiAlH4
sau do H2O

OH
OH O
O
I

G
1. Hợp chất A có công thức phân tử C10H10O5. Viết công thức cấu tạo A.
2. Trong các mệnh đề liên quan đến việc chuyển hoá từ A thành C sau đây thì mệnh đề nào đúng,
mệnh đề nào sai?
a) OsO4 là tác nhân oxy hóa trong phản ứng chuyển A thành B.
b) MeOH là sản phẩm phụ trong phản ứng chuyển hóa B thành C.
c) Proton đóng vai trò xúc tác trong phản ứng chuyển hóa B thành C.
d) C có thể được tạo thành với hiệu suất thấp khi không có Me2C(OMe)2.
Enzym pig liver esteraza có thể thủy phân este thành axit cacboxylic. Thuỷ phân C bằng enzym
pig liver esteraza sinh ra hỗn hợp D và E trong đó E là sản phẩm chính. Góc quay cực của hỗn hợp là:
[α]D20 = -37,1o còn của E tinh khiết là [α]D20 = -49,0o.
3. Tính tỉ lệ D/E (theo số mol) trong hỗn hợp phản ứng.
4 Phản ứng của F với axit m – clopebenzoic (MCPBA) sinh ra từ sản phẩm G. Chỉ ra rằng các mệnh
đề sau đây là đúng hay sai:
a) Bản chất của phản ứng là sự oxy hóa F.
b) Nguyên tử oxy thêm vào có nguồn gốc từ MCPBA.
c) Tỉ lệ của hai hợp chất C1 – (R) và C1 – (S) trước và sau phản ứng không thay đổi.
Công thức phân tử của H là C9H16O5. Các gía trị phổ NMR của H cho dưới đây: 1HNMR
(CDCl3) δ 1,24 (s, 3H); 3,24 (m, 1H); 3,35 (s, 3H); 3,58 (m, 2H); 4,33 (m, 1H); 4,50 (d, J = 6Hz, 1H);
4,89 (s, 1H).
a) Viết công thức cấu tạo của H.
b) Xác định cấu hình tuyệt đối của C1; C2; C3: C4 của hợp chất I.
c) Trong công thức chiếu Fischer của I (L – ribozơ) thì các chữ cái P, Q, R, S, T và U đại diện cho
những nhóm chức nào?
CHO

P Q

R S

T U

CH2OH
Disaccarit là hợp chất được tạo thành từ hai đơn vị monosaccarit bởi liên kết glycozit.
Polisaccarit chứa từ 10 đến vài ngàn đơn vị monosaccarit. Ví dụ về disaccarit cho dưới đây:
H OH

H O
HO
H H
H OH
H OH
OH O
H
lien ket glycozit O

H OH
H OH

OH H
d) Có bao nhiêu đồng phân dia tạo thành từ pentasaccarit J nếu nó được tạo thành từ 5 đơn
vị D – glucozơ:

H OH

H O
H O
H O H
H OH

OH H
5
BÀI GIẢI:
1.
O

CO2Me

CO2Me
2. Tất cả đều đúng
3. 12,1 : 87,9 hay 12,2 : 87,8
4. a, b đúng; câu c sai
5.
MeO CH2OH

O O

6. C1, 2, 4: S C3: R
7.
CHO

HO H

HO H

HO H

CH2OH
5
8. 2
OLYMPIC HÓA HỌC QUỐC TẾ 2005:
Ete crown có thể tạo được liên kết với các ion kim loại kiềm. Ví dụ các hằng số liên kết của hai
azacrown (ete vòng chứa nitơ) với Na+, K+, Cs+ được cho ở bảng dưới:
Hằng số liên kết (lg10K)
Ion km loại Bán kính (pm) Hợp chất A Hợp chất B
+
Na 98 2,49 3,57
K+ 133 1,83 5,00
Cs+ 165 1,37 3,39
Antraxen cho sự phát quang mạnh với bước sóng phát xạ trung tâm là 325nm. Kết hợp với sự
liên kết chọn lọc của các azacrown đối với ion kim loại kiềm và sự phát quang mạnh của antraxen, một
ion kim loại phát quang chọn lọc E đã được phát triển.

O O
O O

O
O
O O

N
N

H2C
CH2

A B
1. Cho biết các chất C và D trong qúa trình tổng hợp sau:
H
O

1) NaBH4 O
2) PCl5/benzen
C O O

SO2Cl
1) O O
OH HO

pyridin N
- + D
2) t - BuO K
CH2
O O HO(CH2)2NH(CH2)2OH

Để so sánh, thì dẫn xuất thế antraxen của F và G cũng đã được tổng hợp. Các hợp chất E, F, G
đều không có tính phát quang ở các điều kiện trung tính do hiệu ứng chắn sự di chuyển của electron
(photoinduced electron transfer – PET) do cặp electron không phân chia của nguyên tử nitơ nằm ở phía
hoạt động của antraxen:
O

O O
OH OH

N
N O
H2C H2C

F G
2. Khi thêm dung dịch HCl hợp chất nào sẽ phát quang?
a) Không có.
b) E và F.
c) Chỉ G
d) Tất cả.
3. Khi thêm một lượng tương đương kali axetat vào dung dịch loãng của E, F và G trong metanol thì
hợp chất nào sẽ phát quang mạnh nhất?
a) E
b) F
c) G
4. Khi thêm một lượng tương đương axetat kim loại vào dung dịch loãng của F thì axetat kim loại
nào sẽ phát quang mạnh nhất?
a) CH3COONa
b) CH3COOK
c) CH3COOCs
d) Không có.
Trong qúa trình chiếu xạ với tia cực tím trans – stinben chuyển hoá thành một chất trung gian H,
H chịu sự vòng hoá quang hoá để sinh ra dihydrophenantren I. Oxy hoá tiếp I cho ta phenantren.

H
hv hv
H
nhiet nhiet H

oxy hoa

3.5. Viết công thức cấu tạo H


3.6. Hoá lập thể của hai nguyên tử H trong hợp chất I là gì (cis hay trans)?
Dẫn xuất của Dihydroazulen J có tính chất quang hoá học rất thú vị. Khi chiếu xạ thì thì chất J
không màu sẽ chuyển vị thành vinylheptafulven K. Đun nóng K thì ta lại thu được J:
CN
CN 1

CH3
2
CH3
CN 3

4
CN 5
hv 10
nhiet 6

9
7
8
J K
7. Hợp chất nào hấp thụ ánh sáng ở bước sóng dài hơn?
a) J
b) K
8. Hợp chất K có thể phản ứng với một lượng tương đương CF3CO2H để tạo thành hệ thơm bền vững.
Nguyên tử nào cacbon nào của K dễ bị proton hóa nhất?
a) C – 2
b) C – 3
c) C – 4
d) C – 5
BÀI GIẢI:
1.

N
Cl H
H2C
O O

O O

2. d
3. e
4. a
5. Công thức cấu tạo của H:

6. trans
7. K
8. C3
III. BÀI TẬP CHUẨN BỊ CHO OLYMPIC HÓA HỌC QUỐC TẾ:
OLYMPIC HÓA HỌC QUỐC TẾ 1998:
“Chất ong chúa” Q chứa 65,2% cacbon và 6,75% hydro và không có nguyên tố nào thêm nữa
trừ oxy. Q được biết có tính chất axit và chuẩn độ 43,7mg chất này cần 23,7mL dung dịch nước của
natri hydroxit 0,0100M để đạt đến điểm tương đương. Khối lượng phân tử của Q nhỏ hơn 200.
a) Công thức phân tử của Q là gì và những nhóm chức nào gây nên tính axit của hợp chất?
Q phản ứng với hydro có mặt bột mịn platin tạo thành hợp chất mới A. Khử A với natri bohydrua
trong etanol cho chất B. Hợp chất B dễ dàng loại nước khi đun nóng với axit sunfuric để tạo anken C.
Phổ NMR 13C của C cho thấy sự có mặt của một nhóm metyl gắn vào liên kết đôi.
b) Những nhóm chức nào phù hợp với những phản ứng trên?
Phản ứng ozon phân chất C rồi oxy hóa kế tiếp chỉ cho hai phần là axit etanoic và một axit
dicacboxylic mạch thẳng D. Sự phân chia tương tự của Q tạo thành axit oxalic (axit etandioic) và chất E
có chứa một nhóm chức axit cacboxylic.
c) Suy ra cấu tạo của D và E, từ đó hãy xác định công thức cấu tạo có thể có của Q.
BÀI GIẢI:
Công thức thực nghiệm: C10H16O3 (M = 184,13). Do MQ xấp xỉ 200 nên công thức phân tử của Q
trùng với công thức thực nghiệm (C10H16O3).
Q có tính axit nên có thể chứa một nhóm –CO2H (do chỉ có 3 nguyên tử oxy trong phân tử). Vậy
Q là axit đơn chức, phản ứng với NaOH theo tỉ lệ mol 1:1
R – CO2H + NaOH = R – CO2-Na+ + H2O
43,7mg ≡ 43,7.10-3g Q phản ứng với 23,7mL dung dịch NaOH 0,0100M
Vậy nNaOH = 23,7.10-3.0,0100 (mol)
Nếu 43,7.10-3g Q phản ứng với 23,7.10-3.0,0100mol NaOH thì 43,7.10-3.(103/23,7.0,01)g Q phản
ứng với 1 mol NaOH, nghĩa là 184,3g Q phản ứng với 1 mol NaOH. Điều này chứng tỏ rằng 1 mol Q
phản ứng với 1 mol NaOH và như thế cho phép ta kết luận Q là axit đơn chức dạng R – CO2H.
Kế tiếp ta cần xác định nguyên tử oxy còn lại trong C10H16O3. (hay C9H15O – CO2H)
Có thể là: ete: R – O – R.
rượu: R – O – H.
xeton: R – CO – R’.
andehit: R – CHO.
Số liên kết đôi trong Q = 3. Rõ ràng chưa xác định được hai trong số ba liên kết đôi đó nhưng đã
xác định được 1 (của nhóm –CO2H).
Mặt khác, theo các thí nghiệm:
+
/∆
Q ⎯H⎯ ⎯→ A ⎯NaBH
2 / Pt
⎯⎯ ⎯⎯→ B ⎯H⎯⎯
4 / EtOH
→ C + H 2O
C là một anken nên B là rượu vì B loại nước cho ra anken C khi đun nóng với H+
Ngoài ra, C còn có một nhóm metyl gắn vào nối đôi cho nên ta dự đoán:
H H2
H3C C C R H3C C C R

OH H H
Nếu điều này đúng thì A phải là một xeton
H2 H H2
H3C C C R NaBH4 H3C C C R H3C C C R

O OH H H
A B C
Cần biết rằng nhóm andehit và xeton bị khử bằng NaBH4 để cho rượu bậc 1 và rượu bậc hai.
Nếu A chứa nhóm xeton và nhóm chức axit cacboxylic thì A có công thức cấu tạo:
H2
H3C C C (CH2)7 CO2H

O
và giải thích được 2 trong số các liên kết đôi tương đương. Một trong nhóm chức axit và một
trong nhóm xeton.
Có nghĩa là trong Q còn một liên kết đôi nữa và do Q phản ứng cộng với hydro nên đặc điểm cấu
tạo còn lại phải là một liên kết đôi. Vấn đề là vị trí của liên kết đôi trong Q.
H2 H H2
H3C C C (CH2)7 CO2H H3C C C (CH2)7 CO2H
CH3CH=CH(CH2)6CO2H
O OH
A B C
và giả thiết được xác nhận do C bị phá vỡ bởi ozon và một chất oxy hóa tạo thành axit axetic và một axit
dicacboxylic mạch không nhánh.
H3C C O O C(CH2)6CO2H
CH3CH=CH(CH2)6CO2H +
OH OH
Điều này cho phép khẳng định C duy nhất và từ đó khẳng định A
Q này chắc chắn chỉ đơn giản là một phân tử có chứa nhóm chức xeton (tương tự A) và axit
cacboxylic (tương tự A) và một liên kết đôi.
Nhưng Q khi ozon phân và oxy hóa tạo thành HOOC – COOH cùng với E.
Không quan tâm đến E vì một mảnh nhỏ chỉ có thể xuất phát từ phân tử có dạng tổng quát:
R C O O C C O
R - CH=CH - CO2H +
OH OH OH
Như vậy xác định được Q duy nhất là:
H3C C (CH2)5C C CO2H
H H

O
Vấn đề cuối cùng còn lại là sự đồng phân hóa cis – trans của liên kết đôi:
H3C H3C

C C
O O
(H2C)5 CO2H (H2C)5 H

H H H CO2H
cis trans
Vấn đề này không thể kết luận được với các giả thiết đã cho.
OLYMPIC HÓA HỌC QUỐC TẾ 1998:
Axit crisophanic có trong sắc tố antraquinon thiên nhiên cô lập được từ rễ cây đại hoàng với cấu
tạo dưới đây. Một phương pháp tổng hợp phân tử này do Khoa Nghiên Cứu Hóa Học thuộc Đại Học
Quốc Gia Australia đề nghị:
OH O OH

CH3

O
axit crisophanic
a) 3-metylanisol (3-metyl-metoxybenzen) được khử bằng kim loại liti trong hỗn hợp amoniac hóa lỏng
khan nước, tetrahydrofuran và t-butanol để tạo B (C8H12O). Xử lý B với kali amidua trong amoniac
lỏng khan nước rồi xử lý tiếp trong dung dịch nước dẫn đến sự đồng phân hóa B thành C. Hãy viết
ba công thức cấu tạo có thể có của C.
b) Phổ NMR 1H của C cho thấy có hai proton của liên kết đôi không kế cận nhau. Ngoài ra, còn cho
biết có hai nhóm metylen cạnh nhau, một trong hai nhóm này ở kế cận một proton của liên kết đôi.
Hãy viết các công thức cấu tạo của C thoả mãn điều kiện trên.
c) Phản ứng của C với 5-hydroxi-naphtalen-1,4-dion tạo sản phẩm ghép Diels – Alder D(C18H18O4).
Phổ NMR 1H của D cho thấy một cộng hưởng δ10,5 thống nhất với một proton và là chỉ định của
một nhóm hydroxyl liên kết nội phân tử. Hãy đề nghị ba công thức cấu tạo có thể có của hợp chất D.
d) Enol hóa D bằng cách xử lý với kali cacbonat trong metanol nóng rồi oxy hóa kế tiếp với kali
nitrosodisunfonat (muối Fremy) tạo một sản phẩm kiểu quinon màu vàng E (C18H16O4). Phổ NMR
13
C của E chứa tổng cộng 9 cộng hưởng có thể quy cho những cacbon bậc 4. Nhiệt phân E tại 180oC
trong 15 phút lại phóng thích eten bằng phản ứng ngược Diels – Alder đồng thời tạo thành F
(C16H12O4). Phổ NMR 1H của F cho thấy ba vạch đơn, mỗi vạch tương ứng với một proton (vạch
thấp nhất tại δ11,00) và hai vạch đơn 3 proton, một vạch tại δ4,01 và một vạch tại δ2,25ppm. Căn cứ
trên các chứng cứ này, đề nghị các công thức cấu tạo có thể có của các hợp chất E và F.
e) Khi cho F tác dụng với bo triclorua trong diclometan tại –10oC rồi xử lý tiếp thu được một chất rắn
màu cam, khối phổ cho m/e = 245. Trị số này giống như axit crisophanic thiên nhiên. Viết toàn bộ
công thức cấu tạo của qúa trình tổng hợp axit crisophanic.
BÀI GIẢI:
Toàn bộ qúa trình tổng hợp axit crisophanic của câu hỏi diễn ra như sau:
OCH 3 OCH 3 OCH 3

OH CH 3 OH
B C
OH O

OH O OCH 3
OH OH OCH 3

CH 3
CH 3

O
OH D

OH O OCH 3 OH O OCH 3

CH 3 OH
OH O OCH 3
O O
E F

OH

O
axit crisophanic
• Hợp chất B là sản phẩm khử Birch dự kiến
• C là một đồng phân liên hợp (tiếp cách) của B, có thể có bất cứ cấu tạo nào: C1; C2; C3.
CH3 OCH3 OCH3

CH3 CH3 CH3


C1 C2 C3
Trong những cấu tạo này, chỉ có hai cấu tạo C1 và C2 thoả các số liệu NMR. Như vậy loại C3.
• Bất kỳ ba sản phẩm Diels – Alder nào xuất phát từ C1 đến C3 cũng chấp nhận được cho lời giải
của phẩn thứ ba do chúng đều chứa nhóm hydroxyl có liên kết hydro nội phân tử. Tuy nhiên phải
nghĩ cẩn thận để xác định lời giải đúng.
Ở thời điểm này, chưa thể xác định C1 hay C2 là cấu tạo đúng của C. Tuy nhiên ở phần cuối có
thể thấy rõ lời giải xuất phát từ D1 hoặc D11, sản phẩm cộng vòng của C1 là không thể chấp nhận được
do không bị khử metyl với BCl3. Ở -10oC đây là tác nhân chọn lọc để cắt nhóm ete peri với một nhóm
cacbonyl; về mặt này thì BCl3 chọn lọc hơn BBr3
OH O OH O CH3

OH

OCH3

O CH3 O
D1 D11
Tuy nhiên ngay cả ở đây có thể có một kết qủa khác nhưng không phải chất có thể tạo axit
crisophanic. Hóa học vùng của sự cộng vòng nêu trên thực tế có thể có được, nhưng phản ứng cộng
Diels-Alder có thể tiếp diễn để tạo một đồng phân khác nêu dưới đây. Đồng phân này không dẫn đến
axit crisophanic và do cấu tạo này được cho trong câu hỏi nên có thể dễ dàng đoán được lời giải. Cũng
cần thiết để thấy rằng xét về mặt tổng hợp chất thì đây cũng không thực sự là cách không có sai lầm và
đã không thể dùng như là một chứng cớ cho cấu tạo của axit crisophanic. Với chi tiết ấy, đã có thể trả lời
câu hỏi d với hóa học vùng của sự cộng vòng C2 và 5-hidroxinaphtalen-1,4-dion.
OH O OH O

OH CH3

O CH3 O OH
OLYMPIC HÓA HỌC QUỐC TẾ 1998:
Phản ứng chuyển vị kiểu Claisen (Claisen type rearrangement) là một công cụ thuận lợi cho các
nhà hóa học trong tổng hợp chất hữu cơ. Tuy nhiên với học sinh thì phản ứng này thường khó nhận biết
và đòi hỏi suy nghĩ. Một ví dụ điển hình của phản ứng được ghi dưới đây:

R R


O O

OMe OMe
Năm 1977 một sesquiterpen furan (K) được tách từ một loại san hô mềm ở Australia là Sinularia
gonatodes. Hợp chất này có khả năng vô hiệu hóa nọc độc của ong và sự kiện một chất đơn giản như thế
lại phản ứng được như một tác nhân chống viêm gây chú ý đáng kể đến các nhà hóa tổng hợp. Một qúa
trình tổng hợp như thế được mô tả dưới đây.
Axit (A) được este hóa với 2(-trimetylsilyl)etanol [Me3SiH2CH2OH] và xeto-este sinh ra được
khử một cách chọn lọc tại nhóm cacbonyl của xeton với NaBH4/CeCl3 để tạo một hợp chất B
(C14H22O4Si).
HO2C

O
A
Xử lý B với 1,1,1 – trimetoxyetan với sự có mặt của một vết (lượng rất nhỏ) axit khan nước tạo
một chất trung gian C có thể chuyển vị kiểu Claisen khi nung nóng để tạo chất C. Phân tích phổ của C
cho thấy sự hiện diện của một nhóm este trimetylsilyletyl và một nhóm este metyl
a) Suy lụân các cấu tạo của các hợp chất B và C và chất trung gian dẫn đến C.
Khử C với liti bo hydrua tạo thành rượu bậc nhất D(C16H26O4Si) có thể bị oxy hóa với pyridin
clocromat chuyển thành F. Phản ứng của F với tác nhân Wittig G tạo thành hai đồng phân H và I. Sản
phẩm chính H có hóa học lập thể E.

Ph3P CHO
G
b) Nêu cấu tạo của các hợp chất D – I, cẩn thận để chỉ rõ hóa học lập thể chính xác của các đồng
phân H và I.
Phản ứng Wittig sau cùng với ylid chuyển hóa từ metyl triphenylphotphoni iodua tạo thành J.
Chất này sau khi cắt este trmetylsilyletyl với tetra – n – butyl amoniflorua tạo hợp chất mong muốn K.
c) Nêu các cấu tạo để hoàn chỉnh sơ đồ của qúa trình tổng hợp.
BÀI GIẢI:
Lưu ý:
• Không cần qúa quan tâm đến có chế phản ứng do câu hỏi đã có gợi ý khá cụ thể.
• Sự oxy hóa với pyridin clocromat đáng chú ý ở chỗ trong điều kiện khan nước rượu bậc 1 chỉ
bị oxy hóa thành andehit chứ không tạo thành axit cacboxylic.
• Sự chuyển đổi C thành D phải gồm có sự khử este metyl do D vẫn còn chứa silic. Nếu như
este trimetylsililetyl bị khử sẽ không còn silic.
• Cuối cùng, cũng cần lưu ý rằng bài này sử dụng cách kí hiệu cho tác nhân Wittig, nghĩa là
cấu tạo với một liên kết đôi hình thức thay vì cấu tạo lưỡng cực thường dùng cho ylid. Mặt
khác tác nhân G cũng cho phép làm tăng mạch cacbon của một andehit.
Các phản ứng xảy ra như sau:
M e 3 H 2 S iH 2 C C O 2 C M e 3 H 2 S iH 2 C C O 2 C

O O

O OH
A B

M e 3 H 2 S iH 2 C C O 2 C
M e 3 H 2 S iH 2 C C O 2 C

O C O 2M e
O C H 2O H

OH
OH
C
D

M e 3 H 2 S iH 2 C C O 2 C M e 3 H 2 S iH 2 C C O 2 C

O CHO O

OH OH
F H CHO

H O 2C
M e 3 H 2 S iH 2 C C O 2 C

O
O
OH
K OH
J

Các sản phẩm và qúa trình trung gian:


Me3H2SiH2CCO2C

CHO

O I

Me3H2SiH2CO2CC
Me3H2SiH2CO2CC

O CH3
H H+ O CH3
H
H3CO OCH3 CH3
CH3
OCH3

Me3H2SiH2CO2CC
Me3H2SiH2CO2CC

O H
O

CH3
H3C O
OCH3

H
CHẤT TRUNG GIAN CHO PHẢN ỨNG KIỂU CHUYỂN VỊ CLAISEN
OLYMPIC HÓA HỌC QUỐC TẾ 1998:
SỰ KHỬ:

(i) Li/NH3/t-BuOH (vi) HN NH


CO2Me

OCH3 Ghi chú: Di-imit: NH=NH được tạo


thành do sự oxy hóa hydrazin
(vii) O
(ii) Na/NH3/EtOH
LiAlH4/AlCl3
CO2H
O
(iii) Zn-Hg/H+ (viii) O
NaBH4

(iv) CHO
NH2NH2/OH-

(v)
CO2Me H2/Pd

SƯ OXY HÓA:
(ix) (xiv)
axit m-clopebenzoic
KMnO4/H2O/OH-
CH2Cl2
to
(xv)
(i) B2H6
(x) (ii) H2O2/OH-
H+/Cr2O72-/axeton
(xvi)

OH OsO4/Me3NO
(xi)
OH piridin clocromat(PCC)
CH2Cl2

(xii)
(i) O3
(ii) Me2S

(xiii)
(i) O3
(ii) KMnO4
CÁC PHẢN ỨNG KHÁC:
(xvii) (xxiii) O
CO2H to to
+ O
CO2H
(xviii) O
CO2H to (xxiv) OH
CH3C O CCH3/piridin
O O
HO2C
(xix) CHO
Ph3P CH2CH3
(xxv)

OH axit axetic/H+
(xx)
O
H- (xxvi)
Ph3P+ C CO2Me
PhS-

O
(xxi) OCH3
CHO (EtO)2PCH2CO2Et/NaH
(xxvii)
OCH3O
CO2Me BBr3/CH2Cl2
(xxii)
to
+
O
CO2Me OCH3

(xxviii)
CO2H anhydrit axetic/to

H3C CO2H
Ghi chú thêm: Trừ trường hợp (xiii), (xvi), còn lại học sinh phải biết được tên của chất ban đầu theo
danh pháp IUPAC. Trừ trường hợp (xiii), (xiv), (xx), (xxii), (xxiii), (xvii) còn lại thì học sinh phải biết
được tên của sản phẩm cũng theo IUPAC.
BÀI GIẢI:
SƯ KHỬ:
(i) (ii)

OCH3 CO2H

(iii) (iv)
(v) (vi)
CO2Me CO2Me
Nhớ: Chất khử XS có thể
khử anken phân cực
OH
(vii) (viii)

SỰ OXY HÓA:
(ix) (x)
COOH

COOH CO2H

(xi) (xii) H

O + O
CHO
H

(xiii) (xiv)
O O
+ CO2

(xv) (xvi)
HO
HO
H
OH H
CÁC PHẢN ỨNG KHÁC:
(xvii) O

O
(xviii): Không phản ứng trừ khi ở nhiệt độ rất cao
CO2Me
(xix) (xx)

(xxi) (xxii) O
CO2Me CO2Me

CO2Me
(xxiii) H O (xxiv)
O
O C CH3
O

H O
(xxv) (xxvi)
O CH3

O
OH
(xxvii) (xxviii)
OH O O

O
H3C
OH O
OLYMPIC HÓA HỌC QUỐC TẾ 1999:
a) Xystin (C6H12N2O4S2) là một axit diamino – dicacboxylic, là sản phẩm nhị hợp của L – xystein.
Sản phẩm nhị hợp này có thể được tách đôi khi xử lý với một thiol như mercaptoetanol
(HOCH2CH2SH) để cho L – xystein (C3H7NO2S).
(i) Viết công thức cấu tạo của xystin với cấu hình tuyệt đối
(ii) Vai trò của mercaptoetanol trong phản ứng này là gì?
1 mol xystin có thể được tách đôi nhờ xử lý với axit pefomic HCOO2H tạo thành 2 mol axit
xysteic C3H7NO5S là một axit mạnh.
(iii) Viết cấu tạo của axit xysteic tại điểm đẳng điện
(iv) Khi một peptit gồm 2 mạch A và B liên kết nhờ một liên kết đơn disunfua giữa hai gốc
xystein trong mỗi mạch được xử lý với axit pefomic thu được hai peptit mới A’ và B’ tại pH
7,0 với điện tích tổng cộng theo thứ tự bằng +5 và -3. Hãy tính điện tích tổng cộng của peptit
ban đầu tại cùng pH.
b) Khi peptit C (M = 465) được thuỷ phân hoàn toàn bằng dung dịch HCl trong nước thì trong dung
dịch sau thủy phân thu được các lượng có số mol bằng nhau của glyxin (Gly), phenylalanin
(Phe); axit aspactic (Asp), axit Glutamic (Glu) và một đương lượng amoniac (NH3).
Khi xử lý C với enzym cacboxipeptidaza thu được axit glutamic và một tripeptit. Thuỷ phân một
phần tripeptit trong axit cho một hỗn hợp sản phẩm, trong đó có hai chất được xác định là axit
glyxilaspactic (Gly – Asp) và aspactilphanylalanin (Asp – Phe).
(i) Từ các thong tin trên hãy suy ra trật tự của toàn bộ peptit C.
(ii) Điểm đẳng điện gần đúng của peptit C (pH < 7; pH ≈ 7; pH > 7) là bao nhiêu?
BÀI GIẢI:
a) (i)
NH3+

S CO2-
-
O2 C S

NH3+
(ii) tác nhân khử
(iii) Công thức cấu tạo
NH3+

SO3-
HO2C
(iv) +4
b) (i) Gly – Asp – Phe – Glu.
(iii) pH < 7
OLYMPIC HÓA HỌC QUỐC TẾ 1999:
a) Đề nghị một hay nhiều cấu tạo vòng với hóa lập thể có thể có của (D) – Tagalozơ trong dung
dịch bằng công thức chiếu Harworth:
CH2OH

HO H

HO H

H OH

CH2OH (D) – Tagalozơ


b) Hai sản phẩm với cùng công thức phân tử C6H10O6 thu được khi D – arabinozơ được cho tác
dụng với natri xianua trong môi trường axit rồi thủy phân kế tiếp cũng trong môi trường axit.
Viết cấu tạo kèm hóa học lập thể có thể có của hai hợp chất và chúng được tạo thành như thế
nào?
CHO

HO H
1) NaCN / H+
? + ?
HO H 2) H3O+/to

H OH

CH2OH
(D) - arabinozơ
c) Khi một disaccarit (có tính khử) là turanozơ được đem thủy phân, thu được D – glucozơ và D –
fructozơ với số mol bằng nhau và bằng số mol saccarit đã dùng. Metyl hóa turanozơ với metyl iodua
có mặt bạc oxit rồi thuỷ phân kế tiếp tạo thành 2,3,4,6 – tetra – O – metyl – D – fructozơ.
Hãy đề xuất cấu tạo có thể có của turanozơ mà không cần xác định hóa học lập thể tại các vị trí
anome.
BÀI GIẢI:
a) Các công thức chiếu Harworth có thể có của Tagalozơ:
CH2OH CH2OH (OH) O OH (CH2OH)
O
OH OH và OH OH
H OH (CH2OH) OH CH2OH (OH)
H H
H H
b) Các phản ứng xảy ra:
CHO CHO
CHO
H OH HO H
HO H
1) NaCN/H+ HO H + HO H
H OH
H OH H OH
H OH
H OH H OH
CH2OH
CH2OH CH2OH
CHO COOH CH2OH
H OH H OH O
H
HO H HO H -H2O H
OH H O
H OH H OH
OH
H OH H OH
H OH
CH2OH H3O+/to CH2OH
+ +
CHO COOH
H OH H OH CH2OH
HO H HO H -H2O H O
H
H OH H OH OH OH O
H OH H OH OH
CH2OH CH2OH H H

c) Công thức cấu tạo có thể có của turanozơ (không chú ý đến mặt lập thể tại các vị trí anome):
O
HO OH(CH2OH)

HOH2C OH CH2OH(OH)
O
HO
HO O

OH
OLYMPIC HÓA HỌC QUỐC TẾ 1999:
a) Hãy cho biết cách tổng hợp các hợp chất được đánh dấu bằng cách dùng bất kỳ vật liệu hữu cơ nào
nếu ban đầu không đánh dấu. Được phép dùng bất kỳ chất vô cơ cần thiết nào dù có đánh dấu hay
không
(i) 1 – D – etanol
(ii) (S) – CH3CHDCH2CH3
b) Clobenzen phản ứng với dung dịch NaOH đậm đặc trong nước dưới nhiệt độ và áp suất cao (350oC,
4500psi), nhưng phản ứng của 4 – nitroclobenzen xảy ra dễ dàng hơn (NaOH 15%, 160oC). 2,4 –
Dinitroclobenzen thuỷ phân trong dung dịch nước của natri cacbonat tại 130oC và 2,4,6 –
trinitroclobenzen thuỷ phân chỉ cần nước đun nóng. Sản phẩm của tất cả các phản ứng trên là các
phenol tương ứng.
(i) Xác định loại phản ứng trên và chỉ rõ cơ chế tổng quát của phản ứng này.
(ii) 3 – nitroclobenzen phản ứng với dung dịch hydroxit trong nước nhanh hơn hay chậm
hơn so với 4 – nitroclobenzen?
(iii) 2,4 – Dinitroclobenzen phản ứng với N – metylanilin cho một amin bậc ba, hãy viết
công thức cấu tạo của amin này.
(iv) Nếu 2,4 – dinitroflobenzen phản ứng với các chất có tính nucleophin nhanh hơn 2,4 –
dinitroclobenzen thì có thể thêm chi tiết gì vào cơ chế phản ứng trên?
BÀI GIẢI:
a)
(i)
O OH

1) LiAlD4 hay NaBD4


H3C C D
C 2) H3O+ H
H3C H 1 - D - etanol
axetandehit
(ii) H
D
1) C6H5SO2Cl/piridin
C
H3C 2) LiAlD4 C
CH2CH3 H3C
HO 3) H3O+ H
CH2CH3

(R) - 2 - butanol (S) - 2 -deuterobutan


b)
(i) Phản ứng thế nucleophin trên nhân thơm
Cơ chế tổng quát:
X Nu-
X Nu
(1) (2)
+ Nu- - + X-

E E E
X = Cl
E = NO2 (1 – 3 nhóm)
(ii) Chậm hơn
(iii) Công thức cấu tạo:
NO2 CH3
N

O2N
(iv) Theo cơ chế phản ứng ở câu (i), bước (1) chậm hơn bước (2) nên bước (1) là bước quyết định tốc
độ phản ứng.
OLYMPIC HÓA HỌC QUỐC TẾ 1999:
a) Xét hai phản ứng cộng dươi đây:
Z
R
I) (CH2)n H+
?
Nu:
Z = O; Nu: = C; N; O; S; n = 2, 3, 4

Z
II) (CH2)n H+
?
Nu:
Z = N; Nu: = C; N; O; S; n = 2, 3, 4
(i) Từ hai phản ứng có thể thu được các đồng phân lập thể nào của sản phẩm?
(ii) Trong phản ứng (I), nếu Z = O và Nu: = NH2 thì cấu tạo của sản phẩm cuối cùng như thế
nào?
b) Dự đoán (một hay nhiều) sản phẩm từ phản ứng:
HO HCHO
NHCH3
BÀI GIẢI:
a) (i)
Z-
ZH
R
R
(CH2)n (CH2)n
Z Nu Nu
R
+ Z-

I) H+ + ZH
(CH2)n
R
R
Nu:
(CH2)n (CH2)n
Nu Nu

Đồng phân đối quang (ảnh) hay anome


H

Z
Z
C
C
(CH2)n
(CH2)n
Nu
Z Nu
II) (CH2)n + Z- H+
+ Z-
Nu: C H
C
(CH2)n
(CH2)n
Nu
Nu
Đồng phân không đối quang (syn & anti)
(ii)
OH

R
(CH2)n
O NH
R R
+ OH
(CH2)n H+ + H2 O
(CH2)n
R
NH2 N
(CH2)n
NH

b)
HO HCHO HO
+
NHCH3 N N
CH3 CH3
OH
OLYMPIC HÓA HỌC QUỐC TẾ 1999:
a) Tính bazơ của một số hợp chất chứa nitơ có cấu tạo tương quan được nêu như sau:
Hợp chất Cấu tạo pKa Hợp chất Cấu tạo pKa
Piridin 5,17 Anilin NH2 4,58
N
Pirol H 0,40 Xiclohexylamin NH2 10,64
N
Pirolidin H 11,20 p-Aminopiridin NH2 9,11
N
N
Morpholin 8,33 m-Aminopiridin NH2 6,03
O NH
N
Piperidin 11,11
NH

So sánh và giải thích sự khác biệt trong tính bazơ của mỗi cặp sau:
(i) Piperiđin / piriin
(ii) Piridin / pirol
(iii) Anilin / xiclohexylamin
(iv) p – aminopiridin / piridin
(v) morpholin / piperidin
b) Sự khác biệt trong tính chất vật lý của hỗn hợp triệt quang (raxemic) axit cis – 2 – aminoxiclohexan
– 1 – cacboxylic và axit 2 – aminobenzoic được nêu trong bảng
Axit cis-2-aminoxiclohexan-1- Axit – 2 - aminobenzoic
cacboxylic
Điểm nóng chảy (oC) 240 146 – 147
Tính tan trong: nước (pH = 7) tan Không tan
HCl 0,1M rất dễ tan Không tan
NaOH 0,1M rất dễ tan Không tan
Et2O Không tan rất dễ tan
-1
Dải hấp thụ IR (thể rắn, cm ) 1610 - 1550 1690
pKa1 3,56 2,41
pKa2 10,21 4,85
(i) Đề nghị các cấu tạo hợp lý của axit cis – 2 – aminoxiclohexan -1 – cacboxylic và axit 2 -
aminobenzoictại các pH axit, trung tính và bazơ.
(ii) Nếu điểm đẳng điện được định nghĩa là pH mà tại đó phân tử có điện tích tổng cộng bằng 0
thì hãy tính điểm đẳng điện gần đúng của axit – 2 – aminoxiclohexan – 1 – cacboxylic.
BÀI GIẢI:
a)
(i) Piridin có tính bazơ yếu hơn piperidin vì đôi electron gây tính bazơ của piridin thuộc obitan sp2;
nó bị giữ chặt và không sẵn sang để cho cặp electron như trường hợp piperidin với obitan sp3.
(ii) Piridin có đôi electron (thuộc obitan sp2) sẵn sang để dùng chung với axit; trong khi pirol chỉ có
thể kết hợp với proton khi đánh đổi tính thơm của vòng.
(iii) Có hai lý do. Thứ nhất, nguyên tử nitơ trong anilin liên kết với nguyên tử cacbon ở trạng thái
lai sp2 của vòng thơm, nguyên tử cacbon này có độ âm điện mạnh hơn nguyên tử cacbon ở
trạng thái lai sp3 của xiclohexylamin. Thứ hai, các electron không liên kết có thể được phân
tán trên vòng thơm. Các công thức cộng hưởng chỉ ra rằng có sự giảm mật độ electron tại
nitơ. Vì vậy xiclohexylamin có tính bazơ mạnh hơn anilin:
NH2 NH2 NH2 NH2
(iv) Có thể xảy ra sự phân tán các electron không liên kết của nhóm –NH2 vào nhân. Hệ qủa là có
sự tăng mật độ electron trên nguyên tử nitơ của dị vòng, do đó có sự tăng tính bazơ ở vị trí
này.
NH2 NH2 NH2 NH2
N N N N

(v) Piperidin có tính bazơ mạnh hơn morpholin. Nguyên tử oxy trong morpholin có độ âm điện
lớn hơn nhóm metylen (ở cùng vị trí) của piperidin, vì thế mật độ electron trên nguyên tử
nitơ của morpholin sẽ nhỏ hơn so với piperidin.
b)
(i)
CO2H CO2- CO2-
NH3+ NH3 +
NH2

Tính axit trung tính tính bazơ


(ii)
CO2H CO2- CO2-
NH3+ NH3+ NH2

Tính axit trung tính tính bazơ


(iii) Điểm đẳng điện = (pKa1 + pKa2)/2 = 6,88
OLYMPIC HÓA HỌC QUỐC TẾ 2000:
(-)-Atractyligenin là một hợp chất hoạt động sinh học được cô lập từ cây gọi là Thistle chết choc.
Người Zulu dùng nó để chữa bệnh nhưng thường gây hiểm họa chết người. Để có thể tạo một hợp chất
tương đương có độc tính kém hơn, nay đã có nhiều phương pháp tổng hợp hóa học:

HO

OH
COOH
(-)-Atractyligenin
Br
O
B C D E G H
C5H10O C5H8O C7H12O2
A

Trong sự tổng hợp chất tương đương của (-) – Atractyligenin, hơp chất A đuợc dùng làm nguyên
liệu đầu. Trước hết cho chất A tác dụng với dung dịch natri hydroxit trong nước tạo thành B (C5H10O).
a) Hãy viết công thức cấu tạo của hơp chất B
Khi cho hợp chất B tác dụng với kali dicromat trong dung dịch nước của axit sunfuric thu được
C (C5H8O).
b) Hãy viết công thức cấu tạo của hợp chất C
Hoà tan hợp chất C trong toluene rồi sục khí hydro bromua qua dung dịch cho đến khi toàn bộ C
phản ứng hết thu được hợp chất D.
c) Hãy viết công thức cấu tạo của hợp chất D
d) Hợp chất D có đồng phân lập thể không? Nếu có, hãy cho biết có bao nhiêu đồng phân lập thể?
Cho hợp chất D tác dụng với dung dịch natri hidroxit trong etanol tạo thành hợp chất E, khi đun
nóng sẽ chuyển vị thành chất F
e) Hãy viết công thức cấu tạo của hợp chất E.
f) Hợp chất F có đồng phân lập thể không? Nếu có, hãy cho biết có bao nhiêu đồng phân lập thể?
Dung dịch hợp chất F tác dụng với kali pemanganat cho hợp chất G. Cuối cùng G được đun nóng
hồi lưu trong etanol có mặt xúc tác axit sunfuric tạo thành hợp chất H.
g) Hãy cho biết tên IUPAC của H
Hợp chất H được cho tác dụng với 2 – metylbutylmagie bromua. Sau khi axit hóa, thu được một
hỗn hợp các sản phẩm đồng phân I.
h) Hãy viết công thức cấu tạo sản phẩm I (không xét đồng phân)
i) Hãy trả lời các câu hỏi sau
(i) Số đồng phân lập thể cô lập được của sản phẩm I là 2, 4, 6 hay 8?
(ii) Tất cả các đồng phân lập thể cô lập được của sản phẩm I được tạo thành với số lượng bằng
nhau, hay một phần có số lượng khác nhau hay tất cả có số lượng khác nhau?
(iii) Trong các đồng phân lập thể cô lập được của sản phẩm I thì tất cả có cùng điểm nóng chảy
hay tất cả có điểm nóng chảy khác nhau hay một số có điểm nóng chảy giống nhau?
BÀI GIẢI:
Công thức cấu tạo các chất như sau:
Br OH O O
O O
*
Br
A B C D E F

O O * OH
OH O *
G H
I

Chất D có 2 đồng phân đối quang


Chất F có hai đồng phân hình học
(i) Chất I có 4 đồng phân lập thể riêng biệt
(ii) Cặp đồng phân không đối quang có lượng khác nhau. Cặp đối quang có lượng bằng nhau
(iii) Cặp đồng phân không đối quang có điểm nóng chảy khác nhau. Cặp đồng phân đối quang
có điểm nóng chảy bằng nhau.
OLYMPIC HÓA HỌC QUỐC TẾ 2000:
Trong sự điều chế một dược chất, có ba chất trung gian được điều chế trong các phản ứng riêng
biệt mô tả dưới đây:
a) Hãy viết công thức và tên IUPAC của sản phẩm thu được trong phản ứng sau:
1) HNO2 + H2SO4
NH2 2) CuCN + KCN

NO2
b) Hãy viết công thức theo cách không thể nhầm lẫn về mặt hóa lập thể và tên IUPAC đầy đủ của
sản phẩm thu được khi (2S) – 2 – clorobutan phản ứng với natri metoxit trong một phản ứng kiểu
SN2.
c) Hãy viết các công thức của các sản phẩm thu được trong ba giai đoạn phản ứng sau đây và cho
biết số đồng phân lập thể thu được ở giai đoạn cuối:
O O 1) H+ + HOCH2CH2OH

O 2) Dư CH3MgI trong ete, rồi thêm nước


3) H3O+
BÀI GIẢI:
a)

NH2 N2+ CN
HNO2 + H2SO4 CuCN + KCN

NO2 NO2 NO2


b)
CH3 CH3
Na+OCH3-
Cl CH2CH3 H3CH2C OCH3
H H
(2R) - 2 - Metoxibutan
c)
O O O OMgI H O OH
H++ HOCH2CH2OH O O CH3MgI (du) O O 2 O O
O O ete

H3O+
O OH

Chỉ một cấu tạo lập thể duy nhất


OLYMPIC HÓA HỌC QUỐC TẾ 2000:
Một thành phần S của dầu hoa hướng dương có cấu tạo sau:
cis
H2C OOC(CH2)7 C C (CH2)7CH3
H H
cis H2 cis
HC OOC(CH2)7 C C C C C (CH2)4CH3
H H H H

H2C OOC(CH2)18CH3
a) Có bao nhiêu đồng phân đối quang của S? Dùng dấu hoa thị (*) để chỉ ra các tâm đối xứng
gương của phân tử, nếu có.
b) S tác dụng với natri metoxit cho hỗn hợp 3 este metyl. Đề nghị tên của 3 este này. Dùng cách ghi
Z, E nếu cần.
c) Cho các este metyl chưa no tác dụng với ozon rồi với kẽm để xác định vị trí của các liên kết đôi
trong phân tử. Viết công thức cấu tạo của 4 hợp chất có nhóm chức andehit. Viết tên IUPAC của
chúng.
Chỉ số xà phòng hóa của một chất béo được định nghĩa là số miligam kali hydroxit cần để thuỷ
phân 1g chất béo. Chỉ số này được dùng để so sánh các khối lượng phân tử tương đối của chất béo.
d) Hãy tính thể tích của kali hydroxit 0,996M cần dùng để xà phòng hóa 10,0g chất béo S
e) Chỉ số xà phòng hóa của S là bao nhiêu?
Chỉ số iot của chất béo được định nghĩa là số gam iot (I2) có thể tham gia phản ứng cộng với
100g chất béo.
f) Hãy tính chỉ số iot của chất S.
BÀI GIẢI:
a) Các phân tử của chất thành phần S có hai đồng phân đối quang
cis
H2C OOC(CH2)7 C C (CH2)7CH3
H H
cis H2 cis
HC* OOC(CH2)7 C C C C C (CH2)4CH3
H H H H

H2C OOC(CH2)18CH3
b) Các công thức cấu tạo và tên gọi của este metyl là:
O
(Z) - metyloctadec-9-enoat
O
O

O (9Z;12Z) - metyloctadec-9,12-dienoat

O
metyl octadecanoat
O
c) Ozon phân S rồi tác dụng với Zn cho các andehit sau:
O O O O
nonal hexanal metan dicacbadehit

O O metyl nonalat
d) Số mol KOH đã dùng nhiêu gấp 3 lần số mol triglixerit S
n(S) = 10,0/885.402 = 11,29 (mmol)
V(KOH) = 3 . n/V = 34,0mL
e) Chỉ số xà phòng hóa = 3,40(mmol/g).56,11(g/mol) = 191
f) 112,9mmol S có chứa 3 . 112,9mmol liên kết đôi C = C
Vậy m(I2) = 0,3388(mol) . 253,8(g/mol) = 85,98g ⇒ Chỉ số iot = 86
OLYMPIC HÓA HỌC QUỐC TẾ 2001:
Axit xitric (axit-2-hydroxy-1,2,3-propantricacboxylic) là axit sơ cấp của các loại cây thuộc họ
cam, chanh và cũng góp phần làm nên vị chua đặc trưng của nó. Cách sản xuất thủ công nhất của nó là
lên men mật mía hay tinh bột, họ sử dụng loại nấm Aspergillus niger tại pH = 3,5. Axit xitric được sử
dụng rộng rãi trong công nghiệp thực phẩm, sản xuất nước ngọt và làm thuốc cắn màu trong công
nghiệp phẩm nhuộm. Ngoài ra nó cũng là một chất trung gian quan trọng trong các qúa trình sinh hóa.
a) Chất nào được sinh ra khi ta đun nóng axit xitric với axit sunfuric đặc ở 45 – 50oC. Hãy viết
công thức cấu tạo và đọc tên của sản phẩm tạo thành theo IUPAC. Loại axit hữu cơ nào tham gia
được phản ứng trên?
Sau khi đun nóng axit xitric với axit sunfuric, người ta thêm vào anisol (metoxybenzen) vào hỗn
hợp phản ứng và nhận được chất A(C12H12O5).
• A tạo anhydrit khi đun nóng với anhydrit axetic
• Để trung hoà 118mg A cần 20mL dung dịch KOH 0,05N
• Cùng một lượng chất A như trên có thể phản ứng hết được với 80mg brom để tạo thành sản
phẩm cộng
b) Xác định công thức cấu tạo A
c) Xác định các đồng phân có thể có của A sinh ra trong phản ứng trên, hãy xác định cấu dạng và
viết tên IUPAC của chúng.
d) Trong phản ứng brom hóa thì có bao nhiêu đồng phân lập thể của A được sinh ra. Hãy vẽ công
thức chiếu Fischer của chúng.
e) Xác định cấu hình tuyệt đối của các trung tâm bất đối trong các sản phẩm ở câu d
Nếu trong phản ứng hình thành A ta thay anisol bằng chất khác như phenol hay resoxinol thì
tương ứng ta thu được các chất B và C. B không cho phản ứng màu khi tác dụng với FeCl3 nhưng C thì
có. Trong cùng điều kiện phản ứng tạo thành 2 chất B, C thì hiệu suất tạo thành C cao hơn.
f) Xác định công thức cấu tạo của B và C
g) Phản ứng tạo thành A và B khác nhau ở điểm cơ bản nào?
h) Lý do tại sao hiệu suất tạo thành C cao hơn tạo thành B
BÀI GIẢI:
a) Phản ứng
H2C COOH H2C COOH

H2SO4 + H2O + CO
HO C COOH C O

H2C COOH H2C COOH


axit-1,3-pentadioic
Chỉ có các axit α - hydroxy cacboxylic mới tham gia phản ứng này
b) Khối lượng phân tử A = 236
20mL KOH 0,05M phản ứng đủ với 118mg A
1000mL KOH 1M phản ứng đủ với 118g A
⇒ A phải là axit 2 chức
Khối lượng phân tử A = 236
80mg Br2 phản ứng đủ với 118mg A
160mg Br2 phản ứng đủ với 236mg A
⇒ A có chứa một liên kết đôi
Trong phân tử A có chứa vòng anisol
Nó lại được hình thành từ axit HOOC – CH2 – CO – CH2 – COOH
Và có công thức phân tử C12H12O5
Do có sự cản trở không gian của các nguyên tử hydro trong anisol nên nhóm thế buộc phải nằm
ở vị trí para so với nhóm –OCH3. Như vậy công thức cấu tạo của chất A sẽ như sau:
COOH
H3CO
COOH
Vì A tạo được anhydrit nên hai nhóm -COOH buộc phải ở cùng phía so với liên kết đôi.
c) Các đồng phân của A
OCH3
COOH

COOH
axit-(E)-3-(2-metoxyphenyl)-2-pentendioic
OCH3
COOH

COOH
axit-(Z)-3-(2-metoxyphenyl)-2-pentendioic
HOOC

H3CO
COOH
axit-(Z)-3-(4-metoxyphenyl)-2-pentendioic
d) Hai sản phẩm khi cho A tác dụng với brom
COOH COOH
H Br Br H
HOOCH2C Br Br CH2COOH

OCH3 OCH3
1 2
Hai chất này là enantiome
e) Cấu hình tuyệt đối tại các trung tâm lập thể
COOH COOH
S R
H Br Br H
R S
HOOCH2C Br Br CH2COOH

OCH3 OCH3
1 2
f) Công thức cấu tạo các chất B và C
CH2COOH CH2COOH

O O HO O O
B C
g) Trong sự hình thành chất A từ anisol thì hướng tấn công là vị trí para so với nhóm –OCH3. Tuy
nhiên trong phản ứng tạo thành chất B từ phenol thì vị trí tấn công là ortho so với nhóm –OH. Sự
khác nhau này là do sự cản trở không gian của các nguyên tử hydro trong nhóm –OCH3. Như
vậy sự tấn công có thể xảy ra ở hai vị trí ortho và para nhưng hướng tấn công ortho được ưu tiên
hơn do sản phẩm trung gian có thể vòng hóa được để tạo sản phẩm bền B.
h) Phenol chỉ có 1 nhóm –OH trong vòng phenyl còn resoxinol thì lại có đến 2 nhóm –OH trong
vòng phenyl mà hai nhóm này lại ở vị trí meta. Điều này dẫn đến vị trí 4 trong resoxinol có mật
độ electron lớn đối với resoxinol.

5
6 4
1 3
OH HO OH
2
Như vậy thì trong cùng điều kiện phản ứng thì lượng sản phẩm C luôn nhiều hơn B
OLYMPIC HÓA HỌC QUỐC TẾ 2001:
Etylen có rất nhiều ứng dụng quan trọng trong việc sản xuất polyme và các monome quan trọng
khác. Nó được sản xuất một lượng lớn bằng phương pháp crackinh nhiệt và crackinh xúc tác các ankan
nhận được từ khí thiên nhiên và dầu mỏ.
Trong sự có mặt của xúc tác bạc, etylen phản ứng với oxy để tạo thành chất P. Chất P khi đun
nóng với H3O+ sẽ chuyển thành chất Q. Phổ cộng hưởng từ proton của P chỉ có duy nhất một tín hiệu
còn Q thì lại cho 2 tín hiệu
a) Xác định và hãy vẽ công thức cấu tạo hai chất P và Q
Chất R nhận được khi P và Q phản ứng với nhau. R phản ứng được với SOCl2 để thu được chất
S. Khi đun nóng với KOH trong ancol thì S tạo thành T.
b) Xác định công thức cấu tạo của ba chất R, S và T
Một hợp chất khác là benzen-1,4-bis(axetat) có thể được tổng hợp từ p – xylem. Qúa trình tổng
hợp benzen-1,4-bis(axetat) đi qua nhiều hợp chất trung gian khác nhau và sử dụng nhiều tác nhân khác
nhau và được cụ thể ở sơ đồ dưới đây:
COOH COOCH3
CH 3 CH 2Br CH 2CN CH 2 CH 2
? ? ? ?

? ? ? ?

CH 3 CH 2Br CH 2CN CH 2 CH 2
COOH COOCH3
p - xilen
Dimetybenzen-1,4-
bisaxetat

c) Xác định các tác nhân sử dụng trong qúa trình tổng hợp trên
d) Có bao nhiêu pic mà ta có thể nhận ra được trong phổ 1H – NMR của dimetylbenzen – 1,4 –
bisaxetat.
Khi dỉetylbanzen – 1,4 – bisaxetat (được tổng hợp từ p – xylen) và hợp chất R (nhận được từ
etylen) được đun nóng cùng nhau thì một polyme mới được hình thành.
e) Vẽ công thức của polyme mới này
f) Khi xử lý polyme này với các chất: i) KOH(aq); to, sau đó H+/H2O; ii) LiAlH4 thì chuyện gì sẽ
xảy ra?
g) Khi vô tình đun nóng lượng dư dimetylbenzen – 1,4 – bis(axetat) với glyxerol thì ta thu được
một polyme khác. Xác định công thức cấu tạo của polyme này và cho biết nó có thể dùng để kéo
sợi được không?
BÀI GIẢI:
a)
xt Ag
H 2 C CH 2 + 1/2O2 H 2C CH 2
250 oC
O
P
H 2C CH 2 H+ OH
+ H2 O HO
O
Q

b)
H 2C CH 2 OH O
+ HO HO OH
O
P Q R

O SOCl2 O
HO OH Cl Cl
R S
O KOH/ancol O
Cl Cl
S T
c)
COOH COOCH 3
CH3 CH2 Br CH2 CN CH2 CH2
NBS KCN H +/H 2O, to CH3 OH, to

peroxit EtOH/H 2O hay: i) NaOH/H2 O H+


CCl4 ii) H+ / H 2O
CH3 CH2 Br CH2 CN CH2 CH2
COOH COOCH 3
p - xilen
Dimetybenzen-1,4-bisaxetat

d) Ba tín hiệu: của nhóm –CH3; -CH2 và proton của vòng thơm
e) Cấu trúc của polyme:
O
H2 H2 H2 H2 H2 H2
H3C C C C C O C C O C C O H
O
n
f) Các phản ứng:
i) KOH/H 2O/to O
Polyme HOOCH 2C CH2 COOH + HO OH
ii) H + / H 2O

LiAlH4
O
Polyme HOH2 CH 2 C CH 2CH2 OH + HO OH

g) Nếu ta sử dụng glyxerol (một triol) sốthì liên kết giữa các mạch tăng lên. Nhóm hydroxyl
bậc hai sẽ tạo nên một hệ mạng không gian ba chiều với cấu trúc có thể như sau:
H2 C OH
HC OH
H2 C OH
Glyxerol
O H O
H2 H2 H2
C C C O C C C O C C
H2 H2
O
C O
CH2

CH2
C O
O

Polyme này không thể sử dụng để kéo sợi được do cấu trúc của nó có qúa nhiều liên kết giữa các
đoạn mạch. Tính chất đặc trưng của nó là tính chất kháng.
OLYMPIC HÓA HỌC QUỐC TẾ 2001:
Một trong số những vấn đề nan giải của hóa hữu cơ là sự tổng hợp các dẫn xuất benzen hai lần
thế thông qua phản ứng thế electrophin vào dẫn xuất benzen một lần thế. Bài này sẽ đề cập đến sự tổng
hợp Tramadol, một loại dược phẩm dùng để giảm đau được mô tả dưới đây. Bước đầu tiên của qúa trình
này là:
HSbF6
Phenol A
halogen
A cho hai pic có cường độ như nhau ở điểm có khối lượng phân tử tương ứng là 172 và 174
trong phổ khối lượng. Nó tạo ra một hỗn hợp ba dẫn xuất mononitro đồng phân khi ta nitrat hóa trong
những điều kiện êm dịu.
a) Viết công thức cấu tạo A. Đâu là sự chọn lọc hướng phản ứng trong phản ứng chuyển từ phenol thành
A? Đâu là điểm mấu chốt của phản ứng?
Xét chuỗi phản ứng sau: A ⎯(⎯ ⎯ ⎯ ⎯⎯→ B ⎯Mg
CH 3 ) 2 so4 / NaOH
⎯/⎯ ⎯⎯ ⎯→ C
THF / toluen

Phổ khối lượng của B chỉ ra hai pic có cường độ như nhau ở điểm có khối lượng phân tử tương
ứng là 186 và 188.
b) Xác định công thức cấu tạo của B và C. Khả năng phản ứng của chất B thay đổi như thế nào khi
nó chuyển thành chất C?
Một chất trung gian D cần thiết trong qúa trình tổng hợp Tramadol đã nhận được qua sơ đồ sau:
parafomandehit
Xiclohexanon D (C 9H 17 NO)
dimetylamin (tan trong HCl)

thủy phân
C + D [E] Tramadol
c) Cho biết công thức của chất D và sản phẩm Tramadol
d) Cho biết cấu hình ở các trung tâm lập thể có thể có của Tramadol
BÀI GIẢI:
a) Từ các thông tin về phổ khối lượng mà đề bài đã cho thì sản phẩm nhận được khi halogen hóa
phenol với xúc tác HSbF6 là m – bromphenol. Sự brom hóa trực tiếp vào phenol sẽ cho các dẫn
xuất ở vị trí ortho và para vì nhóm –OH là nhóm thế loại I
b) Các phản ứng xảy ra:
OH OH OCH 3 OCH3
xt HSbF6 (CH 3 )2 SO 4 Mg/THF/toluen
Br 2 NaOH
Br Br MgBr
Hợp chất B có thể tham gia được phản ứng thế nucleophin ở cacbon mang nguyên tử brom. Còn hợp
chất C thì lại có sự tồn tại của cacbanion nên sẽ tham gia phản ứng thế electrophin. Như vậy sự
chuyển hợp chất B sang hợp chất C sẽ dẫn đến sự nghịch đảo lại khả năng phản ứng.
c)
O O
CH 2O CH3
N
CH3
(CH 3 )2 NH
D
O OH
OH
CH 3
N
+ CH 3 OH
MgBr CH3
D N
C CH3

Tramadol
d)
HO PhOCH3 HO PhOCH3
N(CH3 )2 H
H N(CH3 )2

HO PhOCH3 HO PhOCH3
H N(CH3 )2
N(CH3 )2 H

Tramadol có hai nguyên tử cacbon bất đối nên có hai cặp đồng phân lập thể
OLYMPIC HÓA HỌC QUỐC TẾ 2001:
Các xetoeste là những phân tử có hai nhóm chức cực kỳ hoạt động và là những tác nhân quan
trọng trong sự tổng hợp các hợp chất có tính đặc thù lập thể cũng như các hợp chất dị vòng.
a) Xetoeste có công thức phân tử C5H8O3 có hai đồng phân X và Y. Xác định các công thức phân tử
có thể có của chúng.
Mỗi este này đều đầu tiên cho phản ứng với benzoyl bromua trong sự có mặt của CH3ONa và
sản phẩm thu được được đun nóng với 1 hay 2 đương lượng bazơ mạnh (ví dụ như liti diisopropylamit
LDA) và tiếp theo là một đương lượng CH3I
Sản phẩm của bước thứ hai này đều được thủy phân trong môi trường HCl
b) Viết các phản ứng xảy ra
c) Sản phẩm cuối của xetoeste X là một hợp chất trung tính (có công thức phân tử C11H14O) còn
xetoeste Y cho một xetoaxit (có công thức phân tử C12H14O3). Hãy giải thích sự khác nhau này.
d) Xetoeste X cho các phản ứng khác nhau tùy thuộc vào lượng LDA sử dụng. Hãy giải thích kết
qủa thu được khi:
(i) Sử dụng một đuơng lượng LDA
(ii) Sử dụng hai đương lượng LDA
BÀI GIẢI:
a) Công thức phân tử của các xetoeste là C5H8O3. Vì X và Y là xetoeste nên chúng buộc phải có
nhóm chức xeton (C – CO – ) và nhóm chức este (C – COO – ). Tổng số nguyên tử của cả hai
đơn vị này là C4O3. Như vậy chỉ có thể có hai nhóm đính với đầu este : metyl hoặc etyl.
Như vậy xetoeste ban đầu có thể có một trong ba công thức cấu tạo sau:
O O O O
H2 H2
H3 C C C C O CH3 H3 C C C C O CH 3
I II

O O

H2
H3 C C C O C CH 3
III
b) Phản ứng của các xetoeste tương ứng:
O O O O

H2 CH3ONa H
H 3C C C C O CH3 PhCH2Br H3 C C C C O CH 3
I
CH 2Ph
Xetoeste C12H 14O3
CH 3 O O CH 3 O O

1 equiv LDA H+/H 2O/to


H3 C C C C O CH 3 H3 C C C C O H
MeI
CH 2Ph CH 2Ph

Xetoaxit C12H14O3

O O O O

H2 CH3ONa H2
H 3C C C O C CH 3 PhCH2Br H2 C C C O C CH3
III
CH2 Ph

CH3 O O CH3 O O

1 equiv LDA H2 H+/H2O/t o H2


HC C C O C CH3 HC C C O C CH3
MeI
CH2 Ph CH2 Ph
Xetoeste C11H12O3
Đối với chất II:
O O

H2
H3 C C C C O CH3

NaOMe PhCH2Br

O O

2 equiv LDA H 1 equiv LDA


H3 C C C C O CH 3
MeI MeI
CH 2Ph

O O O CH 3 O
H
H2 C C C C O CH 3 H3 C C C C O CH 3

CH 3 CH 2Ph CH 2Ph

H+/H2O/to H+/H2O/to

O O O CH3 O

H
H2 C C C C O H H 3C C C C O H

CH 3 CH 2Ph CH2 Ph
β-xetoaxit β-xetoaxit

- CO2 - CO2

O O CH3

H2 H2 H2
H2 C C C C Ph H 3C C C C Ph
H

CH 3
(C11H14O) (C11H14O)
• Chất I cho sản phẩm cuối cùng là một xetoaxit có công thức phân tử là C12H14O3 ứng với công
thức của xetoaxit nhận được từ chất Y. Vậy công thức cấu tạo của chất Y là công thức I.
• Chất II cho sản phẩm cuối cùng là một chất có tính chất trung tính với công thức phân tử là
C11H14O ứng với công thức của chất nhận được từ X. Vậy công thức cấu tạo của chất X là công
thức II.
• Chất III cho sản phẩm cuối cùng là một xetoaxit có công thức C11H12O3 không hề ứng với một
trong các công thức nào ở trên.
c) Chất β - xetoeste nhận được dễ bị thủy phân trong môi trường axit ra β - xetoaxit, chất này dễ
tham gia phản ứng decacboxyl hóa qua trạng thái chuyển tiếp vòng 6 trung tâm cho ra sản phẩm
trung tính.

O O O H

H2
H2 C C C H3 C C C C CH2 Ph + CO2
O
CH
CH3 H

CH2 Ph

H2 H2
H 3C C C C CH 2Ph
d) i) Khi sử dụng một đương lượng LDA thì hợp chất X sẽ tạo ra monoanion như hình vẽ:
O O O O

H2 1 equiv LDA H
H3 C C C C O CH3 H3 C C C C O CH 3
ii) Khi sử dụng hai đương lượng LDA thì sẽ tạo ra dianion như hình vẽ:
O O O O

H2 2 equiv LDA H
H3 C C C C O CH 3 H 2C C C C O CH 3

OLYMPIC HÓA HỌC QUỐC TẾ 2002:


Lactozơ (đường sữa) được sản xuất ở hầu hết các trang trại trên khắp đất nước Hà Lan, nó được
sản xuất từ váng sữa (sản phẩm phụ trong qúa trình sản xuất phomát). Lactozơ có nhiều ứng dụng rộng
rãi trong thức ăn cho trẻ em và trong các loại dược phẩm. Nó là một disaccarit được hình thành từ một
đơn vị D – galactozơ và D – glucozơ. Công thức chiếu Haworth của nó được chỉ ra dưới đây. Phía bên
trái của công thức là đơn vị D – galactozơ.
CH 2OH OH
OH O
O
OH
OH OH
O
OH OH
1) Hãy vẽ công thức chiếu Fischer cho D – galactozơ và D – glucozơ
Sự thủy phân lactozơ trong môi trường axit dẫn đến sự tạo thành D – galactozơ và D – glucozơ.
2) Dựa vào công thức của lactozơ hãy chỉ ra:
a) Nguyên tử oxy mà ở đó sẽ có thêm proton sau khi lactozơ bị thuỷ phân
b) Liên kết C- O nào bị phá vỡ trong phản ứng thuỷ phân
c) Nguyên tử cacbon nào sẽ bị khử khi phản ứng với thuốc thử Fehling
Sự thủy phân lactozơ có thể kết hợp được với phản ứng hydro hóa khi ta sử dụng xúc tác kim
loại, điều này dẫn đến sự tạo thành ancol đa chức là sorbitol và galactitol, chúng cũng đã được biết dưới
hai cái tên tương ứng là gluxitol và dulcitol.
3) Hãy vẽ công thức chiếu Fischer của hai ancol đa chức này và cho biết chúng có hoạt động quang học
hay không?
Trong công nghiệp, qúa trình sản xuất lactozơ chịu phản ứng đồng phân hóa thành lactolozơ, đây
là một loại dược phẩm dùng để chữa các bệnh về ruột. Sự hydro hóa lactozơ dẫn đến lactitol, một poliol
– C12 với ít calori và có độ ngọt cao. Cả hai qúa trình này đều được thực hiện ở Hà Lan.
4) a) Vẽ công thức Haworth của lactolozơ (lưu ý rằng phần glucozơ trong lactozơ đã bị đồng phân hóa
thành fructozơ).
b) Vẽ công thức chiếu Haworth của lactitol.
BÀI GIẢI:
1) Công thức cấu tạo của D – galactozơ và D – glucozơ là:
CHO CHO
H OH H OH
HO H HO H
H OH HO H
H OH H OH
CH 2OH CH 2OH
D – glucozơ D – galactozơ
2)
a
c

CH 2OH OH
OH O
O
OH
OH OH
O
b
OH OH
3)
CH 2OH CH 2OH
H OH H OH
HO H HO H
H OH HO H
H OH H OH
CH 2OH CH 2OH
Sorbitol (hoạt động quang học) Galactitol (không hoạt động quang học)
4)
CH 2OH OH CH 2 OH
OH O OH O O OH
O
OH
OH CH 2OH OH OH CH 2OH
OH O
OH OH OH CH 2 OH

Lactitol Lactolose (f uranose)


CH 2OH OH
OH O CH2 OH
O OH
OH OH O

OH
Lactolose (pyranose)
OLYMPIC HÓA HỌC QUỐC TẾ 2002:
Sự phát triển của các loại dược phẩm mới phụ thuộc chủ yếu vào tổng hợp hữu cơ. Phương pháp
chỉnh lại phân tử (fine-tuning) được sử dụng để có thể nhận được các tính chất như mong muốn. Sau đây
là qúa trình tổng hợp thuốc gây mê cục bộ proparacaine (còn được gọi là proxymetacaine), chất này
được sử dụng để điều trị các bệnh về mắt.
1. Hoàn thành qúa trình tổng hựop bằng cách viết công thức cấu tạo các chất A, B, C, D và E
O

OH HNO3 n-C 3H 7 Cl SOCl2 HOCH 2CH2 N(C 2H 5) 2 H 2 /Pd(C)


A B C D E
baz
HO
Tất cả đều là các sản phẩm chính
2. Khi ta nitro hóa axit - m – hydroxybenzoic thì thu được những sản phẩm nào?
3. Khi t – C4H9Cl được sử dụng ở bước 2 thay cho C3H7Cl thì điều này sẽ dẫn tới:
a) Một sản phẩm tương tự B (nếu điều này đúng hãy viết CTCT sản phẩm)
b) Không có phản ứng
c) Sự phân hủy t – C4H9Cl
d) Một phản ứng thể SE
Hãy chọn câu trả lời đúng
BÀI GIẢI:
1) Công thức cấu tạo các chất từ A đến E:
O O O
O 2N O 2N O2 N
OH OH Cl

HO H3 CH 2 CH 2CO H 3CH2 CH 2 CO
A B C

O O
O2 N H 2N
OCH2 CH 2N(C 2 H5 )2 OCH 2CH2 N(C2 H 5) 2

H 3 CH 2CH 2CO H3 CH2 CH 2 CO


D E
2) Công thức cấu tạo các sản phẩm:
NO2 O O

OH OH
+
O2 N
OH OH
3) c
OLYMPIC HÓA HỌC QUỐC TẾ 2002:
Protein hiện diện trong hầu hết các tế bào sống và đóng một vai trò quan trọng trong hóa học của
sự sống. Nó được tạo thành từ các đơn vị cấu trúc là các axit - α - aminocacboxylic. Peptit là cac protein
“thu nhỏ” với một vài aminoaxit. Liên kết peptit là các liên kết amit được hình thành từ sự ngưng tụ của
nhóm amin của aminoaxit này với nhóm cacboxyl của aminoaxit kế cận.
1. Peptit nào nhận được từ phenylalanine F và alanin A? Chỉ ra cấu trúc của chúng.

Ph
CH3

H2 N COOH H 2N COOH H 2N COOH H2N COOH


F A G L
Trong phương pháp phân tích cấu trúc peptit thì việc nhận diện aminoaxit đầu N và đuôi C đóng
vai trò cực kỳ quan trọng. Phương pháp Sanger giúp ta nhận diện được aminoaxit đầu N bằng cách xử lý
aminoaxit với 2,4-dinitroflobenzen trong môi trường kiềm yếu, sau đó là thuỷ phân toàn bộ chuỗi peptit
với xúc tác axit. Aminoaxit đầu N sẽ tạo ra kết tủa màu vàng và dễ dàng được nhận diện bằng phương
pháp sắc ký giấy. Sanger đã được trao giải thưởng Nobel năm 1958 và 1980.
2. Viết phản ứng xảy ra khi ta sử dụng tác nhân Sanger (để cho gọn ta viết aminoaxit đầu N có công
thức là H2NR) để nhận diện aminoaxit đầu N.
Với aminoaxit đuôi C, chứa nhóm chức –COOH tự do trong peptit được phân lập bằng cách sử
dụng enzym cacboxipeptidaza để thủy phân, enzym này chỉ thủy phân aminoaxit ở cuối mạch. Đối với
một tetrapeptit chứa cac aminoaxit F, A, glyxin G và leuxin L thì phương pháp thủy phân bằng enzym
cacboxipeptidaza thì aminoaxit đuôi C được nhận diện là F. phương pháp Sanger cho biết aminoaxit đầu
N được nhận diện là G.
3. Đề nghị công thức cấu tạo của peptit. Hãy viết các công thức cấu tạo của chúng.
BÀI GIẢI:
1. Công thức của các peptit nhận được:
Ph Ph
O O O O
H H H H
N N N N
H2 N OH H 2N OH H 2N OH H 2N OH
O O O O
Ph Ph
FA AF FF AA
2. Phản ứng xảy ra:
NO 2 NO 2

O 2N F + 2H 2NR O 2N NHR + RNH 3+F-

3. Không thể biết được thứ tự hai aminoaxit giữa là AL hay LA nên peptit ban đầu có có thể có cấu
tạo như sau: GALF hoặc GLAF.
OLYMPIC HÓA HỌC QUỐC TẾ 2003:
Coniin là hợp chất rất độc được tìm thấy trong cây độc sâm (conium maculatum). Triết gia cổ đại
người Hy Lạp Socrates đã bị giết bởi chất này. Coniin là một hợp chất chứa nitơ và là một ancaloit.
Xác định hóa tính và hóa lập thể của coniin bằng cách hoàn thành các chuỗi phản ứng sau. Vẽ
CTCT A, B, C.

Hofmann exhaustive methylation: sư metyl hóa triệt để theo Hofmann


Optically active: hoạt động quang học
BÀI GIẢI:
Bước oxy hóa bằng KMnO4 tham khảo trong tài liệu: A. M. Castano, J.M. Cuerva, A. M.
Echavarren, Tetrahedron Letters, 35, 7435-7438 (1994)
OLYMPIC HÓA HỌC QUỐC TẾ 2003:
Hợp chất Cistus L có mùi thơm, có trong các nhánh cây bụi và là thành phần chính của hoa Hy
Lạp. Nó hay được tìm thấy ở đồi và các sườn dốc đá, ngoài ra nó còn được tìm thấy ở các rừng thông.
Trong y học dân gian, nhánh hoa của Citrus monospeliensis được dùng để trị bệnh suyễn còn lá được
dùng thay thế trà. Flavonoid được phân lập rất rộng rãi từ thực vật dưới dạng glycozit hay dạng tự do
aglycon (hợp chất sinh ra khi thuỷ phân glycozit). Chúng có một ứng dụng rộng rãi trong các tính chất
dược lý bao gồm: kháng vi sinh vật, kháng u, kháng độc, ức chế enzym và tăng cường các hoạt động
mạch máu.
Apigenin là một loại flavonoid hay gặp và có công thức cấu tạo như sau:
1) Viết công thức cấu tạo của các sản phẩm B và C

excess: lượng dư
2) Apigenin có tạo thành một liên kết hydro giữa nhóm hydroxyl phenolic của C5 và nhóm cacbonyl
của C4. Phổ 1H – NMR của proton phenolic ở C5 sẽ bị dịch chuyển do liên kết hydro về phía:
a) trường yếu
b) trường mạnh
c) không bị dịch chuyển
3) Khi đun nóng với dung dịch NaOH 2M, apigenin sinh ra hai sản phẩm D và E.

Hợp chất D (C6H6O3) cho phản ứng dương tính với FeCl3 và phổ 1H – NMR chỉ gồm một pic duy nhất
của hệ thơm (phổ I). Hợp chất E (C9H12O2) cũng cho phản ứng dương tính với FeCl3. Trong phổ 1H –
NMR thì phần không thơm cho hai vân ba và một pic nhiều vân trong khi đó phần không thơm chỉ gồm
một vân đôi (phổ II). Viết công thức cấu tạo của D và E
4) Sử dụng mũi tên hãy chỉ ra ba nguyên tử cacbon ở công thức C sẽ cho ra ba pic đặc trưng trong phổ
13
C – NMR khác với phổ 13C – NMR của B.
BÀI GIẢI:
1)

2) Trường yếu, bởi vì phổ 1H – NMR của proton phenolic luôn dịch chuyển về phía trường yếu khi
proton này chịu ảnh hưởng của liên kết hydro.
3) Công thức cấu tạo của các chất như sau:

4) Các nguyên tử cacbon thoả mãn yêu cầu đề bài là:


OLYMPIC HÓA HỌC QUỐC TẾ 2003:
Peptit là các polyamit mạch thẳng sinh ra bằng các liên kết “đuôi nối với đuôi” của các
aminoaxit có cấu hình L (tức S).
1) Các dipeptit nào có thể được tạo thành bằng cách ngưng tụ L – alanin và L – phenylalanin? Sử dụng
các công thức lập thể trong câu trả lời.
2) Sự kéo dài từng phần mạch peptit hầu hết luôn bắt đầu từ nguyên tử C của các aminoaxit bậc ba (sử
dụng ở dạng este) liên kết với mỗi đơn vị aminoaxit kế tiếp (sử dụng ở dạng dẫn xuất thế ở Nitơ) dẫn
đến sự thay thế nguyên tử N - thế trước khi đơn vị kế tiếp được gắn vào. Dẫn xuất thế thường được
sử dụng là các nhóm ankoxy cacbonyl ROCO – và dẫn xuất cacbamat của nó.
Hãy giải thích lý do tại sao sự hiện diện của nhóm thế (nhóm bảo vệ) của nguyên tử nitơ amin
làm trở ngại việc tạo liên kết amit với nhóm cacboxyl.
a) Vì nitơ bây giờ chỉ còn có 1H
b) Vì nhóm bảo vệ có mật độ electron ít hơn nguyên tử nitơ.
c) Vì nhóm bảo vệ chắn sự tấn công của nhóm cacbonyl
d) Vì sự kháng tĩnh điện
e) Vì nó vốn đã là một amit.
3) Vẽ các công thức cộng hưởng của một nửa nhóm amit. Sử dụng các ký hiệu lập thể và các mũi tên
để chỉ rõ sự chuyển dịch electron.
4) Tác nhân nào dưới đây sẽ được sử dụng để gắn nhóm benzylcacbamat vào một amin (nhóm
Bergmann – Zervas). Viết phản ứng.
a. C6H5CH2OCONH2, b. C6H5CH2OCO2CH3, c. C6H5CH2OCO2C(CH3)3,
d. C6H5CH2OCOCl, e. C6H5OCOCl
5) Việc loại nhóm bảo vệ ankoxycacbonyl thường kèm theo phản ứng cắt mạch dưới tác dụng của các
axit theo sơ đồ:

Xếp khả năng tăng dần tính hoạt động của các cacbamat sau đây dưới tác dụng của axit:
BÀI GIẢI:
1. Công thức cấu tạo các peptit có thể có:

Các dipeptit vòng (dixeto piperazin) cũng được chấp nhận:

2. Câu trả lời tốt nhất là 5 và 2


3.

4. Tác nhân e (benzylclofomiat) sẽ phản ứng với amin theo sơ đồ sau:

5. Nếu chúng ta giả thiết trạng thái chuyển tiếp có tạo thành ion cacboni thì chất nào tạo thành ion
cacboni dễ dàng nhất thì tính bền cũng tỉ lệ thuận với khả năng đó. Trong chất D thì có sự giải toả
electron mạnh nhất:

và khó nhất ở A:

Giải thích tương tự ta thấy cation tạo thành từ B bền hơn C. Như vậy thứ tự sẽ là: D>B>C>A
OLYMPIC HÓA HỌC QUỐC TẾ 2003:
Oleuropein (A) có công thức cấu tạo như sau với R là nhóm ankylpolyphenolic:

1) Sự thuỷ phân có xúc tác axit của oleuropin cho ra glucozơ và hai hợp chất khác gồm: polyphenolic
(A1) và một monoterpenoid (A2). Hãy sử dụng mũi tên để chỉ vào công thức của oleuropin.
a) Nguyên tử oxy sẽ bị proton hóa trong phản ứng thuỷ phân xúc tác axit để tạo ra polyphenolic A1.
b) Liên kết C – O bị phân cắt để tạo ra glucozơ.
2) Ở phổ khối của A1, pic tương ứng với ion phân tử có m/z = 154. Phổ 1H – NMR của (A1) cho dưới
đây. Nhóm proton hydroxyl đã tham gia phản ứng trao đổi nên không có mặt trong các pic trên.

Với những thông tin trên hãy chỉ ra A1 là chất nào trong ba chất sau?
3) Từ công thức cấu tạo của A1 hãy chỉ ra proton nào của A1 phù hợp với các tín hiệu trên phổ 1H –
NMR.
BÀI GIẢI:

Công thức cấu tạo đúng là C


OLYMPIC HÓA HỌC QUỐC TẾ 2003:
Người ta cho biết rằng sự cộng hợp brom vào nối đôi xảy ra theo hướng lập thể anti
Viết sản phẩm phản ứng khi ta tiến hành phản ứng cộng brom vào các anken sau. Hãy sử dụng
công thức chiếu Fischer để viết cấu tạo sản phẩm. Đánh dấu * vào các nguyên tử cacbon bất đối:
BÀI GIẢI:

Yes: có tính quang hoạt


No: không có tính quang hoạt
OLYMPIC HÓA HỌC QUỐC TẾ 2003:
Một ankin A quang hoạt có %C = 89,5% và 10,4%H. Sau khi đã hydro hóa hoàn toàn bằng Pd/C
thu được 1 – metyl – 4 – propylxiclohexan. Khi cho A phản ứng với CH3MgBr ta không thu được sản
phẩm khí. Hydro hóa A trên xúc tác Lindlar tiếp theo là ozon phân và phản ứng với KMnO4 sinh ra B,
chất này cho một pic ở 207ppm trong phổ 13C – NMR. Sản phẩm B phản ứng với I2/NaOH cho ra kết
tủa màu vàng. Lọc kết tủa, axit hóa dịch lọc thu được sản phẩm C quang hoạt không có pic ở 175ppm
trong phổ 13C – NMR. Xác định công thức cấu tạo A, B, C.
BÀI GIẢI:

OLYMPIC HÓA HỌC QUỐC TẾ 2003:


Lipaza là các enzym thuỷ phân các liên kết este trong phân tử triaxylglyxerol trong khi các
proteaza chỉ thuỷ phân các liên kết amin trong protein và peptit. Hợp chất hạn chế được sự thuỷ phân
các triaxylglyxerol và peptit có thể có ích trong việc điều trị nhiều loại bệnh tật khác nhau.
Chúng ta sẽ tiếp cận sự phát triển của việc hạn chế hiệu qủa của enzym serin proteaza bao gồm
sự thay thế liên kết amit bị đứt bằng nhóm cacboxyl đã được hoạt hóa. Kết qủa là nhóm hydroxyl ở phía
hoạt hóa của enzym sẽ phản ứng với nhóm cacbonyl đã được hoạt hóa sinh ra sản phẩm cộng bền vững
axyl enzym và chất này không bị thuỷ phân tiếp nữa.
1. Sắp xếp các nhóm sau đây theo thứ tự giảm dần hoạt tính phản ứng với nhóm hydroxyl của serin

2. Tetrahydrolipsatin là một chất kháng protein của loại enzym lipaza tiêu hóa (trong y học nó được sử
dụng để chống béo phì). Chỉ ra bằng mũi tên nhóm cacbonyl của tetrahydrolipsatin bị tấn công bởi
tâm hoạt động của serin lipaza.
3. Este và amit có thể bị thuỷ phân dưới tác dụng của axit hay bazơ. Xếp khả năng thuỷ phân của hợp
chất này theo thứ tự giảm dần khi ta thuỷ phân trong môi trường bazơ.

BÀI GIẢI:

OLYMPIC HÓA HỌC QUỐC TẾ 2004:


Loại dược phẩm được sử dụng nhiều nhất trong tất cả mọi thời đại là axit axetylsalixilic (ASS),
nó được bày bán rộng rãi trên thị trường dưới tên thương mại là aspirin để làm thuốc chống nhức đầu.
Cái tên aspirin là do một công ty ở Đức đề xuất năm 1899. ASS có thể được tổng hợp bằng con đường
sau đây:
1) Viết công thức cấu tạo của A, B, C và ASS.
2) Xác định các mệnh đề sau có liên quan đến axit salixilic là đúng, sai hay không đủ cơ sở để kết luận?
a) ASS tan nhiều trong nước có pH = 2 hơn là pH = 9
b) Phản ứng thế electrophin có thể xảy ra ở vị trí ortho của nhóm –COOH
c) Bazơ liên hợp của nó khó tan trong nước hơn axit.
d) Phổ NMR của hợp chấ này chỉ cho ta thấy được hai tín hiệu của nhóm –CH vòng thơm.
e) Phổ 1H – NMR trong D2O/DMSO cho ta một hỗn hợp gồm 5 tín hiệu.
Một trong những loại dược phẩm đầu tiên được tổng hợp là phenacetin, chất này trở nên thông
dụng bắt đầu từ năm 1888 là một loại thuốc giảm đau nhẹ. Do có phản ứng phụ nên nó đã bị cấm lưu
hành trên thị trường từ năm 1986. Phenancetin E có thể được tổng hợp từ sơ đồ sau:

Phổ 1H – NMR của E có dạng như sau:


3) Viết công thức cấu tạo các chất từ A đến E và chỉ ra các proton nào của E tương ứng với các vạch
phổ đã cho.
4) Bây giờ chúng ta sẽ so sánh axit axetylsalixilic ASS và phenancetin E, các mệnh đề sau đây là đúng
hay sai hay không đủ cơ sở để quyết định?
a) Ở pH = 9 thì phenacetin phân cực hơn ASS.
b) Cả hai chất này đều có thể bị deproton hoá bằng NaHCO3
c) Không có chất nào quang hoạt cả.
d) Trong phương pháp sắc ký bản mỏng bằng silicagel với dung môi là 5% axit axetic trong etyl
axetat thì gía trị Rf của phenacetin lớn hơn so với ASS.
BÀI GIẢI:
1) Phản ứng đầu tiên là phản ứng Kolbe – Schmidt, tiếp theo là phản ứng proton hóa, sau phản ứng
này thì axit salixilic B được hình thành. Phản ứng với anhydrit axetic sẽ dẫn đến việc tạo thành
axit axetylsalixilic ASS.

2) a.; b.; c.; d.: sai; e: đúng


3) Các phản ứng xảy ra như sau: khử nhóm nitro thành amin (A), axyl hóa (B) và tiến hành phản ứng
sunfo hóa ở vị trí para thu được C (dựa trên phổ NMR thì sản phẩm tạo thành không phải là thế ở ortho
vì điều đó sẽ dẫn đến 4 tín hiệu CH (xem phổ). Phản ứng với NaOH dưới điều kiện khắc nghiệt sẽ dẫn
đến sự hình thành phenol D và sự ete hóa theo Williamson sẽ dẫn đến sự tạo thành chất E.

Phổ NMR:
4) a,b: sai; c, d, e: đúng
OLYMPIC HÓA HỌC QUỐC TẾ 2004:
Nhóm cacbonyl C=O là một nhóm chức có khả năng tham gia nhiều phản ứng hóa học. Nó có
thể tham gia các phản ứng hình thành liên kết C – C. Phản ứng tách loại proton ở vị trí α dẫn đến sự tạo
thành enolat và sự tấn công của một nhóm nucleophin vào nguyên tử C của nhóm cacbonyl là hai phản
ứng quan trọng nhất của nhóm C=O.

Một số lượng lớn các phản ứng chọn lọc lập thể và chọn lọc vùng đã được nghiên cứu dựa trên
hai phản ứng này, đặc biệt là khi nhóm cacbonyl không đối xứng. Trong phản ứng ankyl hóa chọn lọc
vùng của 2 – metylxiclohexan (chỉ xem xét phản ứng monoankyl hóa).

LDA: liti diisopropylamit; Pr2NLi: một bazơ mạnh nhưng không mang tính nucleophin
Room temperature: nhiệt độ phòng
1) Viết công thức cấu tạo của A, B, A‘, B’ (không cần chú ý đến mặt lập thể) và dưới các điều kiện
phản ứng đã cho hãy giải thích kết qủa thu được.
2) Tại sao ta không thể sử dụng butylliti (BuLi) để làm tác nhân loại proton.
Phản ứng ankyl hóa trực tiếp các enolat thì thường không được sử dụng để ankyl hóa hợp chất
cacbonyl vì dẫn đến các sản phẩm phụ là các dẫn xuất hai hay ba lần thế. Chính vì vậy các enamin đã
được sử dụng.
3) Viết cơ chế phản ứng hình thành C và hãy chỉ ra sự chọn lọc hướng phản ứng ở đây.
4) Hãy giải thích lý do enamin phản ứng với các tác nhân electrophin bằng các cấu trúc cộng hưởng.
5) Viết công thức cấu tạo của chất D (không cần chú ý đến mặt lập thể)
Đây là phản ứng tổng hợp dẫn xuất của cumarin với xúc tác axit

6) Hãy viết công thức cấu tạo E


BÀI GIẢI:
1)

Sự deproton hóa là thuận nghịch, chịu sự khống chế nhiệt động học và tạo nên dẫn xuất enolat
nhiều lần thế.

Sự deproton hóa không thuận nghịch, khống chế động học, proton nào có tính axit cao hơn bị
loại ra và tạo nên dẫn xuất ít lần thế hơn.
2) BuLi cũng có thể phản ứng như là một tác nhân nucleophin vào nguyên tử C của nhóm cacbonyl,
chính vì vậy một bazơ không có tínnh nucleophin như LDA đã được chọn.
3) – 5) Sự hình thành của enamin:

Enamin có tính nucleophin do cặp electron không liên kết trên nguyên tử nitơ có thể đẩy electron
sang nguyên tử cacbon - β (xem cấu trúc cộng hưởng)
chịu ảnh hưởng không gian = không tồn tại ở dạng phẳng

6)

OLYMPIC HÓA HỌC QUỐC TẾ 2004:


A tạo thành B khi cho A phản ứng với một bazơ mạnh mà không có tính nucleophin, B phản ứng
với brom để tạo thành raxemic C. Sản phẩm cuối S (sản phẩm chính) và E (sản phẩm phụ) được tạo
thành bởi phản ứng giữa C với một bazơ mạnh mà không có tính nucleophin

1) Viết công thức lập thể của A ở cấu dạng bền nhất. Hãy khoanh tròn các nguyên tử có thể chuyển hóa
thành B
2) Viết công thức cấu tạo B
3) Viết công thức lập thể của C (chỉ cần vẽ một đối quang) ở cấu dạng bền nhất. Khoanh tròn các
nguyên tử có thể chuyển hóa thành D và E.
4) Viết công thức cấu tạo của D và E
BÀI GIẢI:
1) Dạng ghế là cấu dạng bền nhất của vòng xiclohexan. Kích thước nhóm thế lớn là thuận lợi cho phản
ứng tách.
Trong các phản ứng tách kiểu E2 thì các nhóm bị tách ra buộc phải không cùng ở trên một mặt
phẳng và không cùng hướng về một phía. Điều này chỉ có thể xảy ra đối với cấu dạng ghế nếu mỗi
nhóm chức đều ở vị trí axial.

2 - 4:

Bởi vì các nhóm gây ra hiệu ứng cảm ứng âm là brom nên nguyên tử hydro của nguyên tử
cacbon liên kết trực tiếp với nguyên tử brom trở nên có tính axit. Proton này dễ dàng bị tách ra dưới tác
dụng của bazơ nên D sẽ là sản phẩm chính.
OLYMPIC HÓA HỌC QUỐC TẾ 2004:
Có nhiều hợp chất có công thức phân tử C4H8O nhưng một số ít trong chúng mới có hoạt tính
quang học.
1) Chất nào có ít nhất một trung tâm bất đối? Hãy vẽ cấu trúc không gian của chúng với các trung tâm
bất đối được đánh dấu *.
2) Vài chất trong số chúng cho cấu hình (S) ở hầu hết các trung tâm bất đối. Vẽ cấu dạng tương ứng.
3) Nếu có hợp chất meso ứng với công thức này thì hãy vẽ ra cấu dạng của nó.
BÀI GIẢI:
Có 5 phân tử ứng với điều kiện đã cho, ba trong số chúng có một trung tâm bất đối và hai chất
còn lại có hai trung tâm bất đối. Một trong số chúng là đối xứng nên có hợp chất meso tồn tại
1) 2) 3)

OLYMPIC HÓA HỌC QUỐC TẾ 2004:


Một monosaccarit A có khối lượng phân tử là 150Da. Khi xử lý A với NaBH4 thì sinh ra hai
đồng phân lập thể B và C không có tính quang hoạt.
1. Vẽ công thức cấu tạo của A, B và C bằng cách sử dụng công thức chiếu Fischer.
2. Xác định cấu hình cấu hình tuyệt đối của các chất từ A đến C bằng cách sử dụng hệ danh pháp CIP
(Cahn – Ingold – Prelog).
3. Hãy xác lập mối quan hệ về mặt lập thể giữa các đồng phân quang học của B.
BÀI GIẢI:
1 – 2.

Một monosaccarit có công thức chung là Cn(H2O)n. Tất nhiên, với khối lượng phân tử là 150Da
thì công thức chỉ có thể là C5(H2O)5. Sau phản ứng khử thì B và C là hai đồng phân duy nhất không hoạt
động quang học. Hai sản phẩm này có thể được coi là tiền chất của A.
Nếu hai nhóm chỉ khác nhau ở khả năng quang học thì đồng phân R sẽ có khả năng hình thành
cao hơn đồng phân S
3.
OLYMPIC HÓA HỌC QUỐC TẾ 2004:
Epibatidin là một hoạt chất được phân lập từ một loại cóc nhiệt đới và có hiệu lực giảm đau gấp
200 lần so với morphin và không gây nghiện. Trong qúa trình tổng hợp epibatidin thì A sẽ chuyển hóa
thành B bằng phản ứng SN2 nội phân tử.

B là tiền chất của epibatidin


1) Đánh dấu * vào các trung tâm bất đối của phân tử A
2) Xác định cấu hình tuyệt đối của những vị trí đó bằng danh pháp CIP.
3) Vẽ công thức lập thể của A và hãy chỉ ra hướng phản ứng bằng mũi tên giữa các trung tâm phản ứng.
4) Vẽ công thức lập thể của B
BÀI GIẢI:
1 – 2. 3 4

Trong phản ứng thế nucleophin nội phân tử (SN2), sự tác nhân nucleophin tấn công vào trung
tâm phản ứng từ phía sau ngược với phía của nhóm đi ra. Để có thể tiếp cận được trung tâm phản ứng
thì nhóm nucleophin (nhóm amino) phải có vị trí axial và phải tấn công vào nguyên tử brom ở phía xa
nhất so với nhóm nucleophin.
OLYMPIC HÓA HỌC QUỐC TẾ 2004:
Aminoancol B là một chất trung gian quan trọng để tổng hợp Crixvan, chất này có khả năng ức
chế tiềm tàng proteaza của HIV. Các nhà hóa học của Merck muốn sử dụng epoxit A như là vật liệu ban
đầu trong qúa trình tổng hợp.
major: sản phẩm chính
minor: sản phẩm phụ
1) Sau khi xử lý A với benzyl amin trong sự có mặt của axit yếu làm chất xúc tác, họ nhận được sản
phẩm chính là aminoancol C cùng với sản phẩm phụ D, chất này có thể sử dụng như là tiền chất của
B. Viết công thức cấu tạo C và hãy đề nghị cơ chế của phản ứng này. Hãy chỉ ra hướng phản ứng và
hóa lập thể của nó.
2) Sau khi xử lý A với H2SO4 đặc và CH3CN dưới các điều kiện khống chế nhiệt động học, chỉ có E
sinh ra và sau đó ta thuỷ phân E thu được B. Viết công thức cấu tạo E và cơ chế tạo thành chất này.
Hãy chỉ ra hướng phản ứng và hóa học lập thể của nó.
BÀI GIẢI:
1)

Sự hình thành chất C (cơ chế SN2): sự tấn công từ phía sau đối với nhóm benzyl; điện tích dương
ở trạng thái chuyển tiếp của trung tâm phản ứng được ổn định hóa nhờ nhóm phenyl.
Sự hình thành C và D (cơ chế SN1): hướng phản ứng lúc này phụ thuộc vào nhóm benzyl do các
công thức cộng hưởng chỉ ra sự ổn định của cacbocation bởi nhóm phenyl.
2)
Chất F không thể được tạo thành vì điều đó dẫn đến sự biến dạng vòng
OLYMPIC HÓA HỌC QUỐC TẾ 2004:
Năm 2001, giáo sư R. Noyori nhận được giải thưởng Nobel cho các công trình về sự khử hóa
chọn lọc lập thể của các nối đôi C=C và C=O.
Bây giờ chúng ta sẽ xem xét các phản ứng đơn giản để hiểu ra một số các nguyên tố cơ bản có
thể điều khiển được sự hydro hóa chọn lọc lập thể như thế nào.
Ví dụ: β - xetoeste A (raxemic) có thể bị khử bằng hydro để tạo thành raxemic B với khả năng
chọn lọc sự tạo thành đồng phân dia (diastereoselectivity) cao trong sự có mặt của xúc tác kim loại chỉ
với phản ứng chọn lọc hóa học tấn công vào liên kết C=C. Đồng phân tinh khiết quang học β - xetoeste
C bị khử chọn lọc lập thể để tạo ra raxemic B. Thêm vào một đương lượng LiCl cực kỳ quan trọng để
hướng phản ứng theo hướng tạo ra đồng phân lập thể đã chọn.

1. Viết công thức cấu tạo của raxemic B


2. Có hai đồng phân lập thể không đối quang của hợp chất đã cho ở câu 1 có thể tạo thành hợp chất
raxemic B’ trong điều kiện này. Hãy viết công thức cấu tạo của chúng.
3. Xây dựng cơ chế cho thấy rằng phản ứng mô tả ở trên chỉ tạo ra B (không sinh B’)
BÀI GIẢI:
1.
Các đồng phân đối quang của B:
2. Các đồng phân đối quang của B’

3 Sự chọn lọc lập thể của phản ứng dehydro hóa

A và C phải bị khử bởi một trạng thái chuyển tiếp không đối xứng, chỉ C bị raxemic hóa trong
qúa trình này. Sự raxemic hóa C có thể được hiểu là do sự tạo thành enolat D1 có khả năng tồn tại lâu
hơn D2 do hiệu ứng tạo phức chelat cao ở Li+ hiện diện trong hỗn hợp phản ứng. Sự nhận nguyên tử
hydro bằng kim loại bằng cách này (cộng syn) từ đỉnh hay từ đáy của chất D1 dẫn đến sự tạo thành hỗn
hợp raxemic E1/E2 như là các đồng phân dia đơn nhất. Sự thủy phân chúng dẫn đến việc hình thành
raxemic B.
OLYMPIC HÓA HỌC QUỐC TẾ 2005:
Cacbocation là các trạng thái trung gian của phản ứng, chúng mang điện tích +1 trên nguyên tử
cacbon trung tâm. Trung tâm cacbocation là trung tâm thiếu electron và có cấu tạo phẳng. Cộng hưởng
từ proton (H-NMR) là một trong những phương pháp công cụ đầu tiên được dùng để xác định cấu trúc
và tính chất của cacbocation. Trong môi trường axit mạnh, như là SbF5, cacbocation bền có thể hình
thành và được trực tiếp theo dõi bằng NMR. SbF5 là một axit Lewis mạnh mà có thể tạo phức với một
bazơ yếu như là F- để hình thành SbF6-.
1) Cho biết sản phẩm A trong phản ứng sau đây?
F
SbF5
H3C A
CH3
H3C
2) Hai phổ H-NMR của (CH3)3CF được thu được khi lần lượt dùng (CH3)3CF tinh khiết và
(CH3)3CF trong SbF5. Một phổ, kí hiệu là phổ I, cho thấy một vạch đơn tại δ 4.35, và phổ kia
(phổ II), cho thấy một vạch đôi tại δ 1,30 với hằng số tương tác J = 20 Hz. Phổ nào là của
(CH3)3CF trong SbF5?
3) Ion tropylium B là một trong những cacbocation bền nhất. Có bao nhiêu electron π trong ion
tropylium?
H H

H H
+
H H
H
B
4) Ion tropylium B có phải là cấu trúc thơm hay không? Giải thích.
5) Độ dịch chuyển hóa học của benzen trong phổ 1H NMR là δ 7.27. Phổ H-NMR của B sẽ là:
(a) Một vạch đơn tại δ 9.17.
(b) Một vạch đơn tại δ 5.37
(c) Một vạch ba tại δ 9.17.
(d) Một vạch ba tại δ 5.37.
6) 4-Isopropyltropolon C là ví dụ đầu tiên của một hợp chất thơm không giống benzen. Nó được
giáo sư T. Nozoe, trường Đại Học Quốc Gia Đài Loan tách từ các cây bách tại Đài Loan do vào
năm 1938. Vẽ cấu trúc cộng hưởng để minh họa tính thơm của C.
O

OH

C
7) Proton của nhóm OH trong tropolon có tính axit. Ba mol tropolon C có thể tác dụng với một mol
tris(2,4-pentanedionato) sắt(III) [Fe(acac)3] để hình thành một phức có màu đỏ D. Cho biết cấu
trúc của D?
BÀI GIẢI:
1) (CH3)3C+SbF6-
2) Phổ 1: (CH3)3CF trong SbF5
3) 6e
4) Có tính thơm
5) A
6)
7) Công thức cấu tạo chất D:

OLYMPIC HÓA HỌC QUỐC TẾ 2005:


1,3,5-Hexatrien trải qua quá trình khép vòng quang hóa hình thành 1,3-xyclohexadien. Phản ứng
quang hóa này thuận nghịch và có đặc thù lập thể. Vì vậy, chiếu sáng (2E,4Z,6E)-octatrien (A) bằng tia
cực tím thì hình thành nên xyclohexadien (B). Sự lựa chọn độ dài sóng ánh sáng phụ thuộc vào cực đại
hấp thụ của hợp chất được chiếu sáng, và cực đại hấp thụ có liên quan đến số liên kết đôi liên hợp trong
một mạch.
chiÕu s¸ ng

H3C CH3 H3C CH3

A B
1) Cho biết tên gọi của hợp chất trien ban đầu (C) trong phản ứng dưới đây?
chiÕu s¸ ng
C
H3C CH3
D
Một cơ chế phản ứng tương tự liên quan đến quá trình tổng hợp các phân tử hoạt động sinh học.
Ví dụ, dưới tác dụng của ánh sáng mặt trời, 7-dehydrocholesterol (E) trải qua phản ứng mở vòng điện
vòng để tạo ra tiền vitamin D3 (F), chất này sau đó chuyển hóa qua [1,7]-hydrogen để tạo ra vitamin D3
(G).
H3C CH2)3 CH3 H3C CH2)3 CH3
( (
H3C H3C
CH3 CH3
H3C
¸ nh s¸ ng [1,7] H-shift
tiÒn vitamin D3 (F)
H H H
HO

7-dehydrocholesterol (E) vitamin D3 (G)


HO

2) Trong hai hợp chất 7-dehydrocholesterol (E) và vitamin D3 (G), hợp chất nào bạn nghĩ rằng sẽ
hấp thụ ánh sáng với năng lượng cao hơn? (E hay G)
3 Cho biết cấu trúc hóa học của F?
Nguyên lý này được nghiên cứu kỹ lưỡng để phát triển những vật liệu thay đổi màu theo ánh
sáng. Ví dụ, chiếu sáng hợp chất không màu H với tia cực tím tạo nên hợp chất có màu. Sự biến đổi màu
diễn ra ngược lại dưới tác dụng của ánh sáng khả kiến.
4) Cho biết cấu trúc của hợp chất màu I.
CH3 CH3
H3C O tia tö ngo¹i
I hî p chÊt cã mµu
O ¸ nh s¸ ng kh¶ kiÕn

H O
kh«ng mµu

Các hydrocacbon thơm thường phát huỳnh quang mạnh. Tuy nhiên, một nhóm thế amino lân cận
có thể làm tắt hình quang này. Cơ chế sự dập tắt này dựa vào sự chuyển electron khử quang (PET -
Photoinduced Electron Transfer) mà nó được minh họa rõ ràng bởi các giản đồ obitan phân tử chỉ ra
dưới đây. Dựa vào sự chiếu sáng với ánh sáng có độ dài sóng thích hợp (bước 1) thì nhóm mang màu
thơm ban đầu (trạng thái a) sẽ đẩy một electron từ HOMO đến LUMO (trạng thái b). Khi có mặt một
nhóm amoni lân cận, một trong số các electron tự do trên nguyên tử niơ sẽ di chuyển đến HOMO của
nhóm mang màu bị kích thích (bước 2), và như vậy sẽ khóa con đường phát huỳnh quang thông thường
(trạng thái c). Sự phối trí của các electron tự do amin đến proton hoặc các ion kim loại, đủ ngăn cản quá
trình PET và tái lập huỳnh quang của nhóm mang màu thơm (bước 3).
(a) (b) (c)
LUMO LUMO LUMO
hν PET

(1) (2)

amine
lone-pair
HOMO electrons HOMO HOMO
N no fluorescence

M+ (3)

(d)
LUMO

HOMO

N M

Nhiều bộ cảm biến huỳnh quang các ion kim loại hoặc proton nhạy và rất đáng quan tâm đã được
phát triển dựa trên sự vận dụng quá trình PET. Ví dụ, hợp chất J được dùng làm bộ cảm biến pH.

Cl

J
5) Theo bạn thì hợp chất J có phát huỳnh quang trong dung dịch kiềm (pH = 10.0) hay không?
BÀI GIẢI:
1) (2E, 4Z, 6Z) – octatrien
2) E
3) Cấu trúc hóa học của F:

4) Cấu trúc hóa học của I:

5) Không
OLYMPIC HÓA HỌC QUỐC TẾ 2005:
Người ta dùng công thức chiếu Fischer để mô tả sự sắp xếp không gian ba chiều của các nhóm
có liên kết với một tâm cacbon trên không gian hai chiều. Trong loại công thức này, giao điểm của hai
đường thẳng vuông góc biểu thị một tâm sp3. Đường nằm ngang nối liền B và D qua tâm cacbon thể
hiện liên kết ngoài mặt phẳng giấy (gần người quan sát). Đường thẳng đứng nối hai điểm A và C qua
tâm cacbon thể hiện liên kết nằm trong mặt giấy (xa người quan sát).
into plane
A A
D B D B out of plane
C C
1) Giáo sư Kagan đã phát triển hợp chất ChiraPhos, một chất có nhiều ứng dụng trong tổng hợp bất đối.
Dựa trên công thức chiếu Fisher dưới đây, hãy cho biết cấu hình tuyệt đối R/S của các trung tâm bất
đối trong ChiraPhos theo quy tắc hơn cấp Cahn-Ingold-Prelog.
CH3
H 2 PPh2
Ph2P 3 H
CH3
ChiraPhos
2) Một trong những đồng phân lập thể của ChiraPhos là một hợp chất meso. Xác định X và Y trong
công thức chiếu Fischer dưới đây.
H
X 2 Y
H3C 3 PPh2
H
meso-ChiraPhos
Lập thể của các hợp chất cacbohydrat thường được biểu thị bằng các công thức chiếu Fischer. Ví
dụ, công thức chiếu Fischer dưới đây biểu thị cấu trúc của D-glucozơ. Một điều thú vị là glucozơ mạch
hở có thể chuyển hóa thành cấu trúc vòng qua sự hình thành hemiaxetal giữa hai nhóm C5-OH và C1-
andehit.
O H OH
CH HO C H C
H 2 OH H OH H OH
HO 3 H HO H or HO H
H 4 OH H OH H OH
H 5 OH H O H O
CH2OH CH2OH CH2OH
D-glucose α-anomer β-anomer
(open chain) (cyclic structures)

Sự hình thành hemiaxetal sinh ra hai đồng phân lập thể, được gọi là các “anome”. α -anome
nguyên chất của D-glucozơ có góc quay cực riêng là +112.2o, trong khi β -anome có góc quay cực riêng
là +18.7o. Trong nước thì α -anome hay β -anome đều tạo ra một hỗn hợp cân bằng với góc quay cực
riêng bằng +52.6o.
3) Tính phần trăm các đồng phân α -anome trong hỗn hợp cân bằng của D-glucozơ trong nước.
4) Giữa α -anome và β -anome, anome nào bền hơn trong nước? (α hay β)
5) Vẽ cấu dạng ghế của đồng phân β-anome.
6) Cho biết hợp chất trung gian thông thường của sự chuyển hóa lẫn nhau giữa α - và β -anome?
Phản ứng cộng HCN vào một andehit tạo ra một xyanohydrin, chất này sau đó có thể bị khử
thành một α -hydroxyandehit.
O OH OH
HCN reduction
H
R H R CN R
O
cyanohydrin

Các đồng đẳng cao của cacbohydrat, như là D-talos, có thể được tổng hợp từ D-glyxerandehit
bằng sự lập lại ba lần điều kiện phản ứng giống như dưới đây.
CHO
CHO HO H
CHO 1. HCN HO H HO H
H OH H OH HO H
CH2OH 2. reduction
CH2OH H OH
CH2OH
D-glixerandehit vµ ®èi quang
D-talose
+ ®èi quang

7) Trong hỗn hợp sản phẩm cuối cùng có bao nhiêu cặp chất đồng phân đối quang?
Enzim là một chất xúc tác sinh học rất đáng quan tâm, chúng kiểm soát mô hình chuyển hóa hóa
học trong cơ thể sống. Bởi vì khả năng và đặc trưng xúc tác tốt của chúng, nên ứng dụng enzim trong
tổng hợp hữu cơ trở thành một trong những lĩnh vực lớn mạnh nhanh nhất cho sự phát triển của phương
pháp tổng hợp mới. Sau đây là dữ liệu cho phân giải động học xúc tác men của raxemic xiclohexanon
chứa nhóm thế ở vị trí số 2 theo các phản ứng Baeyer Villiger (Bảng 1).
Bảng 1. Phân giải động học xúc tác men của raxemic xiclohexanon chứa nhóm thế ở vị trí số 2 theo các
phản ứng Baeyer - Villiger
O O O
R R
men O
+
R

hçn hî p raxemic

O O
R
O
R

STT R Hiệu suất (%) ee% Hiệu suất (%) ee%

1 Et 79 95 69 98
2 n-Pr 54 97 66 92

3 Allyl 59 98 58 98

ee: lượng dư đồng phân đối quang


8) Cho biết tỷ lệ đồng phân (R)/(S) của 6-allycaprolacton trong dữ kiện số 3?
9) AMCPB (axit meta-cloperbenzoic) là một tác nhân oxi hóa thông thường cho các phản ứng Baeyer
Villiger. Sử dụng AMCPB như một tác nhân oxi hóa cho phản ứng trên, thay vì men. Cho biết
phần trăm lượng dư của sản phẩm caprolacton?
BÀI GIẢI:
1) (2S, 3S)
2) X = CH3; Y = PPh2
3) 36%
4) β
5) Công thức cấu trúc:

6) Công thức chiếu Fischer:

7) Không
8) 99:1
9) 0
OLYMPIC HÓA HỌC QUỐC TẾ 2005:
Một trong những yêu cầu ban đầu cho sự phát triển các diot phát xạ ánh sáng hữu cơ (OLED) là
sự tìm kiếm vật liệu phát quang có hiệu quả cao mà có thể là những phân tử nhỏ hoặc những hợp chất
cao phân tử. Ví dụ như floren, một biphenyl cầu nối metylen, có hiệu suất lượng tử huỳnh quang cao
hơn biphenyl.
6 5 4 3
7 2
8 1
9
Fluoren Biphenyl

Nhiều dẫn xuất floren đã được phát triển, chúng có tiềm năng ứng dụng trong công nghệ màn
hình phẳng. Để tránh sự tương tác giữa các phân tử, các nhóm thế lớn được đưa vào vị trí C9 của
fluoren. Một ví dụ của trường hợp này là hợp chất C, một phát minh về vật liệu phát xạ ánh sáng xanh
có hiệu quả cao rất thú vị và bổ ích. Dưới đây là giản đồ phản ứng tổng hợp chất này.
NH2 1) NaNO2, HCl
0-5 oC 1) Mg, Et2O HOAc, HCl
A B C (C25H16)
ng- î c dßng
2)
2) KI
O
3) H2O

1) Xác định cấu tạo của A, B, và C.

Tinh thể lỏng đã trở thành một phần cuộc sống hàng ngày của chúng ta, từ những đồng hồ đeo
tay, máy tính bỏ túi, đến màn hình phẳng màu. Các phân tử tinh thể lỏng thường chứa một trung tâm
cứng nhắc và đuôi là một mạch ankyl linh hoạt như được trình bày dưới đây.

CN

phÇn m¹ ch trung t©m


ankyl linh ho¹ t cøng nh¾c

Biphenyl và terphenyl là những cấu trúc cơ bản cho trung tâm cứng nhắc của các tinh thể lỏng.
Đây là loại cấu trúc có thể được tổng hợp có hiệu quả qua phản ứng ghép một aryl bromua hoặc iodua
với axit arylboronic có xúc tác paladi (phản ứng ghép mạch Suzuki).

Terphenyl

Một ví dụ đặc trưng của phản ứng ghép mạch Suzuki được biểu diễn dưới đây. Brombenzen tác
dụng với axit phenylboronic có mặt xúc tác paladi để tạo ra biphenyl.
Pd(0) catalyst
Br + B(OH)2

Dưới đây sơ đồ tổng hợp hai phân tử tinh thể lỏng, 4-xyano-4’-pentylbiphenyl và G.
NC C5H11

CuCN
DMF
Br2 C4H9COCl NH2NH2
D E F
AlCl3 KOH, t
F F

C8H17O B(OH)2

Pd(PPh3)4
Na2CO3
MeOCH2CH2OMe, H2O

2) Cho biết cấu trúc của D, E, F, và G?


BÀI GIẢI:
1) Công thức cấu tạo của A, B, C:

2) Công thức cấu tạo của D, E, F và G


OLYMPIC HÓA HỌC QUỐC TẾ 2005:
Các polyme hữu cơ có ảnh hưởng rất lớn đối với cuộc sống hàng ngày của chúng ta. Hàng nghìn
tấn các loại cao phân tử khác nhau được sản xuất mỗi năm. Tổng hợp các polime hữu cơ được sử dụng
trên nhiều lĩnh vực, từ nguyên liệu dệt cho đến các con chip máy tính, và đến cả van tim nhân tạo.
Chúng được sử dụng rộng rãi như chất dẻo, keo dán, vật liệu xây dựng, chất dẻo có khả năng phân hủy
và sơn. Poly (vinyl ancol) (PVA) là một ví dụ quan trọng của một polyme có khả năng hòa tan trong
nước. Giản đồ 1 dưới đây tóm tắt một phương pháp tổng hợp PVA.
Gi¶n ®å 1
polyme hãa
Monome A Polyme B Poly(vinyl ancol) (PVA)
Polyme B trên cũng là thành phần chính trong kẹo cao su. Phân tích nguyên tố chất A cho tỉ lệ
C:H:O = 56:7:37. Thêm vào đó, phân tích nguyên tố chất B cho ra thành phần C, H và O gần giống như
vậy. Dưới đây là phổ IR và 1H NMR của monome A.

Phổ 1H NMR của Monome A


110

100
3503
3094 1434

849
90 1295
977

951 876
Transmittance (%T)

80 1021

1372

70 1648

60 1138

50 1761 1217

40

30
4000 3500 3000 2500 2000 1500 1000
-1
wavenumber (cm )
Phổ IR của Monome A
1) Cho biết công thức phân tử của A?
2) Nhóm chức nào cho dải hấp thụ IR ở 1761 cm-1?
3) Cho biết cấu tạo của A?
4) Vẽ một phần polyme B. Biểu diễn ít nhất ba mắt xích.
5) Đề nghị một phương pháp chuyển hóa B thành PVA.
6) Có bao nhiêu cặp đồng phân đối quang sẽ thu được từ polyme B có khối lượng phân tử 8600, giả
sử rằng polyme đó được tắt mạch bởi sự hấp thụ hidro và bỏ qua khối lượng các nhóm cuối
mạch.
7 Hợp chất C, một đồng phân của A, cũng là một monome quan trọng trong việc tổng hợp các polime.
Dựa vào phổ 1H NMR và phổ IR được cung cấp dưới đây, lập luận xác định cấu tạo C.
Phổ 1H NMR của Monome C

100
2697 2587
3107 1945
2856 2062
35533445
3632

80
Transmittance (%T)

2999
662

854
60
2955

1634

40
812
988
1069
1439
1404
1279
1205
1731
20

4000 3500 3000 2500 2000 1500 1000


-1
wavenumber (cm )
Phổ IR của Monome C
Polyme D là một phân tử lớn nhạy axit. Khi xử lý D với một axit thì giải phóng khí E, F và hình
thành một polyme mới G. Khí E làm đục dung dịch Ca(OH)2, còn khí F tác dụng với brom tạo ra một
dung dịch không màu H.

n H+
E + F + G

dung dÞch Br2


Ca( OH) 2
O O

O vÈn ®ôc H (kh«ng mµu)


D

8) Xác định cấu tạo E, F, G và H?


Trộn polyme D với chất quang sinh axit (PAG - a photo acid-generator) tạo vật liệu quang ảnh.
Sau khi được phủ lên trên chất nền và đem ra ngoài sáng, PAG sinh proton làm xúc tác cho phản ứng
hóa học trong khuôn polyme. Nếu chiếu sáng qua một tấm mạng tạo hình (Hình 1), một ảnh của tấm
mạng sẽ hình thành trên khuôn polyme. Sau khi sấy và rửa các vật liệu có tính axit bằng nước rửa ảnh
thông thường tạo ra nền đã được trang trí I.
¸ nh s¸ ng

m¹ ng t¹ o h×nh
Polyme D + PAG
ChÊt nÒn

H×nh 1
17-9 Giản đồ nào dưới đây minh họa tốt nhất cho chất nền đã được trang trí I?
(a) (b) (c) (d)

màu sẫm thể hiện cấu trúc đã bị biến đổi so với ban đầu:
BÀI GIẢI:
1) C4H6O2
2) Nhóm C=O
3) Công thức cấu tạo A:

4) Công thức cấu tạo B:


In: chất khơi mào:
5) Các phản ứng hữu cơ có thể chuyển nhóm axetat thành ancol như là phản ứng thủy phân bằng
axit hay bazơ, phản ứng ancol phân hay khử bằng LiAlH4.
6) Có 100 đơn vị/ phân tử tuy nhiên đơn vị cuối cùng lại không mang trung tâm bất đối nên chỉ có
99 trung tâm bất đối và mỗi một trong số chúng lại có cấu hình là R hay S. Tổng cộng lại ta có
299 đồng phân quang học bao gồm cả đồng phân lập thể đối quang lẫn không đối quang. Như vậy
số các cặp đồng phân lập thể đối quang (enantiomer) sẽ là 299/2 = 298
7) Công thức cấu tạo C:

8) Công thức cấu tạo của các chất E, F, G, H:

9) hình d
OLYMPIC HÓA HỌC QUỐC TẾ 2005:
Charles Pederson đã khám phá ra ete vòng năm 1967. Jean-Marie Lehn, Donald Cram và Charles
Pederson cùng nhận Nobel Prize năm 1987 cho sự đóng góp của họ vào hóa học siêu phân tử.
Sơ đồ phản ứng dưới đây biểu diễn một phương pháp tổng hợp diol A. Tuy nhiên, do có mặt của
catechol trong các chất tham gia mà sản phẩm gồm hỗn hợp của A và sản phẩm phụ B.
OH HO
OH
Cl Cl
+ 1) NaOH
O O O
O O 2) H3O+ O
cã t¹ p chÊt lµ
catechol (c«ng A
thøc d- í i)
+
OH
B
OH
Catechol
B có thành phần nguyên tố C:H:O = 66.5:6.7:26.6 và khối lượng phân tử bằng 360. Phổ 1H NMR
của B cho thấy bốn tập tín hiệu proton. Hai trong đó được thu được tại δ 7,0-7,5 và tín hiệu kia tại δ
3,7-4,2. Tỷ lệ tích phân của bốn tập tín hiệu là 1:1:2:2. Hợp chất B liên kết bền với ion kali. Một ví dụ
thuyết phục cho điều này là việc dùng B để giúp hòa tan KMnO4 trong benzen tạo ra một chất nhuộm
màu tím.
1) Xác định cấu tạo của B.
2) Cho biết vai trò chính của H3O+ trong phản ứng trên.
(a) Để hoạt hóa ClCH2CH2OCH2CH2Cl.
(b) Để trung hòa NaOH.
(c) Để tách nhóm tetrahydropyran.
(d) Để hoạt động như một hệ đệm kiểm soát pH của dung dịch.
Dưới đây là con đường tổng hợp [2.2.2]cryptand:

O O

O O SOCl2 NH2 H2N LiAlH4


O O C D E
OH HO high dilution

O O
C 1, B2H6 O O
F N N
2. H2O
O O
[2.2.2]Cryptand

3) Xác định cấu trúc các chất từ C-F.


4) Tại sao quá trình tổng hợp D từ C yêu cầu điều kiện rất loãng?
(a) Do phản ứng giữa C và diamin thì tỏa nhiệt mạnh, dung dịch loãng được dùng
hấp thụ nhiệt thoát ra từ phản ứng.
(b) Điều kiện loãng cao được áp dụng để ngăn chặn sự hình thành polyme và oligome.
(c) Cân bằng nhiệt được ưu tiên để tạo ra D trong điều kiện rất loãng.
(d) Độ tan của nguyên liệu ban đầu nhỏ.
Ái lực với một cation kim loại được kiểm soát bởi một số yếu tố như sự tương ứng về kích thước
giữa khoảng trống của chất mang là ete vòng và của cation khách; số nguyên tử phối trí của chất mang.
Bảng 1 dưới đây cho biết bán kính của các cation kim loại kiềm và đường kính khoảng trống của một số
ete vòng.

O O O
O O O O
O O Cavity
O O
O O
O O O O O O
O O
O
12-C-4 15-C-5 18-C-6 21-C-7
Bảng 1. Bán kính của các cation kim loại kiềm và đường kính khoảng trống của một số ete vòng.
Cation (bán kính, pm) Khoảng trống của ete vòng (đường kính, pm)
Li+ (68) 12-C-4 (120-150)
Na+ (98) 15-C-5 (170-220)
K+ (133) 18-C-6 (260-320)
Cs+ (165) 21-C-7 (340-430)
5) Dựa vào cơ sở dữ liệu này, hãy lựa chọn đường cong thực nghiệm trong Hình 1 tương ứng với
các ete vòng xiclohexyl từ G đến I.
log Kf
O O
6.0 O O O
III O
O
5.0 O O O
G H
O
4.0
II
O O
3.0

O O
2.0 I
I

1.0
Na K Cs B¸n kÝnh cation
Hình 1: Khả năng tạo phức của các ete vòng với các ion kim loại trong methanol
BÀI GIẢI:
1) Công thức cấu tạo của B:

2) câu c đúng
3) Công thức cấu tạo của C, D, E và F:
4) Câu b đúng
5) Đường cong I cho chất I; đường cong II cho chất G; đường cong III cho chất H

You might also like